Vous êtes sur la page 1sur 213

S

econdE
dion
Legal Notice
This book is copyright 2016 with all rights reserved. It is illegal to copy,
distribute, or create derivative works from this book in whole or in part or
to contribute to the copying, distribution, or creating of derivative works
of this book.
320 Level 1, 2, 3, 4, and 5 AP
Calculus Problems
Dr. Steve Warner

2016, All Rights Reserved


Second edition

iii
BOOKS BY DR. STEVE WARNER FOR COLLEGE BOUND STUDENTS

28 New SAT Math Lessons to Improve Your Score in One Month


Beginner Course
Intermediate Course
Advanced Course
New SAT Math Problems arranged by Topic and Difficulty Level
New SAT Verbal Prep Book for Reading and Writing Mastery
320 SAT Math Subject Test Problems
Level 1 Test
Level 2 Test
The 32 Most Effective SAT Math Strategies
SAT Prep Official Study Guide Math Companion
Vocabulary Builder
320 ACT Math Problems arranged by Topic and Difficulty Level
320 GRE Math Problems arranged by Topic and Difficulty Level
320 SAT Math Problems arranged by Topic and Difficulty Level
320 AP Calculus AB Problems
320 AP Calculus BC Problems
555 Math IQ Questions for Middle School Students
555 Advanced Math Problems for Middle School Students
555 Geometry Problems for High School Students
Algebra Handbook for Gifted Middle School Students
SHSAT Verbal Prep Book to Improve Your Score in Two Months

CONNECT WITH DR. STEVE WARNER

iv
Table of Contents
Introduction: The Proper Way to Prepare 7
1. Using this book effectively 7
2. Overview of the AP Calculus exam 7
3. Structure of this book 8
4. Practice in small amounts over a long period of time 9
5. Redo the problems you get wrong over and over and
over until you get them right 9
6. Check your answers properly 10
7. Take a guess whenever you cannot solve a problem 10
8. Pace yourself 10

Problems by Level and Topic with Fully Explained Solutions 11


Level 1: Precalculus 11
Level 1: Differentiation 18
Level 1: Integration 26
Level 1: Limits and Continuity 34
Level 2: Precalculus 42
Level 2: Differentiation 50
Level 2: Integration 61
Level 2: Limits and Continuity 70
Level 3: Precalculus 78
Level 3: Differentiation 84
Level 3: Integration 98
Level 3: Limits and Continuity 109
Level 4: Differentiation 112
Level 4: Integration 129
Level 5: Free Response Questions 152

Supplemental Problems Questions 173


Level 1: Precalculus 173
Level 1: Differentiation 175
Level 1: Integration 176
Level 1: Limits and Continuity 177
Level 2: Precalculus 179
Level 2: Differentiation 181
Level 2: Integration 182
Level 2: Limits and Continuity 183
Level 3: Precalculus 184
Level 3: Differentiation 185

v
Level 3: Integration 187
Level 3: Limits and Continuity 188
Level 4: Differentiation 189
Level 4: Integration 193
Level 5: Free Response Questions 196

Answers to Supplemental Problems 200

About the Author 210

Books by Dr. Steve Warner 211

vi
I N T R O D U C T I O N
THE PROPER WAY TO PREPARE

here are many ways that a student can prepare for the AP
Calculus AB exam. But not all preparation is created equal. I always teach
my students the methods that will give them the maximum result with
the minimum amount of effort.
The book you are now reading is self-contained. Each problem was
carefully created to ensure that you are making the most effective use of
your time while preparing for the AP Calculus exam. By grouping the
problems given here by level and topic I have ensured that you can focus
on the types of problems that will be most effective to improving your
score.

1. Using this book effectively


Begin studying at least three months before the AP Calculus exam
Practice AP Calculus problems twenty to thirty minutes each day
Choose a consistent study time and location
You will retain much more of what you study if you study in short bursts
rather than if you try to tackle everything at once. So try to choose about
a thirty-minute block of time that you will dedicate to AP Calculus each
day. Make it a habit. The results are well worth this small time
commitment.
Every time you get a question wrong, mark it off, no matter what
your mistake.
Begin each study session by first redoing problems from previous
study sessions that you have marked off.
If you get a problem wrong again, keep it marked off.

2. Overview of the AP Calculus exam


There are four types of questions that you will encounter on the AP
Calculus exam:

7
Multiple choice questions where calculators are not allowed
(Section 1, Part A, 30 Questions, 60 Minutes).
Multiple choice questions where calculators are allowed (Section
1, Part B, 15 Questions, 45 Minutes).
Free response questions where calculators are allowed (Section
2, Part A, 2 Questions, 30 Minutes).
Free response questions where calculators are not allowed
(Section 2, Part B, 4 Questions, 60 Minutes).
This book will prepare you for all of these question types. In this book,
questions that require a calculator are marked with an asterisk (*).
If a question is not marked with an asterisk, then it could show up on a
part where a calculator is or is not allowed. I therefore recommend always
trying to solve each of these questions both with and without a calculator.
It is especially important that you can solve these without a calculator.
The AP Calculus exam is graded on a scale of 1 through 5, with a score of
3 or above interpreted as qualified. To get a 3 on the exam you will need
to get about 50% of the questions correct.
3. Structure of this book
This book has been organized in such a way to produce maximum results
with the least amount of effort. Every question that is in this book is
similar to a question that has appeared on an actual AP Calculus exam.
Furthermore, just about every question type that you can expect to
encounter is covered in this book.
The organization of this book is by Level and Topic. At first you want to
practice each of the four general math topics given on the AP Calculus
exam and improve in each independently. The four topics are Precalculus,
Differentiation, Integration, and Limits and Continuity. The first 3 Levels
are broken into these four topics. Level 4 is broken into just two of these
topics, differentiation and integration. And Level 5 mixes all the topics
together in the form of free response questions just like the ones you will
encounter in Section 2 of the exam.
Speaking of Level, you will want to progress through the 5 Levels of
difficulty at your own pace. Stay at each Level as long as you need to. Keep
redoing each problem you get wrong over and over again until you can
get each one right on your own.

8
I strongly recommend that for each topic you do not move on to the next
level until you are getting most of the questions from the previous level
correct. This will reduce your frustration and keep you from burning out.
There are two parts to this book. The first part contains 160 problems,
and the solution to each problem appears right after the problem is given.
The second part contains 160 supplemental problems with an answer key
at the very end. Full solutions to the supplemental problems are not given
in this book. Most of these additional problems are similar to problems
from the first section, but the limits, derivatives and integrals tend to be
a bit more challenging.
Any student that can successfully answer all 160 questions from either
part of this book should be able to get a 5 on the exam.

4. Practice in small amounts over a long period of time


Ideally you want to practice doing AP Calculus problems twenty to thirty
minutes each day beginning at least three months before the exam. You
will retain much more of what you study if you study in short bursts than
if you try to tackle everything at once.
So try to choose about a thirty-minute block of time that you will dedicate
to AP Calculus every night. Make it a habit. The results are well worth this
small time commitment.

5. Redo the problems you get wrong over and over and
over until you get them right
If you get a problem wrong, and never attempt the problem again, then it
is extremely unlikely that you will get a similar problem correct if it
appears on the AP exam.
Most students will read an explanation of the solution, or have someone
explain it to them, and then never look at the problem again. This is not
how you optimize your score on a standardized test. To be sure that you
will get a similar problem correct on the actual exam, you must get the
problem correct before the examand without actually remembering
the problem. This means that after getting a problem incorrect, you
should go over and understand why you got it wrong, wait at least a few
days, then attempt the same problem again. If you get it right, you can
cross it off your list of problems to review. If you get it wrong, keep
revisiting it every few days until you get it right. Your score does not
improve by getting problems correct.

9
Your score improves when you learn from your mistakes.

6. Check your answers properly


When you are taking the exam and you go back to check your earlier
answers for careless errors do not simply look over your work to try to
catch a mistake. This is usually a waste of time. Always redo the problem
without looking at any of your previous work. If possible, use a different
method than you used the first time.

7. Take a guess whenever you cannot solve a problem


There is no guessing penalty on the AP Calculus AB exam. Whenever you
do not know how to solve a problem take a guess. Ideally you should
eliminate as many answer choices as possible before taking your guess,
but if you have no idea whatsoever do not waste time overthinking.
Simply put down an answer and move on. You should certainly mark it off
and come back to it later if you have time.
Try not to leave free response questions completely blank. Begin writing
anything you can related to the problem. The act of writing can often
spark some insight into how to solve the problem, and even if it does not,
you may still get some partial credit.

8. Pace yourself
Do not waste your time on a question that is too hard or will take too long.
After youve been working on a question for about a minute you need to
make a decision. If you understand the question and think that you can
get the answer in a reasonable amount of time, continue to work on the
problem. If you still do not know how to do the problem or you are using
a technique that is going to take a very long time, mark it off and come
back to it later.
If you do not know the correct answer to a multiple choice question,
eliminate as many answer choices as you can and take a guess. But you
still want to leave open the possibility of coming back to it later.
Remember that every multiple choice question is worth the same
amount. Do not sacrifice problems that you may be able to do by getting
hung up on a problem that is too hard for you.

10
PROBLEMS BY LEVEL AND TOPIC WITH
FULLY EXPLAINED SOLUTIONS

LEVEL 1: PRECALCULUS
1. Let () = 3 and () = 4 2 3 + 2 5 + 1. Then
( )() =
(A) 3
(B) 22
(C) 10,648
(D) 3 4 6 3 + 3 2 15 + 3
Solution: ( )() = (()) = 3, choice (A).
Notes: (1) () = 3 can be read as of anything is 3. So, in particular,
of () is 3, i.e. (()) = 3.
(2) For completeness we have
( )() = (()) = ( 4 2 3 + 2 5 + 1) = 3.

(3) ( )() = (()) is called the composition of the functions and


. We literally plug the function in for inside the function . As an
additional simple example, if () = 2 and () = + 1, then
(()) = ( + 1) = ( + 1)2 and (()) = ( 2 ) = 2 + 1

Notice how, for example, ( something ) = ( something )2 .

The something here is + 1. Note how we keep it in parentheses.


3
2. What is the domain of k() = 8 12 2 + 6 3 ?
(A) All real numbers
(B) < 2
(C) 2 < < 2
(D) >2
Solution: The domain of () = 8 12 2 + 6 3 is all real numbers
3
since () is a polynomial. The domain of () = is also all real
numbers. The function () is the composition of these two functions and
therefore also has domain all real numbers. So the answer is choice (A).

11
Notes: (1) Polynomials and cube roots do not cause any problems. In
other words, you can evaluate a polynomial at any real number and you
can take the cube root of any real number.
(2) Square roots on the other hand do have some problems. We cannot
take the square root of negative real numbers (in the reals). We have the
same problem for any even root, and there are no problems for odd roots.
3
For example 8 is undefined, whereas 8 = 2 because
(2)3 = (2)(2)(2) = 8.
(3) See problem 5 for more information about polynomials.
3. If () = log 5 for > 0, then 1 () =
(A) log 5
(B) 5

(C)
5
5
(D)

Solution: The inverse of the logarithmic function () = log 5 is the


exponential function 1 () = 5 , choice (B).
Notes: (1) The word logarithm just means exponent.
(2) The equation = log can be read as is the exponent when we
rewrite with a base of . In other words we are raising to the power
. So the equation can be written in exponential form as = .
In this problem = 5, and so the logarithmic equation = log 5 can be
written in exponential form as = 5 .
(3) In general, the functions = and = log are inverses of each
other. In fact, that is precisely the definition of a logarithm.
(4) The usual procedure to find the inverse of a function = () is to
interchange the roles of and and solve for . In this example, the
inverse of = log 5 is = log 5 . To solve this equation for we can
simply change the equation to its exponential form = 5 .

12
4. If () = +1 , which of the following lines is an asymptote to
the graph of g(x) ?
(A) = 0
(B) = 0
(C) = 1
(D) = 1
Solution: The graph of = has a horizontal asymptote of = 0. To
get the graph of = +1 we shift the graph of = to the left one
unit. A horizontal shift does not have any effect on a horizontal
asymptote. So the answer is = 0, choice (B).
Notes: (1) It is worth reviewing the following basic transformations:
Let = (), and > 0. We can move the graph of around by applying
the following basic transformations.
= () + shift up units
= () shift down units
= ( ) shift right units
= ( + ) shift left units
= () reflect in -axis
= () reflect in -axis.
For the function () = +1 , we are replacing by + 1 in () = .
In other words, () = ( + 1). So we get the graph of by shifting the
graph of 1 unit to the left.
(2) The horizontal line with equation = is a horizontal asymptote for
the graph of the function = () if approaches as gets larger and
larger, or smaller and smaller (as in very large in the negative direction).
(3) We can also find a horizontal asymptote by plugging into our calculator
a really large negative value for such as 999,999,999 (if a calculator is
allowed for the problem). We get 999,999,999+1 = 0.
(Note that the answer is not really zero, but the calculator gives an answer
of 0 because the actual answer is so close to 0 that the calculator cannot
tell the difference.)
(4) It is worth memorizing that = has a horizontal asymptote of
= , and = has a vertical asymptote of = .

13
As an even better alternative, you should be able to visualize the graphs
of both of these functions. Here is a picture.

5. Which of the following equations has a graph that is symmetric


with respect to the y-axis?
(A) = ( + 1)3
(B) = ( + 1)2 1
(C) = 2 6 3 2 + 5
1
(D) =
2

Solution: The equation = 2 6 3 2 + 5 is a polynomial equation with


only even exponents. It is therefore an even function, and so its graph is
symmetric with respect to the -axis. So the answer is (C).
Notes: (1) A function with the property that () = () for all is
called an even function. Even functions have graphs which are symmetric
with respect to the -axis.
(2) A function f with the property that () = () for all is called
an odd function. Odd functions have graphs which are symmetric with
respect to the origin.

14
(3) A polynomial has the form + 1 1 + + 1 + 0 where
0 , 1 ,, are real numbers. For example, 2 6 3 2 + 5 is a
polynomial.
(4) Polynomial functions with only even powers of are even functions
(and therefore are symmetric with respect to the -axis). Keep in mind
that a constant is the same as 0 , and so is an even power of . For
example 5 is an even power of . From this observation we can see that
the polynomial in answer choice (C) is an even function.
(5) Polynomial functions with only odd powers of are odd functions (and
therefore are symmetric with respect to the origin). Keep in mind that
is the same as 1 , and so is an odd power of . An example of an odd
function is the polynomial = 4 3 + 2.
(6) Note that the functions given in answer choices (A) and (B) are also
polynomials. This can be seen by expanding the given expressions. For
example, lets look at the function given in choice (B).
( + 1)2 1 = ( + 1)( + 1) 1 = 2 + + + 1 1 = 2 + 2.
Since this expression has both an even and an odd power of , the
polynomial given in choice (B) is neither even nor odd.
I leave it as an exercise to expand ( + 1)3 and observe that it has
both even and odd powers of .
(7) A rational function is a quotient of polynomials. The function given in
choice (D) is a rational function. To determine if this function is even we
need to use the definition of being even:
()1 1 +1 1
= =
2() 2 2 2

It follows that the function is not even, and therefore its graph is not
symmetric with respect to the -axis.
(8) The graph of an even function is symmetric with respect to the -axis.
This means that the -axis acts like a mirror, and the graph reflects
across this mirror. If you put choice (C) into your graphing calculator, you
will see that this graph has this property.

15
Similarly, the graph of an odd function is symmetric with respect to the
origin. This means that if you rotate the graph 180 degrees (or
equivalently, turn it upside down) it will look the same as it did right side
up. If you put the polynomial = 4 3 + 2 into your graphing
calculator, you will see that this graph has this property.
Put the other three answer choices in your graphing calculator and
observe that they have neither of these symmetries.
(9) If we were allowed to use a calculator, we could solve this problem by
graphing each equation, and checking to see if the -axis acts like a mirror.
6. If () = 3 + 2 + + , and if (0) = 2, (1) = 7,
3
and (1) = 4, then + =
4

(A) 36
(B) 18
(C) 18
(D) It cannot be determined from the information given
Solution: Since (0) = 2, we have
2 = 03 + (0)2 + (0) + = .
Since (1) = 7, we have
7 = (1)3 + (1)2 + (1) + = 1 + + .
We already found that = 2, so we have
7 = 1 + 2 = 3.
So = 10.
Since (1) = 4, we have
4 = 13 + (1)2 + (1) + = 1 + + 2 = + 1.
So + = 5.
Lets add these last two equations.
= 10
+ = 5
2 = 15

16
15 15 5
It follows that = , and so = 5 = 5 = .
2 2 2
3 15 5 3 75 3 72
Finally, + = ( ) ( ) + = + = = 18, choice (C).
4 2 2 4 4 4 4
1
7. If the solutions of () = 0 are 3, and 5, then the solutions of
2
(3) = 0 are
1 5
(A) 1, and
6 3
3
(B) 9, and 15
2
5
(C) 6, and 2
2
7
(D) 0, and 8
2
1
Solution: We simply set 3 equal to 3, , and 5, and then solve each of
2
1 5
these equations for . We get 1, , and . So the answer is choice (A).
6 3

Notes: (1) Since 3 is a solution of () = 0, it follows that (3) = 0.


So we have (3(1)) = 0. So 1 is a solution of (3) = 0.

(2) To get that 1 is a solution of (3) = 0 formally, we simply divide


3 by 3, or equivalently, we solve the equation 3 = 3 for .
1 5
(3) Similarly, we have (3 ( )) = 0 and (3 ( )) = 0, and we can
6 3
1 5
formally find that and are solutions of (3) = 0 by solving the
6 3
1
equations 3 = and 3 = 5.
2

2 1
8. If () = and () = ln 2 , then (()) =
+2

(A) 0.12
(B) 0.50
(C) 0.51
(D) 0.75
22 1
Solution: () = ln 2 = 2. So (()) = (2) = = 0.75, choice (D).
2+2

Notes: (1) We first substituted into the function to get 2. We then


substituted 2 into the function to get 0.75.

17
(2) See the notes at the end of problem 3 for more information on
logarithms.
(3) There are several ways to compute ln 2 .
Method 1: Simply use your calculator (if allowed).
Method 2: Recall that the functions and ln are inverses of each other.
This means that ln = and ln = . Substituting = 2 into the second
equation gives the desired result.
Method 3: Remember that ln = log . So we can rewrite the equation
= ln 2 in exponential form as = 2 . So = 2.
Method 4: Recall that ln = 1. We have ln 2 = 2 ln = 2(1) = 2. Here we
have used the last law in the following table:
Laws of Logarithms: Here is a review of the basic laws of logarithms.
Law Example
logb1 = 0 log21 = 0
logbb = 1 log66 = 1
logbx + logby = logb(xy) log57 + log52 = log514

logbx logby = logb( ) log321 log37 = log33 = 1

logbxn = nlogbx log8 35 = 5log83

LEVEL 1: DIFFERENTIATION
9. If () = 2 + cos , then () =
(A) 2 + 1 sin
(B) 2 + 1 + sin
(C) 2 sin
1 1
(D) 3 + 2 sin
3 2

Solution: () = 2 + 1 + sin . This is choice (B).


Notes: (1) If is any real number, then the derivative of is 1 .

Symbolically, [ ] = .


For example, [ 2 ] = 2 1 = 2.

18

As another example, [ ] = [ 1 ] = 1 0 = 1(1) = 1.


(2) Of course it is worth just remembering that [ ] = 1.

(3) You should know the derivatives of the six basic trig functions:

[ ] = [ ] =


[ ] = [ ] =


[ ] = [ ] =

(4) If and are functions, then ( + ) () = () + ().


In other words, when differentiating a sum, we can simply differentiate
term by term.
Similarly, ( ) () = () ().
(5) In the given problem we differentiate each of 2 , , and cos
separately and then use note (4) to write the final answer.
+2
10. If () = , then (2) =
2
1
(A)
4
(B) 1
(C) 1
1
(D)
4
(2)(1)(+2)(1) 22 4
Solution: () = (2)2
= (2)2
= (2)2. So we have
4 4 4 1
(2) = (22)2 = (4)2
= = , choice (A).
16 4

Notes: (1) We used the quotient rule which says the following:
()
If f () = , then
()

() () ()()
() =
[()]2
I like to use the letters for numerator and D for denominator.

19
(2) The derivative of + 2 is 1 because the derivative of is 1, and the
derivative of any constant is 0.
Similarly, the derivative of 2 is also 1.
(3) If we could use a calculator for this problem, we can compute (2)
using our TI-84 calculator by first selecting nDeriv( (or pressing 8) under
the MATH menu, then typing the following: (X + 2)/(X 2), X, 2 ), and
pressing ENTER. The display will show approximately .25.
1
11. If () = 3 2 ln + , then () =
12
1 2 1
(A) 2 +
3 2
3
1 2 2
(B) 2 + 2
4 ln 3
1 2 2 1
(C) +
4 2
1 2 1
(D) 2 +
4 ln 2
1
1 1 1 1 2 1
Solution: () = 3 2 2 ( ) + 2 = 2 + , choice (C).
12 2 4 2

Notes: (1) The derivative of a constant times a function is the constant


times the derivative of the function.

Symbolically, [()] = [()].

1 1 1 1
For example, [ 3] = [ 3 ] = (3 2 ) = 2 .
12 12 12 4
1
(2) The derivative of ln is .

1
Symbolically, [ln ] = .

1 2
(3) Combining (1) and (2), we have [2 ln ] = 2 [ln ] = 2 ( ) = .

1 1
1
(4) can be written as 2 . So the derivative of is 2 .
2
1
1 1
(5) 2 = 1 = .
2

1 1
1 1
(6) Combining (4) and (5) we have [] = [ 2 ] = 2 = .
2 2

20
(7) Here is a review of the laws of exponents:

Law Example
x0 = 1 30 = 1
x1 = x 91 = 9
xaxb = xa+b x3x5 = x8
xa/xb = xa-b x11/x4 = x7
(xa)b = xab (x5)3 = x15
(xy)a = xaya (xy)4 = x4y4
(x/y)a = xa/ya (x/y)6 = x6/y6
x-1 = 1/x 3-1 = 1/3
x-a = 1/xa 9-2 = 1/81
3
x1/n = x1/3 =
9
xm/n = =( ) x9/2 = 9 =()

12. The slope of the tangent line to the graph of = 3 at = ln 2


is
(A) 8 ln 2
(B) 8
(C) 24 ln 2
(D) 24
Solution: = 3 (3) = 3 3 . When = ln 2, we have that the slope of
the tangent line is
3
|=ln 2 = 3 3 ln 2 = 3 ln 2 = 3 ln 8 = 3(8) = 24.
This is choice (D).
Notes: (1) To find the slope of a tangent line to the graph of a function,
we simply take the derivative of that function. If we want the slope of the
tangent line at a specified -value, we substitute that -value into the
derivative of the function.
(2) The derivative of () = is () =
(3) In this problem we used the chain rule which says the following:
If () = ( )() = (()) , then

() = (()) ()

Here we have () = 3 and () = . So () = 3, and

21
(()) = () = 3 .

(4) See the notes at the end of problem 3 for information on logarithms.
(5) ln is an abbreviation for log .
(6) The functions and ln are inverses of each other. This means that
ln = and ln = .
(7) ln = ln
3
(8) Using notes (6) and (7) together we get 3 ln 2 = ln 2 = ln 8 = 8.
(9) See the notes at the end of problem 8 for a review of the laws of
logarithms.
(10) If we could use a calculator for this problem, we can compute at
= ln 2 using our TI-84 calculator by first selecting nDeriv( (or pressing
8) under the MATH menu, then typing the following: e^(3X), X, ln 2), and
pressing ENTER. The display will show approximately 24.
13. The instantaneous rate of change at = 3 of the function
() = + 1 is
1
(A)
4
3
(B)
4
5
(C)
4
11
(D)
4
1
1
Solution: () = ( + 1)2 + + 1(1) = + + 1.
2 2+1
3 3 3 3 8 11
So (3) = + 3 + 1 = + 4 = +2= + = , choice
23+1 24 22 4 4 4
(D).
Notes: (1) We used the product rule which says the following:
If f () = ()() , then
() = () () + () ()
(2) The derivative of is 1.

22
1
(3) + 1 can be written as ( + 1)2 . So the derivative of + 1 is
1
1
( + 1)2 (technically we need to use the chain rule here and also take
2

the derivative of + 1, but [ + 1] is just 1).

1
1 1
(4) ( + 1)2 = 1 = .
(+1)2 +1

(5) Combining (3) and (4) we have


1 1
1 1
[ + 1] = [( + 1)2 ] = ( + 1)2 = .
2 2+1

(6) See problem 11 for a review of all of the laws of exponents you should
know.
(7) If we can use a calculator for this problem, we can compute (3) using
our TI-84 calculator by first selecting nDeriv( (or pressing 8) under the
MATH menu, then typing the following: X(X+1), X, 3), and pressing
ENTER. The display will show approximately 2.75. Type 2.75, then press
11
MATH ENTER ENTER to change this decimal to .
4
1
14. [ 5 + 3 + 11 ] =
2

Solution:
5
1 2 2
[ 5 + 3 + 11 ] = 0 3 + 11 (ln 11) = 3 + (ln 11)11 .
2 3 3 5


Notes: (1) 5 is a constant. Therefore [ 5 ] = 0.

2 2 2 5
1 1 1 2 2
(2) 3 = 2 = 3 . So [3 ]= [ 3 ] = 31 = 3 .
2 3
2 3 3

5
2 2 2
(3) 3 = 5 = 3 .
3 3 5
3 3

(4) If > 0, then [ ] = (ln ).


In particular, [11 ] = 11 (ln 11).

(5) For > 0, = ln .

23

To see this, first observe that ln = ln by the power rule for
logarithms (see problem 8 for the laws of logarithms).
Second, recall that the functions and ln are inverses of each other.
This means that ln = and ln = . Replacing by in the first

formula yields ln = .
(6) The formula in note (5) gives an alternate method for differentiating
11 . We can rewrite 11 as ln 11 and use the chain rule. Here are the
details:

[11 ] = [ ln 11 ] = ln 11 (ln 11) = 11 (ln 11).

Note that in the last step we rewrote ln 11 as 11 .


(7) There is one more method we can use to differentiate 11 . We can
use logarithmic differentiation.
We start by writing = 11 .
We then take the natural log of each side of this equation: ln = ln 11 .
We now use the power rule for logarithms to bring the out of the
exponent: ln = ln 11.
1
Now we differentiate implicitly to get = ln 11.


Solve for by multiplying each side of the last equation by to get

= ln 11.


Finally, replacing by 11 gives us = 11 (ln 11).

(8) Logarithmic differentiation is a general method that can often be used


to handle expressions that have exponents with variables.
(9) See problem 45 for more information on implicit differentiation.
15. If = cos , then =
Solution: We take the natural logarithm of each side of the given equation
to get ln = ln cos = (cos )(ln ).
We now differentiate implicitly to get

24
1 1 cos (ln )(sin )
= (cos ) ( ) + (ln )( sin ) =

Multiplying each side of this last equation by yields


cos (ln )(sin ) cos (ln )(sin )
= [ ] = cos [ ]

Notes: (1) Since the exponent of the expression cos contains the
variable , we used logarithmic differentiation (see problem 14 for more
details).
1 1
(2) (cos ) ( ) + (ln )( sin ) = (cos ) ( ) + ( ) [(ln )( sin )]

cos (ln )( sin ) cos (ln )(sin )
= + = .

(3) Remember to replace by cos at the end.


(4) See problem 45 for more information on implicit differentiation.
cot 3
16. Differentiate () = and express your answer as a simple

fraction.
Solution:
1
( cot 3 )( csc2 3)(3) cot 3 (2) 6(csc2 3) cot 3 cot 3
() = = .
2

Notes: (1) [ ] =

[cot ] = csc 2

[3 ] = 3

1 1
1 1 1 1 1 1
[] = [ 2 ] = 2 = 1 = =
2 2 2 2
2

(2) We start off using the quotient rule (see problem 10 for a detailed
explanation of the quotient rule). Here we get

[ cot 3 ] cot 3 []
2
()

25

(3) [ cot 3 ] requires two applications of the chain rule. See problem 12

for a detailed explanation of the chain rule. Here we get

[ cot 3 ] = cot 3 ( csc 2 3)(3).

(4) After differentiating we wind up with a complex fraction:


1
( cot 3 )( csc2 3)(3) cot 3 (2)

We simplify this complex fraction by multiplying the numerator and


denominator by 2.
Note the following:

(2) = 2 (this is where the final denominator comes from).


( cot 3 )( csc 2 3)(3)(2) = 6 (csc 2 3) cot 3 = 6 (csc 2 3) cot 3
1
cot 3 ( ) (2) = cot 3
2

The last two results together with the distributive property give the final
numerator.

LEVEL 1: INTEGRATION
17. ( 4 6 2 + 3) =
(A) 5 5 18 3 + 3 +
(B) 4 3 12 + 3 +
5
(C) 3 2 + 3 +
4
5
(D) 2 3 + 3 +
5

Solution:
5 6 3 5
( 4 6 2 + 3) = 5

3
+ 3 + =
5
2 3 + 3 + .

This is choice (D).


+1
Notes: (1) If is any real number, then an antiderivative of is .
+1

26
+1
Symbolically, = + , where is an arbitrary constant.
+1

5
For example, 4 = + .
5

3 1
As another example, 3 = 3 0 = + = 3 + .
1

(2) Of course it is worth just remembering that = for any


constant .
(3) If and are functions, then

[() + ()] = () + ().


In other words, when integrating a sum we can simply integrate term by
term.
Similarly, [() ()] = () ().
(4) If is a function and is a constant, then

() = ()

3
For example, 6 2 = 6 2 = 6 ( ) + = 2 3 + .
3

(5) In the given problem we integrate each of 4 , 2 , and 3 separately and


then use notes (3) and (4) to write the final answer.
(6) We do not need to include a constant for each individual integration
since if we add or subtract two or more constants we simply get a new
constant. This is why we simply add one constant at the end of the
integration.
(7) It is also possible to solve this problem by differentiating the answer
choices. For example, if we start with choice (C), then we have that
5 5 4
( )= . So we can immediately see that choice (C) is incorrect.
4 4

When we differentiate choice (D) however, we get


5 5 4
[ 2 3 + 3 + ] = 3(2 2 ) + 3 + 0 = 4 6 2 + 3.
5 5

This is the integrand (the expression between the integral symbol and
) that we started with. So the answer is choice (D).

27

(8) Note that the derivative of any constant is always 0, ie. [ ] = 0.

2
18. 1(3 2 2) =

(A) 2
(B) 4
(C) 6
(D) 14
2
2 3 3 2 2 2
Solution: 1(3 2 2) = ( )| = ( 3 2 ) |1 =
3 2 1
(23 22 ) ((1)3 (1)2 ) = (8 4) (1 1) = 4 (2) = 6.
This is choice (C).

Notes: (1) () = () () where is any antiderivative of .

In this example, () = 3 2 is an antiderivative of the function


2
() = 3 2 2. So 1 () = (2) (1)

(2) We sometimes write () () as () | .

This is just a convenient way of focusing on finding an antiderivative


before worrying about plugging in the upper and lower limits of
integration (these are the numbers and , respectively).
(3) For details on how to find an antiderivative here, see the notes at the
end of the solution to problem 17.
19. 3 3 =
(A) 3 +
(B) +
(C) +
(D) 3 +
Solution: 3 3 = 3 (3) = 3 + , choice (D).
Notes: (1) We can formally make the substitution = 3. It then follows
that = 3. So we have

3 (3) = = + = 3 + .

28
We get the leftmost equality by replacing 3 by , and 3 by .
We get the second equality by the basic integration formula

= + .
And we get the rightmost equality by replacing with 3.
(2) Note that the function () = 3 can be written as the composition
() = (()) where () = and () = 3.
Since () = 3 is the inner part of the composition, it is natural to try
the substitution = 3.
Note that the derivative of 3 is 3, so that = 3.
(3) With a little practice, we can evaluate an integral like this very quickly
with the following reasoning: The derivative of 3 is 3. So to integrate
3 3 we simply pretend we are integrating but as we do it we leave
the 3 where it is. This is essentially what was done in the above solution.
Note that the 3 goes away because it is the derivative of 3. We need it
there for everything to work.
(4) We can also solve this problem by differentiating the answer choices.

In fact, we have [ 3 + ] = 3 (3) + 0 = 3 3 . So the answer is

choice (D).

20. ( 2 + 2) =
3
(A) 2 + 4 2 + +
7 3
2 4
(B) 2 + 2 +
7 3
5 1
2
(C) + 2 2 +
2
5
5 2
(D) + +
2

Solution:
1 1
( 2 + 2) = ( 2 + 2) = 2 2 + 2 2
5 1 7 3
2 4
= 2 + 2 2 = 2 + 2 +
7 3

29
This is choice (B).
1 1
Notes: (1) = 2 . So 2 = 2 2 .
1 1 4 1 5
(2) 2 2 = 2+2 = 2+2 = 2 .
(3) See problem 11 for more information on the laws of exponents used
here.
+1
(4) To integrate we used the power rule twice: = + .
+1
3
1 3 3 3
2 3 2 2
=
2 3 + = 2 + = 2 + = 2 + .
2 3 3
2

1 1 3 3
2 4
It follows that 2 2 = 2 2 = 2 2 + = 2 + .
3 3
7
5 7 7 7
2 7 2 2
2 = 7 + = 2 + = 2 + = 2 + .
2 7 7
2

2 2 3
21. 0 ( 2 4) 6 =
16
(A)
3
(B) 0
16
(C)
3
1 16
(D)
3

Solution:
2 2 3 1 2 3 2 1 1 16 +1
0 ( 2 4) 6 = 6
3
|0 = 16 ( 0 ) =
3 3 3

This is equivalent to choice (D).


2 3
Notes: (1) To evaluate ( 2 4) 6 , we can formally make the
substitution = 6 2 3 . It then follows that
= (12 3 2 ) = 3(4 2 ) = 3( 2 4)
Uh oh! There is no factor of 3 inside the integral.

30
1
But constants never pose a problem. We simply multiply by 3 and
3
at the same time. We place the 3 inside the integral where it is needed,
1
and we leave the outside of the integral sign as follows:
3
2 3 1 2 3
( 2 4) 6 = (3)( 2 4) 6
3

1
We have this flexibility to place the 3 and where we like because
3
multiplication is commutative, and constants can be pulled outside of the
integral sign freely.
We now have
2 3 1 2 3
( 2 4) 6 = (3)( 2 4) 6
3

1 1 1 2 3
= = + = 6 +
3 3 3

(2) If we are doing the substitution formally, we can save some time by
changing the limits of integration. We do this as follows:
2 2 3 1 2 2 3
0 ( 2 4) 6 = 0 (3)( 2 4) 6
3

1 16 1 16 1 16 +1
= 0 = |0 = ( 16 0 ) =
3 3 3 3

Notice that the limits 0 and 2 were changed to the limits 0 and 16,
respectively. We made this change using the formula that we chose for
the substitution: = 6 2 3 . When = 0, we have = 0 and when
= 2, we have = 6(2)2 23 = 6 4 8 = 24 8 = 16.
2 1 7
22. ( 2 + 5 + 3 ) =
5

1 5
2 1 7 1
Solution: ( + 5 + 3 ) = (2 2 + 5 2 + 7 3
2 5

3 2

2 1 5 2 7 3 2 10 21
= + ln|| 3 + 2 + = + ln|| 3 3 + .
1
2

3
3 2 2

1
Notes: (1) Recall from problem 11 that [ln ] = . It therefore seems

1
like it should follow that = ln + . But this is not completely

accurate.

31
1 1
Observe that we also have [ln()] = (1) = (the chain rule was

1
used here). So it appears that we also have = ln() + .

How can the same integral lead to two different answers? Well it doesnt.
Note that ln is only defined for > 0, and ln() is only defined for
< 0.
ln if > 0
Furthermore, observe that ln|| = { .
ln() if < 0
It follows that
1
= ln|| + .
2 1 2 2 1
(2) = 2 ( 2 ) = 2 2 . It follows that = 2 2 = + .
2 2 1

2 1 1 2
Also, = 2 1 = 2 ( ) = .
1
3
1 1
2
(3) = 2 . It follows that = 2 = 3 + .
2

3 3
3 3
2 3 2 2 2 2 3
Also, 3 = = =
2 2 = .
2 3 3 3
2

5
7 7 1
(4) 3 = 5 = 7 ( 5 ) = 7 3 . It follows that
5 3 3
2
5
7 3
3 = 7 3 = 7 ( 2 ) + .
5
3

2
2 2
3 2 3 1 3 3 3
Also, 2 = 3 ( ) = 3 ( ) = 2 ( ) = 2 = 3 .

3
3 2
3
2
2 3 2 2

(5) See problem 11 for more information on the laws of exponents used
here.
1
23. =
ln
1
Solution: = ln|ln | + .
ln

32
1
Note: To evaluate , we can formally make the substitution =
ln
1
ln . It then follows that = . So we have

1 1 1 1
ln = ln = = ln|| + = ln|ln | + .
1 1 1 1 1
To get the first equality we simply rewrote as = . This
ln ln ln
way it is easier to see exactly where and are.
1
To get the second equality we simply replaced ln by , and by .

To get the third equality we used the basic integration formula


1
= ln|| + (see problem 22 for details).

To get the last equality we replaced by ln (since we set = ln in the


beginning).
24. 5cot csc 2 =
5cot
Solution: 5cot csc 2 = +
ln 5

Notes: (1) Recall from problem 14 that [5 ] = 5 (ln 5). It follows that

5
5 = ln 5 + .

To verify this, note that


5 1 1
[ + ] = [5 ] + [ ] = 5 (ln 5) + 0 = 5 .
ln 5 ln 5 ln 5


More generally, we have that for any > 0, 1, =
ln

(2) As an alternative way to evaluate 5 , we can rewrite 5 as ln 5


and perform the substitution = ln 5, so that = (ln 5) . So we
have
1 1
5 = ln 5 = ln 5 ln 5 (ln 5) = ln 5
1 1 1 5
= + = ln 5 + = 5 + = + .
ln 5 ln 5 ln 5 ln 5

(3) To evaluate 5cot csc 2 , we can formally make the substitution


= cot . It then follows that = csc 2 . So we have

33
5
5cot csc 2 = 5cot (csc 2 ) = 5 = ln 5 +
5cot
= + .
ln 5

(4) As an alternative, we can combine notes (2) and (3) to evaluate the
integral in a single step by rewriting 5cot csc 2 as (cot )(ln 5) csc 2 ,
and then letting = (cot )(ln 5), so that = ( csc 2 )(ln 5). I
leave the details of this solution to the reader.

LEVEL 1: LIMITS AND CONTINUITY


2 2 137
25. lim =
7 7

(A)
(B) 0
(C) 2
(D) 15
2 2 137 (7)(2+1)
Solution 1: lim = lim = lim (2 + 1)
7 7 7 7 7

= 2(7) + 1 = 15.
This is choice (D).
Notes: (1) When we try to substitute 7 in for we get the indeterminate
0
form . Here is the computation:
0

2(7)2 13(7)7 98917 0


= = .
77 77 0

This means that we cannot use the method of plugging in the number
to get the answer. So we have to use some other method.
(2) One algebraic trick that works in this case is to factor the numerator
as 2 2 13 7 = ( 7)(2 + 1). Note that one of the factors is
( 7) which is identical to the factor in the denominator. This will
always happen when using this trick. This makes factoring pretty easy
in these problems.

34
(3) Most important limit theorem: If () = () for all in some
interval containing = except possibly at itself, then we have
lim () = lim ().

In this problem, our two functions are


2 2 137
() = and () = 2 + 1.
7

Note that and agree everywhere except at = 7. Also note that (7)
is undefined, whereas (7) = 15.
(4) To compute a limit, first try to simply plug in the number. This will only
fail when the result is an indeterminate form. The two basic
0
indeterminate forms are and (the more advanced ones are 0 ,
0
, 00 , 1 , and 0 , but these can always be manipulated into one of the
two basic forms).
If an indeterminate form results from plugging in the number, then there
are two possible options:
Option 1: Use some algebraic manipulations to create a new function that
agrees with the original except at the value that is being approached, and
then use the limit theorem mentioned in note (3).
This is how we solved the problem above.
Option 2: Try L'Hpital's rule (see solution 2 below).
Solution 2: We use L'Hpital's rule to get
2 2 137 413
lim = lim = 4(7) 13 = 15, choice (D).
7 7 7 1

Notes: (1) L'Hpital's rule says the following: Suppose that


(i) and are differentiable on some interval containing (except
possibly at itself).
(ii) lim () = lim () = 0 or lim () = lim () =

()
(iii) lim exists, and
()

(iv) () 0 for all in the interval (except possibly at itself).

35
() ()
Then lim = lim .
() ()

In this problem () = 2 2 13 7 and () = 7.


(2) It is very important that we first check that the expression has the
correct form before applying L'Hpital's rule.
In this problem, note that when we substitute 7 in for in the given
0
expression we get (see note 1 above). So in this case L'Hpital's rule can
0
be applied.
5 2 3+2
26. If () = , then lim () =
3 2 2 1

(A) 4
7
(B)
4
5
(C)
3
(D) 0
5 2 3+2 5(1)2 3(1)+2 53+2 4
Solution: lim () = lim = = = = 4.
1 1 3 2 2 3(1)2 2(1) 32 1

This is choice (A).


Notes: (1) We simply substituted 1 in for . Since we did not get an
indeterminate form, we see that the answer is 4.
(2) It would be incorrect to try to apply L'Hpital's rule here! Lets see
what happens if we try:
5 2 3+2 103 10(1)3 7
lim = lim = = .
1 3 2 2 1 62 6(1)2 4

So we get choice (D) which is wrong!


If we tried to apply L'Hpital's rule twice we would get
5 2 3+2 103 10 5
lim = lim = lim = .
1 3 2 2 1 62 1 6 3

So we get choice (C), also wrong!

36
3 2 2+1
27. lim
7 2 +53
1
(A)
3
3
(B)
7
7
(C)
3
(D)
3 2 2+1 3 2 3
Solution: lim = lim = , choice (B).
7 2 +53 7 2 7

()
Notes: (1) If and are polynomials, then lim = lim where
()
1
() = + 1 + + 1 + 0 and
() = + 1 1 + + 1 + 0 .

(2) If = , then lim = .

(3) Combining notes (1) and (2), we could have gotten the answer to this
problem immediately by simply taking the coefficients of 2 in the
numerator and denominator and dividing.
The coefficient of 2 in the numerator is 3, and the coefficient of 2 in the
3
denominator is 7. So the final answer is .
7
1
(4) If > 0, then lim = 0.

(5) For a more rigorous solution, we can multiply both the numerator and
1
denominator of the fraction by 2 to get

1 2 1
3 2 2+1 ( 2) (3 2 2+1) 3 + 2

= 1 = 5 3 .
7 2 +53 ( 2) (7 2 +53) 7+ 2

1 1
3 2 2+1 lim 32 lim ( )+ lim 2 32(0)+(0) 3

It follows that lim = 1 1 = = .
7 2 +53 lim 7+5 lim ( )3 lim ( 2 ) 7+5(0)3(0) 7

(6) L'Hpital's rule can also be used to solve this problem since the limit

has the form :

3 2 2+1 62 6 3
lim = lim = lim = .
7 2 +53 14+5 14 7

37
Observe that we applied L'Hpital's rule twice. The first time we
differentiated the numerator and denominator with respect to to get

another expression of the form .

28. If the function is continuous for all real numbers and if


2 6
() = for all 3, then (3) =
3

(A) 0
(B) 1
(C) 2
(D) 5
2 6 (3)(+2)
Solution 1: (3) = lim = lim = lim ( + 2)
3 3 3 3 3

= 3 + 2 = 5.
This is choice (D).
Notes: (1) A function is continuous at = if lim () = ().

In this problem we want the function to be continuous for all real


numbers. In particular, we need to be continuous at = 3. So we must
2 6
have (3) = lim () = lim .
3 3 3
0
(2) When we try to substitute 3 in for we get the indeterminate form .
0
Here is the computation:
32 36 99 0
= = .
33 33 0

This means that we cannot use the method of plugging in the number
to compute the limit. We used the same method from the first solution in
problem 25.
(3) As an alternative we could have used L'Hpital's rule (just like we did
for the second solution in problem 25). Here are the details:
2 6 21
(3) = lim = lim = 2(3) 1 = 5, choice (D).
3 3 3 1

See problem 25 for more information about LHpitals rule.

38
sin3 5
29. lim
0 3

(A) 125
(B) 5
(C) 125
(D) The limit does not exist
sin3 5 sin3 5 (sin 5)3
Solution: lim = lim 53 = 125 lim
0 3 50 53 3 50 (5)3

sin 3 sin 3
= 125 lim ( ) = 125(lim ) = 125(1)3 = 125.
0 0

This is choice (C).


Notes: (1) A basic limit worth memorizing is
sin
lim = 1.
0

(2) The limit in note (1) is actually very easy to compute using L'Hpital's
rule:
sin cos
lim = lim = cos 0 = 1
0 0 1

(3) It is not hard to see that 0 if and only if 5 0. This is why we


can replace by 5 in the subscript of the limit above.
(4) sin is an abbreviation for (sin ) .
In particular, sin3 = (sin )3 , and so sin3 5 = (sin 5)3 .
sin 5 sin 5
(5) can be rewritten as 5 .
5

sin3 5 sin3 5
It follows that we can rewrite as 53 .
3 53 3

(sin 5)3 sin 5 3


(6) 53 3 = (5)3 by a basic law of exponents, and (5)3
=( )
5
by another basic law of exponents.
(7) Using the substitution = 5, we have
sin 5 3 sin 3
lim ( ) = lim ( ) .
50 5 0

39
(8) If lim () and lim () both exist, then

lim [() ()] = lim () lim ()


In particular,
lim [()]2 = lim [() ()] = lim () lim () = [lim ()] 2


More generally, lim [()] = [lim ()]

sin 3 sin 3
This is why we have lim ( ) = (lim ) .
0 0

3 tan 3 cos2 tan


30. lim
0 3

(A) 0
1
(B)
3
(C) 3
(D)
3 tan 3 cos2 tan 3 tan (1cos2 )
Solution: lim = lim
0 3 0 3

3 tan sin2 tan sin 2


= lim = 3 (lim ) (lim ) = 3(1)(1) = 3.
0 3 0 0

This is choice (C).


Notes: (1) We get the first equality by factoring 3 tan .
(2) To get the second equality we used the most basic Pythagorean
identity
+ =
Subtracting cos 2 from each side of this equation gives us
sin2 = 1 cos 2
(3) For the third equality we use a basic limit rule concerning products.
See problem 29 for details.
sin
(4) As noted in problem 29, lim = 1.
0

40
tan
We also have lim = 1.
0

Both of these results can easily be seen by using L'Hpital's rule.


For the second one, we have
tan sec2 1 2
lim = lim = sec 2 0 = (sec 0)2 = ( ) = 1.
0 0 1 cos 0

31. If the function is continuous for all in the interval [, ], then


at any point in the interval (, ), which of the following must
be true?
(A) lim () = ()

(B) () exists
(C) () = 0
(D) () = () ()
Solution: is continuous at = if and only if lim () = (), choice

(A).
Notes: (1) This question is simply asking us to pick out the definition of
continuity at a value = .
(2) The function () = || on the interval [1,1] is a counterexample to
choices (B), (C), and (D).
Note that () = || is continuous for all . In particular, is continuous
for all in [1,1].
(0) does not exist, since the graph of has a sharp edge there. This
allows us to eliminate choice (B).
1 1
( ) = 0 and (1) (1) = 1 1 = 0.
2 2

This allows us to eliminate choices (C) and (D).

41
32. The graph of the function is shown in the figure above. Which
of the following statements about is true?
(A) lim () =

(B) lim () =

(C) lim () =

(D) lim () = ()

Solution: From the graph we see that lim () = , choice (C).


Notes: (1) The open circles on the graph at and indicate that there is
no point at that location. The darkened circle at indicates () = .
(2) lim () = and lim+ () = . Therefore lim () does not exist.

(3) is not defined at = , ie. () does not exist. In particular, is not


continuous at . So lim () ().

LEVEL 2: PRECALCULUS
5 ln(2 +1)2 52
33. If (()) = and () = , then () =
ln(2 +1)+3 +3

(A) ln
(B) ln 2
(C) ln(2 + 1)
(D) ln 2

42
Quick solution by observation: What do we replace by in () to get
(())? By observation the answer is ln(2 + 1), choice (C).
Notes: (1) (()) is the composition of the function with the function
().
(2) We compute (()) by replacing in the function () with ().
Since we do not know what () is in this problem, we must answer the
question what should I replace by in the function ()?
(3) If you do not immediately see what to replace with in the function
(), use the answer choices as a guide. If you have no idea which one
will work, then, as usual, you should begin by checking choice (C) first.
(4) If we are guessing that choice (C) is the answer, then we have that
5 ln(2 +1)2
(()) = (ln(2 + 1)) = . So choice (C) is correct.
ln(2 +1)+3

34. Suppose that is a function that is defined on (, ). Which


of the following conditions guarantees that 1 exists?
(A) is symmetric with respect to the origin.
(B) is continuous at all real numbers.
(C) has no points of inflection.
(D) is a strictly decreasing function.
Solution: A strictly decreasing function is always one-to-one, and
therefore invertible. So the answer is choice (D).
Notes: (1) A function is one-to-one if different -values always lead to
different -values.
Symbolically this can be written as () ().
An equivalent way to write this is by using the contrapositive:
() = () = .
Below is a picture showing the graph of a function which is one-to-one.
Notice how () and () are different.

43
() ().
(2) A one-to-one function has a graph that passes the horizontal line test.
This means that every horizontal line hits the graph of the function at
most once.
(3) Using the horizontal line test, it is easy to see that every strictly
increasing or strictly decreasing function is one-to-one (see the figure
above for an example of a strictly increasing function).
It is worth committing to memory the fact that every strictly increasing or
decreasing function is one-to-one.
(4) There are examples of functions that are one-to-one whose graphs are
neither strictly increasing nor strictly decreasing. Such a function could
not be continuous however.
Below is a picture showing the graph of a one-to-one function that is
neither strictly increasing nor strictly decreasing.

(5) If is one-to-one, then is invertible. The inverse of is written 1 ,


and is defined by 1 () = if and only if () = .
Graphically, the function 1 is the reflection of the function in the line
= .

44
(6) Below is the graph of a noninvertible function that is symmetric with
respect to the origin and continuous at all real numbers. This eliminates
choices (A) and (B).

(7) The function () = 2 is an example of a noninvertible function with


no points of inflection. This eliminates choice (C).
35. Let () = sin(arctan ). The range of is
(A) {| 1 1}
(B) {| 1 < < 1}
(C) {|0 1}
(D) {|0 < 1}

Solution: For all , < arctan < . It follows that
2 2

1 < sin(arctan ) < 1.


So the answer is choice (B).

45
Notes: (1) Lets take a look at the graph of the function () = tan .

Note that the graph of tan shown above has consecutive vertical

asymptotes of = and . If we restrict the domain of tan to
2 2

< < , we have a one-to-one function.
2 2

So although () = tan is not invertible (it fails the horizontal line test),

the restricted function () = tan , < < is invertible, and we
2 2
call its inverse 1 () = arctan . Note that 1 () = arctan has

domain all real numbers, and range < < as we can see in the
2 2
following figure.

46
(2) Lets take a look at the graph of () = sin .


Note that for < < , 1 < sin < 1 (in particular 1 and 1 do not
2 2
get hit by sin for any of these -values).
(3) Putting notes (1) and (2) together, we have 1 < sin(arctan ) < 1.
(4) See problem 34 for more general information on one-to-one functions
and inverses.

36. If log (5 ) = , then =
3

(A) 125
(B) 25
(C) 5
(D) 3

Solution: Changing to exponential form, we have 5 = 3 . We raise each
3
side of this equation to the power of to get

3
3
(5 ) = ( 3 )
3
5 =
125 =
So the answer is = 125, choice (A).
Notes: (1) See the notes at the end of problem 3 for more information on
logarithms.


(2) ( ) = = 1 = .

47
(3) It follows from note (2) that we can eliminate an exponent on one side
of an equation by raising each side of the equation to the reciprocal of
that exponent.

(4) In note (2) we used the law of exponents ( ) = .


(5) See problem 11 for more information on the laws of exponents used
here.
37. What is the range of the following function?
() = 2 cos(3 2) 5
(A) 3 7
(B) 3 7
(C) 3 2
(D) 7 3
Solution: The amplitude of () is 2 and there is a vertical shift of 5. So
the minimum and maximum values of () are 2 5 = 7 and
2 5 = 3. So the range is 7 3, choice (D).
Notes: (1) The amplitude of a function of the form cos( + ) + is
||.
(2) The vertical shift of such a function is .
(3) The range of such a function is || + || + .
(4) In this problem we do not need to worry about the phase shift + .
(5) This problem can also be solved using the graphing functions on your
TI-84 calculator (if a calculator is allowed for the problem). Make sure the
calculator is in radian mode (Press MODE and select Radian), enter the
function under Y=, find a viewing window that captures the lowest and
highest points of the function (you can start by pressing ZOOM 7 to get a
standard trig window, and then change Ymin and Ymax under WINDOW
to 10 and 0, respectively, for example), then use the minimum and
maximum features under CALC to find the smallest and largest values in
the range.

48
5
38. * If () = 3 2, what is 1 (2.2) ?
(A) 3.77
(B) 4.23
(C) 4.87
(D) 5.01
Solution by starting with choice (C): We are looking for a value of so
that () = 2.2. Lets start with choice (C) and guess that = 4.87. We
5
have (4.87) = 4.873 2 2.6. This is too big so we can eliminate
choices (C) and (D).
Lets try choice (A) next and guess that = 3.77. We have that
5
(3.77) = 3.773 2 2.2. So the answer is choice (A).
Solution by finding the inverse function: We interchange the roles of
and and solve for .
5
() = 3 2
5
= 3 2
5
= 3 2
5 = 3 2
5 + 2 = 3
3
5 + 2 =
3
1 () = 5 + 2
3
So 1 (2.2) = 2.25 + 2 3.77, choice (A).
2 5
39. What is the period of the graph of y = tan( 2) ?
3 2
4
(A)
15
2
(B)
5
2
(C)
3
4
(D)
15

Solution: The period of the graph of = tan( ) is . So the period

5 2 2
of the graph of the given function is 5 = = = , choice (B).
2 5 5
2

49

40. If arcsin(sin ) = and 0 2, then could equal
4

(A) 0

(B)
6

(C)
3
3
(D)
4

Solution: could certainly equal , but is not an answer choice. So we
4 4

need to find so that sin = sin .
4 4

Since sin > 0 in quadrant II, we find the corresponding second


4 3
quadrant angle. This is = = , choice (D).
4 4 4

LEVEL 2: DIFFERENTIATION

41. If = sin2 5, then =

(A) 2 sin 5
(B) 10 sin 5
(C) 2 sin 5 cos 5
(D) 10 sin 5 cos 5

Solution: = 2 (sin 5 )(cos 5)(5) = 10 sin 5 cos 5, choice (D).

Notes: (1) sin2 is an abbreviation for (sin )2 .


Therefore sin2 5 is really (sin 5)2 .
(2) The derivative of () = sin is () = cos .
(3) We used two applications of the chain rule here. Letting = (), we
have
() = ( )() = ((())) ,
where () = 5, () = sin , and () = 2 . So we have that
() = 5, () = cos , and () = 2

() = ((())) (()) () = 2 (sin 5 )(cos 5)(5)

50
42. If () = tan2 (5 ), then (0) is equal to which of the
following?
(A) 2 tan 5 sec 5
(B) 2 tan 5 sec 2 5
(C) 2 tan2 5 sec 5
(D) 2 tan2 5 sec 2 5
Solution:
() = 2 tan(5 ) sec 2 (5 )(1) = 2 tan(5 ) sec 2 (5 ).
So (0) = 2 tan 5 sec 2 5, choice (B).
Notes: (1) tan2 is an abbreviation for (tan )2 .
Therefore tan2 (5 ) is really (tan(5 ))2 .
(2) The derivative of () = tan is () = sec 2 .
(3) We used two applications of the chain rule here.
We have T () = ( )() = ((())) , where
() = 5 , () = tan , and () = 2 . So we have that
() = 1, () = sec 2 , and () = 2

((())) (()) () = 2 tan(5 ) sec 2 (5 )(1)

51
43. The function , whose graph consists of three line segments, is
shown above. Which of the following are true for on the open
interval (, ) ?
I. lim () exists.

II. The domain of the derivative of is the open interval (, ).
III. The derivative of is negative on the interval (, ).
(A) I only
(B) II only
(C) III only
(D) I and III only
Solution: Since lim () = lim+ (), lim () exists. So I is true.

On (, ), the function fails to be differentiable at = and = only.


So the domain of the derivative of contains points that are not in (, ).
So II is false.
The function is decreasing on (, ). Therefore the derivative of is
negative on (, ). So III is true.
It follows that the answer is choice (D).
Notes: (1) lim () exists because as we approach from both the left

and the right, we are heading toward the same point.
In this case it is actually true that the function is continuous at =
because the value that we are approaching is actually attained.

52
Note however that is not differentiable at = because there is a
sharp edge there. More specifically, the slope of the tangent line from
the left is negative because the function is decreasing to the left of , and
the slope of the tangent line from the right is positive because the
function is increasing to the right of .
Lets compare this to the situation at = . lim () exists because as

we approach from both the left and the right, we are heading toward
the same point.
In this case however the function is not continuous at = because
the value that we are approaching is not the same as () which is a larger
number (because it is higher on the graph).
Of course is not differentiable at = since continuity is required for
differentiability.
(2) The domain of the derivative of is (, ) (, ) (, ). In other
words, the derivative of exists at all values between and except
for = and = .
The derivative of does not exist at = because there is a sharp edge
there, and the derivative of does not exist at = because is
discontinuous at = .
(3) If is differentiable, then the derivative of is positive if and only if
is increasing, and the derivative of is negative if and only if is
decreasing.
In this problem, the function is differentiable and increasing on (, ).
Therefore () > 0 for all in (, ).
The function is decreasing on the intervals (, ) and (, ). Therefore
() < 0 for all in (, ) and for all in (, ).

44. If () = 3 + 5, then () =
9+10
(A)
23+5
9
(B)
23+5
9
(C)
3+5
9
(D)
23+5

53
Solution:
1
1
() = (3 + 5)2 (3) + (3 + 5)(1) =
2

3 (3+5) (23+5) 3 2(3+5) 3+6+10 9+10


+ = + = = .
23+5 1 (23+5) 23+5 23+5 23+5 23+5

This is choice (A).


Notes: (1) We begin with the product rule.

f () = ()() , where () = , and () = 3 + 5.


() = () () + () ()

(2) () = [ ] = 1

1
(3) () = 3 + 5 can be written as () = (3 + 5)2 . To take the
derivative of requires the chain rule. We have
1 1
() = (3 + 5)2 (3)
2
Note that v () = (()) , where () = and () = 3 + 5.

By the chain rule, () = (()) ().


1
1 1
(4) (3 + 5)2 = 1 = .
(3+5)2 3+5

(5) Combining (3) and (4) we have


1 1
1 3
[3 + 5] = [(3 + 5)2 ] = (3 + 5)2 (3) = .
2 23+5

(6) See problem 11 for a review of all of the laws of exponents you should
know.
3 (3+5)
(7) We would like to express + as a single fraction. To do
23+5 1
this we multiply the numerator and denominator of the second term by
23 + 5.

(8) By the definition of the positive square root, = . So we have


3 + 5 3 + 5 = 3 + 5.

54
45. The slope of the tangent line to the curve 5 3 4 = 10 at
( 2, 1) is
(A) 0
2
(B)
7
13
(C)
42
1
(D)
3

Solution: We have

(5 4 ) + 5 (1) 3 (4 3 ) + 4 (3 2 ) = 0.

We now substitute 2 for and 1 for to get



(2) (5(1)4 ) + (1)5 (1) (2)3 (4(1)3 ) + (1)4 (3(2)2 ) = 0


(2) (5 1 ) + (1)(1) (8) (4(1) ) + (1)(3 4) = 0


10 1 32 12 = 0


42 13 = 0

13
So we have 42 = 13, and therefore = , choice (C).
42

Notes: (1) The given equation defines the dependent variable implicitly
as a function of .
This just means that the variable is not by itself on one side of the
equation.
An example of an explicitly defined function is = 2.
This same function can be defined implicitly as 2 = 0.
(2) When the dependent variable is defined implicitly as a function of ,

we can use implicit differentiation to find the derivative .

This is just an application of the chain rule. Since is a function of , we


can write = () for some function . So if we want to differentiate
(), simply note that this is (()), and so the derivative is

(()) () = () .

55

For example, the derivative of 5 is 5 4 , and the derivative of 4 is


4 3 .

(3) To get the derivative in this problem we needed to use the product
rule twice: once for the product 5 , and once for the product 3 4 .
(4) Remember that the derivative of a constant is 0. So once we
differentiate, we get 0 on the right hand side of the equation. A common
mistake is to leave this as 10.
(5) In the solution above after differentiating we plugged in 2 for and

1 for first, and then solved for . This is generally much faster than


solving for first and then plugging in the point.

(6) If we were not given a point in the question, and were simply asked to

find , then we would do the following:


(5 4 ) + 5 (1) 3 (4 3 ) + 4 (3 2 ) = 0


5 4 + 5 4 3 3 3 2 4 = 0

4
5 4 3 3 = 3 2 4 5

3 3
(5 4 4 ) = 3 2 4 5

3 2 4 5 4 (3 2 ) (3 2 )
= = =
5 4 4 3 3 3 (54 2 ) (54 2 )

(7) As an alternative solution to this problem (although not



recommended), after differentiating, we can first solve for as in note

(6), and then plug in 2 for and 1 for as follows:
(3 2 ) (1)(3(2)2 (1)) (1)(3(4)+1) (1)(12+1) 13
= = = = =
(54 2 ) (2)(5(1)4(2)2 ) (2)(54(4)) (2)(516) 42
2
46. Let () = 3 + 5. A value of that satisfies the
conclusion of the Mean Value Theorem for on the interval
[2,2] is
(A) 2
1
(B)
2
1
(C)
6
(D) 0

56
Solution: () = 6 + 1, so that () = 6 + 1.
(2) = 3(2)2 + 2 5 = 3(4) 3 = 12 3 = 15.
(2) = 3(2)2 2 5 = 3(4) 7 = 12 7 = 19.
(2)(2)
We now solve the equation () = to get
2(2)

15(19) 15+19 4
6 + 1 = = = =1
2+2 4 4

So 6 = 1 1 = 0, and = 0, choice (D).


Notes: (1) The Mean Value Theorem says that if is a function that is
continuous on the closed interval [, ] and differentiable on the open
interval (, ), then there is a real number with < < such that
()()
() = .

In this problem = 2, = 2, and () = 3 2 + 5.


(2) Note that the function () = 3 2 + 5 is a polynomial. It is
therefore continuous and differentiable everywhere. In particular, it is
continuous on [2,2] and differentiable on (2,2). Therefore by the
Mean Value Theorem, there is a between 2 and 2 such that
(2)(2)
() = .
2(2)

(3) Geometrically, () is the slope of the tangent line to the graph of the
()()
function at , and is the slope of the secant line passing

through the points (, ()), and (, ()).
The Mean Value Theorem says that the slopes of these two lines are the
same. In other words, they are parallel.

57
1 3 1
47. If () = 5 + + 5, then (1) =

53
(A)
6
58
(B)
15
58
(C)
15
53
(D)
6
2 7
1 5
Solution: () = 5 4 2 3 2 .
3 2
2 7
1 5 1 5 53
(1) = 5(1)4 12 (1)3 (1)2 = 5 1 = .
3 2 3 2 6

So the answer is choice (A).


1 1
Notes: (1) can be written as 1 . So [ ] = 1 2 = 2 .

1 2
1
(2) can be written as 3 . So [ ] = 3 .
3 3
3
5 7
1 1 1 1 5
(3) = 1 = 5 = 2 . So [ ] = 2 .
5 5 2
( 5 )2 2

(4) 1 = 1 for all real numbers .


2 7
In particular, 14 = 1, 12 = 1, 13 = 1, and 12 = 1

58
(5) Although it is not needed in this problem, it is useful to be able to
rewrite the following:
1 1
2 = . So 2 = .
2 2
2 2
1 1 1 1
3 = 2 = 3 2
. So 3 = 3 .
3 3 3 2

7 7
1 1 5 5
2 = 7 = 7
. So 2 = .
2
2 2 7

(6) See problem 11 for a review of all of the laws of exponents you should
know.
7 6
48. The derivative of () = attains its minimum value at
7 5
=
7
(A)
5
6
(B)
5
(C) 1
(D) 0
6
Solution: () = 6 5 .
5

To minimize () we first take its derivative


() = 6 5 6 4 = 6 4 ( 1).
We then check when () is equal to 0 to find the critical numbers of
. The critical numbers of are = 0 and = 1.
Now note that for < 1, () < 0, and for > 1, () > 0. So is
decreasing for < 1 and increasing for > 1. So attains its minimum
value at = 1, choice (C).
Notes: (1) A critical number of a function is a real number in the
domain of such that () = 0 or () is undefined.
(2) A function attains a relative minimum (or local minimum) at a real
number = if there is an interval (, ) containing such that
() < () for all in the interval.

59
If a function is decreasing to the left of , and increasing to the right of ,
then attains a relative minimum at .
In terms of derivatives, if () < 0 for < , and () > 0 for > ,
then attains a relative minimum at .
This method of finding the relative extrema of a function is called the first
derivative test.
(3) Take careful note that in this problem we are trying to minimize the
derivative of . So we need to be checking () near = .
(4) A nice visual way to apply the first derivative test is by creating a sign
chart. We split up the real line into intervals using the critical numbers of
the function (and we also include any points of discontinuity if they exist),
and then check the sign of the derivative in each subinterval formed.

In this case we split up the real line into three pieces. Notice that the
cutoff points are 0 and 1, the critical numbers of . We then plug a real
number from each of these three intervals into to see if the answer is
positive or negative. For example, (2) = 6(2)4 (2 1) > 0. Note that
we do not need to finish the computation. We only need to know if the
answer is positive or negative. Finally, we make a quick sketch that
decreases across the intervals where there is a minus sign, and increases
across the intervals where there is a plus sign. We can then clearly see
that there is a minimum at = 1.
(5) The sketch of the graph in the sign chart is not meant to be completely
accurate (and it is not). It is only being used to demonstrate where the
function is increasing and decreasing, so we can determine if there is a
minimum or maximum (or neither) at each critical number.
(6) Lets take a look at the expression () = 6 4 ( 1).

60
We see that () = 0 when = 0 and = 1. Note that = 0 is a zero
of the equation of multiplicity 4. In particular, it has even multiplicity. It
follows that the graph of does not pass through the -axis at = 0. In
other words, does not change sign there, and so a maximum or
minimum cannot occur at = 0.
On the other hand, = 1 is a zero of multiplicity 1. In particular, it has
odd multiplicity. It follows that the graph of does pass through the
-axis at = 1. In other words, does change sign there, and so a
maximum or minimum does occur at = 1.
(7) Based on the reasoning of note (6) we can get the answer to this
question just by looking at the factored form of . Since = 1 is the only
zero with odd multiplicity, it is the only possible place where a minimum
can occur. So the answer must be choice (C).
In other words, in this problem we do not actually need to perform the
first derivative test (and in particular, we do not need to draw a sign
chart). But its not a bad idea to do so just to check your work.

LEVEL 2: INTEGRATION
2 3 2
49. 0 =
8 3

(A) 42
(B) 1 42
(C) 42 1
(D) 42
Solution:
1 1 1 1
2 3 2 2 (3 2 ) 0 0
0 = 0 = 8 2 = 22 |8 = 2 (02 82 ) =
8 3 8 3
28 = 2 22 = 42.
This is choice (D).
Notes: (1) We made the substitution = 8 3 , so that = 3 2 .
(2) Since the minus sign is missing (or equivalently, a factor of 1), we
multiply by 1 inside and outside the integral. So we have

61
2 3 2 2 (3 2 )
0 = 0 .
8 3 8 3

When we replace 8 3 by and 3 2 by we get


1
2 (3 2 ) 0 0
0 = 8 = 8 2 .
8 3

Notice two things:


First, we changed the limits of integration. We get the new limits by
noticing that when = 0, we have = 8 03 = 8, and when = 2, we
have = 8 23 = 8 8 = 0.
1
1 1
Second, = 1 = 2 .
2
1
+1
(3) To integrate 2 we use the power rule = + . In this case
+1
1
1 1 1 1
2 1
we get 2 = 1 + = 2 + = 2 2 + = 22 + .
2
2

1
(4) 82 = 8 = 4 2 = 4 2 = 22.
(5) We can also evaluate the integral without making a formal substitution
as follows:
1 1 1
2 3 2 2 (3 2 ) 2
0 = 0 = 2(8 3 )2 |0 = 2 (02 82 ) = 42.
8 3 8 3

50. 012 tan2 3 =

(A) 4
4
1
(B) 3
4
4
(C)
12
123
(D)
4


1 12
2 (sec 2
Solution: 0 tan 3 = 0
12 12 3 1) = ( tan 3 ) |
3 0
1 1 4
= ( tan ) (0 0) = (1) = , choice (C).
3 4 12 3 12 12

62
Notes: (1) cos 2 + sin2 = 1 is the most important Pythagorean
identity. Make sure you have this one memorized. You may also want to
memorize the other two Pythagorean identities:
1 + tan2 = sec 2 and cot 2 + 1 = csc 2
The second Pythagorean identity can be derived from the first by dividing
each side of the first equation by cos 2 as follows.
cos2 +sin2 1
=
cos2 cos2
cos2 sin2 1
+ =
cos2 cos2 cos2
2 2
1 + tan = sec
Similarly, the third Pythagorean identity is derived from the first by
dividing each side of the first equation by sin2 .
(2) In this problem we solve the second Pythagorean identity for tan2 to
get tan2 = sec 2 1. We then replace by 3 to get
tan2 3 = sec 2 3 1.

(3) Since [tan ] = sec 2 , it follows that sec 2 = tan + .

(4) To evaluate sec 2 3 , we can formally make the substitution


= 3. We then get = 3. It follows that
1 1 1 1
sec 2 3 = 3 sec 2 3 3 = 3 sec 2 = 3 tan + = 3 tan 3 + .

3 2 +1
51. 1 =
2

(A) 3
4
(B) ln 3
3
(C) ln 3
4
(D) ln 3 +
3

3 2 +1 3 2 1 3 1
Solution: 1 = 1 ( + ) = 1 (1 + 2 )
2 2 2 2
3 1 4
= ( + ln|| + 1 ) |1 = (3 + ln 3 + ) (1 + 0 + 1) = ln 3 + .
3 3

This is choice (D).

63
2
52. The solution to the differential equation = , where y(3) = 0,
4
is
5 5
(A) = 3 45
3
5 5
(B) = 3 9
3
5 5
(C) = 3 45
3
5 5 5
(D) = 3 45
3

Solution: We separate variables to get 4 = 2 . We then integrate


5 3
both sides of this equation to get = + .
5 3

05 33
Now we substitute in = 3 and = 0 to get = + . So = 9, and
5 3
5 3
we have = 9. We multiply each side of this equation by 5 to get
5 3
5
5 = 45, and then we take the fifth root of each side of this last
3
3
5 5
equation to get = 3 45, choice (A).
3

Notes: (1) A separable differential equation (or separable DE) has the

form = ()(). We can solve this type of differential equation by

1
rewriting it in the form = (). This is called separating
()
1
variables. We then integrate each side to get = () + .
()

1
In this problem we have () = 2 and g () = .
4

(2) The condition (3) = 0 is called an initial condition. We can use the
initial condition to find the constant .
In this case we substitute in = 3 and = 0, and then solve for .
(3) It is usually more efficient to find the constant before solving for the
dependent variable .
(4) Note that when we substitute 3 in for into the answer choices, only
choices (A) and (D) yield = 0. We can use this reasoning to eliminate
choices (B) and (C).

64
(ln )2 2
53. Each of the following is an antiderivative of EXCEPT

(ln )3
(A) ln 2
3
(B) ln ln 2
(ln )3
(C) 2 ln||
3
(ln )3
(D) 1 ln 2 +
3

(ln )2 2 (ln )3 (ln )3


Solution: = 2 ln|| + = ln 2 +
3 3

All answer choices have one of the two forms above except choice (B).
(ln )2 2 (ln )2 2
Notes: (1) = . So we integrate each of the two terms

separately.
(ln )2
(2) To evaluate , we can formally make the substitution

1
= ln . It then follows that = . So we have

(ln )2 1 3 (ln )3
= (ln )2 = 2 = + = + .
3 3

(3) With a little practice, we can evaluate an integral like this very quickly
1
with the following reasoning: The derivative of ln is . So to integrate

(ln )2 2
we simply pretend we are integrating but as we do it we leave

the ln where it is. This is essentially what was done in the above
solution.
1
Note that the goes away because it is the derivative of ln x. We need

it there for everything to work.
1
(4) You should know = ln|| + (See problem 22 for details.)

2 1
It follows that = 2 = 2 ln|| + .

(5) Putting notes (1) through (4) together gives us


(ln )2 2 (ln )2 2 (ln )3
= = 2 ln|| + .
3

(6) ln = ln . So we have 2 ln|| = ln||2 = ln 2

65
(7) ||2 = 2 because 2 is always nonnegative.
(8) We can also solve this problem by differentiating the answer choices.
For example, looking at choice (A), we have
(ln )3 3 (ln )2 1 1 (ln )2 2 (ln )2 2
[ ln 2 ] = 2 = = .
3 3 2

This shows that we can eliminate choice (A), and a moments thought
allows us to eliminate choice (D) as well.
Lets look at choice (C) next. We have
(ln )3 3 (ln )2 1 1 (ln )2 2 (ln )2 2
[ 2 ln||] = 2 = = .
3 3

So we can eliminate choice (C) and the answer must be choice (B).
1
(Note that [ln||] = .)

For completeness, lets differentiate the expression in choice (B).


1 1 1 2 1
[ln ln 2 ] = 2 = = .
2
1 (ln )2 2
Since , the answer is definitely choice (B).

8
54. =
1+ 2

(A) 8 ln(1 + 2 ) +
8
(B) 8 +

1
(C) 8 tan +
16
(D) (1+ 2 )2
8 1
Solution: = 8 = 8 tan1 + , choice (C).
1+ 2 1+ 2

Notes: (1) You should know the following basic inverse trig integrals:

= +


+ = +

= +
||

66
(2) If you already know the derivatives corresponding to each of the
integrals given in note (1), then there is no need to memorize anything
new. See problem 75 for the six inverse trig derivatives.
55. The area of the region bounded by the lines = 1, = 4, and
= 0 and the curve = 3 is
1
(A) 3 ( 9 1)
3
(B) 3 ( 9 1)
(C) 12 1
(D) 3 3 ( 9 1)
4 1 4 1 1 1
Solution: 1 3 = 3 |1 = 12 3 = 3 ( 9 1).
3 3 3 3

This is choice (A).


Notes: (1) To compute the area under the graph of a function that lies
entirely above the -axis (the line = 0) from = to = , we simply
integrate the function from to .
In this problem, the function is = 3 , = 1, and = 4.
Note that > 0 for all . So 3 > 0 for all . It follows that the graph
of = 3 lies entirely above the -axis.
(2) Although it is not needed in this problem, here is a sketch of the area
we are being asked to find.

(3) To evaluate 3 , we can formally make the substitution = 3.


It then follows that = 3.

67
1
We place the 3 next to where it is needed, and we leave the outside
3
of the integral sign as follows:
1
3 = 3 3 3

We now have
1 1 1 1
3 = 3 3 3 = 3 = 3 + = 3 3 + .

We get the second equality by replacing 3 by , and 3 by .


We get the third equality by the basic integration formula

= + .
And we get the rightmost equality by replacing with 3.
(4) With a little practice, we can evaluate an integral like this very quickly
with the following reasoning: The derivative of 3 is 3. So we artificially
1
insert a factor of 3 next to , and outside the integral sign. Now to
3
integrate 3 3 we simply pretend we are integrating but as we do it
we leave the 3 where it is. This is essentially what was done in the above
solution.
Note that the 3 goes away because it is the derivative of 3. We need it
to be there for everything to work.
(5) If we are doing the substitution formally, we can save some time by
changing the limits of integration. We do this as follows:
4 1 4 1 12 1 12 1 1
1 3 = 3 1 3 3 = 3 3 = 3 |3 = 3 12 3 3 .

Notice that the limits 1 and 4 were changed to the limits 3 and 12. We
made this change using the formula that we chose for the substitution:
= 3. When = 1, we have = 3(1) = 3. And when = 4, we have
= 3(4) = 12.
Note that this method has the advantage that we do not have to change
back to a function of at the end.

68
5
56. A population of protozoa is growing at a rate of 400 2 protozoa
per second. At = 0 seconds, the number of protozoa present was
160. Find the number present after 2 seconds.
(A) 320 5
(B) 160 5
(C) 160 2
(D) 80 2
Solution: If we let be the number of protozoa present at time , then
5

we are given that = 400 2 . We integrate to get

5 5 5
2
= 400 2 = 400 ( ) 2 + = 160 2 + .
5

We now use the initial condition (0) = 160 to get 160 = 160 + , so
5
that = 0, and = 160 2 .
52
The number of protozoa present after 2 seconds is 160 2 = 160 5 .
This is choice (B).
Notes: (1) Always be on the lookout for the word rate. The word rate
generally indicates a derivative. The first sentence in this problem may as
5
well just say the derivative is 400 2 . Symbolically we write this as
5
= 400 2

5
5
(2) To evaluate 2 , we can formally make the substitution = . It
2
5
then follows that = .
2
5 5 2
Since there is no multiplying we place next to , and we place
2 2 5
outside of the integral sign as follows:
5 5
2 5
2 = 5 2 2

We now have
5 5
2 5 2 2 2 5
2 = 5 2 2 = 5 = 5 + = 5 2 + .

69
LEVEL 2: LIMITS AND CONTINUITY

tan( +)tan( )
4 4
57. What is lim ?
0

(A) 0
(B) 1
(C) 2
(D) The limit does not exist.
tan(+)tan()
Solution 1: If we let () = tan , then () = lim . So
0

tan( +)tan( )
( ) = lim 4 4
.
4 0

Now, the derivative of tan is sec 2 . So we have



tan( +)tan( ) 2
lim 4 4
= ( ) = sec 2 ( ) = (2) = 2.
0 4 4

This is choice (C).


Notes: (1) The derivative of the function is defined by
(+)()
() = lim
0

tan(+)tan()
In this problem () = tan , so that () = lim
0

(2) See problem 9 for the basic trig derivatives. In particular,



[tan ] = sec 2 .

1 1 1
(3) cos ( ) = . Therefore sec ( ) = =1 = 1 2 = 2.
4 2 4 cos( ) 2
4

2 2
(4) sec 2 ( ) = (sec ) = (2) = 2.
4 4

Solution 2: We use L'Hpital's rule to get



tan( +)tan( ) sec2 ( +) 2
lim 4 4
= lim 4
= sec 2 ( ) = (2) = 2.
0 0 1 4

This is choice (C).

70
Note: L'Hpital's rule says the following: Suppose that
(i) and are differentiable on some interval containing (except
possibly at itself).
(ii) lim () = lim () = 0 or lim () = lim () =

()
(iii) lim exists, and
()

(iv) () 0 for all in the interval (except possibly at itself).


() ()
Then lim = lim .
() ()

In this problem () = tan ( + ) tan( ) and () = .
4 4

5 2 +3 3
58. What is lim ?
3 2+3

(A) 1
5
(B)
3
(C) 3
(D) The limit does not exist.
5 2 +3 3 3 3
Solution: lim = lim = 3, choice (C).
3 2+3 3

()
Notes: (1) If and are polynomials, then lim = lim where
()
we have
() = + 1 1 + + 1 + 0 and
() = + 1 1 + + 1 + 0 .

(2) If = , then lim = .

(3) Combining notes (1) and (2), we could have gotten the answer to this
problem immediately by simply taking the coefficients of 3 in the
numerator and denominator and dividing.

71
The coefficient of 3 in the numerator is 3, and the coefficient of 3 in the
3
denominator is 1. So the final answer is = 3.
1

(4) For a more rigorous solution, we can multiply both the numerator and
1
denominator of the fraction by 3 to get

1 5 1
5 2 +3 3 ( 3) (5 2 +3 3 ) +3
3
= 1 = 2 3 .
3 2+3 ( 3) ( 3 2+3) 1 2 + 3

1 1
5 2 +3 3 5 lim ( 3 ) lim ( )+ lim 3 500+3

It follows that lim = 1 1 = = 3.
3 2+3 lim 12 lim ( 2 )+3 lim ( 3 ) 120+30

(5) L'Hpital's rule can also be used to solve this problem since the limit

has the form :

5 2 +3 3 2+9 2 2+18 18
lim = lim = lim = lim = 3.
3 2+3 3 2 2 6 6

Observe that we applied L'Hpital's rule three times. Each time we


differentiated the numerator and denominator with respect to to get

another expression of the form .

1 10+
59. lim ln( ) is equal to
0 10
1
(A)
10
(B) 10
(C) 10
(D) The limit does not exist.
Solution 1: If we let () = ln , then
ln(+)ln() 1 +
() = lim = lim ln( ).
0 0
1 10+
So (10) = lim ln( ).
0 10
1
Now, the derivative of ln is . So we have

1 10+ 1
lim ln( ) = (10) = , choice (A).
0 10 10

Notes: (1) The derivative of the function is defined by


(+)()
() = lim
0

72
ln(+)ln()
In this problem () = ln , so that () = lim .
0

+
(2) Recall that ln ln = ln ( ). So ln( + ) ln() = ln ( ),

ln(+)ln() 1 1 +
and therefore = [ln( + ) ln()] = ln ( ).

(3) See the notes at the end of problem 8 for a review of the laws of
logarithms.
Solution 2: We use L'Hpital's rule to get
1 1
10+ ( 10+ )( )
ln( ) 10
1 10+ 10 10 1
lim ln( ) = lim = lim = , choice (A).
0 10 0 0 1 10

Note: (1) See problem 25 or 57 for more information on L'Hpital's rule.


(2) To apply L'Hpital's rule we separately took the derivative of
10+
() = ln( ) and () = .
10
10+ 10+
(3) () = ln( ) is a composition of the functions ln and . We
10 10
therefore need to use the Chain Rule to differentiate it.
1
The first part of the Chain Rule gives us 10+ .
10

10+ 1
For the second part, it may help to rewrite as (10 + ). It is now
10 10
easy to see that the derivative of this expression with respect to is
1 1
(0 + 1) = .
10 10
tan
, for 0.
60. Let the function be defined by () = {
0 , for = 0.
Which of the following are true about f ?
I. lim () exists.
0
II. (0) exists.
III. is continuous at = 0.
(A) None
(B) I only
(C) 1I only
(D) I and II only

73
tan
Solution: lim () = lim = 1. In particular, lim () exists. So I is
0 0 0
true.
(0) = 0 by the definition of . So II is true.
tan
Since lim = 1 0 = (0), is not continuous at = 0. So III is
0
false.
The answer is therefore choice (D).
tan
Notes: (1) is a piecewise defined function. It is equal to for nonzero

values of , and it is equal to 0 for = 0.
(2) Two basic limits worth memorizing are
sin tan
lim = 1 and lim = 1.
0 0

(3) Each of the limits in note (2) is actually very easy to compute using
L'Hpital's rule. See problems 29 and 30 for details.
(4) A function is continuous at = if and only if
lim () = ().

In this problem lim () = 1 and (0) = 0. Since these two numbers


0
disagree, is not continuous at = 0.
(7+)(7)
61. Let the function satisfy lim = 12.Which of the
0
following must be true ?
I. is continuous at = 7
II. (7) exists
III. is continuous at = 7
(A) I only
(B) 1I only
(C) I and II only
(D) I, II, and III
(7+)(7)
Solution: The equation lim = 12 can be rewritten as
0
(7) = 12. In particular (7)
exists so that II is true.

74
Since is differentiable at = 7, it must also be continuous at = 7. So
I is true.
There is nothing here to suggest that is continuous at = 7. So III does
not need to be true (coming up with a specific counterexample is difficult,
but one is given in note (3) below).
Therefore, the answer is choice (C).
Notes: (1) Differentiability of a function always implies continuity of the
function.
More precisely, if is differentiable at = , then must be continuous
at = .
(+)()
(2) By definition, () = lim .
0

(7+)(7)
So (7) = lim .
0
1
( 7)2 sin , 7
(3) [Advanced Material]: Let () = { 7
0 , = 7
1 1
2( 7) sin cos , 7
Then () = { 7 7
0 , = 7
Observe that (7) = 0 (see note (4) below) so that (7) exists.
Also observe that lim () does not exist, because as approaches 7,
7
1 1
tends toward + or , and so cos oscillates between 1 and 1.
7 7
So is not continuous at = 7.
1
(7+)(7) 2 sin 0 1
(4) (7) = lim = lim
= lim sin = 0
0 0 0
1
because lim = 0 and sin is bounded (this can be proved using the
0
squeeze theorem details are left to the interested reader).
sin 7
62. lim =
0 sin 4
sin 7 74 sin 7 7 sin 7 4
Solution: lim = lim = lim
0 sin 4 0 47 sin 4 4 0 7 sin 4

75
7 sin 7 4 7 sin 1 7 1
= ( lim )( lim ) = (lim ) sin = 1 = .
4 70 7 40 sin 4 4 0 (lim ) 4 1
0

sin
Notes: (1) Once again it is worth knowing lim = 1.
0

(2) It is not hard to see that 0 if and only if 4 0 if and only if


7 0. This is why we can replace by 4 and 7 in the subscripts of
the limits above.
sin 7 74 sin 7
(3) can be rewritten as .
sin 4 47 sin 4
sin 7 7 sin 7 4
It follows that we can rewrite as .
sin 4 4 7 sin 4

(4) Using the substitution = 7, we have


sin 7 sin
lim = lim .
70 7 0

Using the substitution = 4, we have


4 1 1
lim = sin 4 = sin
40 sin 4 lim lim
40 4 0

(5) We can also solve this problem using L'Hpital's rule as follows:
sin 7 7 cos 7 7(1) 7
lim = lim = = .
0 sin 4 0 4cos 4 4(1) 4

63. lim =
11 (11)2

Solution: The function () = (11)2 has a vertical asymptote of = 11.

If is near 11, then (11)2 is positive. Therefore, lim = + .
11 (11)2
11
Notes: (1) When we substitute 11 in for into f () = (11)2 , we get .
0
This is not an indeterminate form.

For a rational function, the form where is a nonzero real number
0
always indicates that = is a vertical asymptote. This means that at
least one of lim () or lim+ () is + or . If both limits agree, then

lim () is the common value. If the two limits disagree, then lim ()

does not exist.

76
(2) A nice visual way to find the left hand and right hand limits is by
creating a sign chart. We split up the real line into intervals using the
-values where the numerator and the denominator of the fraction are
zero, and then check the sign of the function in each subinterval formed.

In this case we split up the real line into three pieces. Notice that the
cutoff points are 0 and 11 because the numerator of the function is zero
when = 0, and the denominator of the function is zero when = 11.
We then plug a real number from each of these three intervals into the
function to see if the answer is positive or negative. For example,
5
(5) = (511)2 > 0. Note that we do not need to finish the computation.
We only need to know if the answer is positive or negative. Since there

are + signs on both sides of = 11, we have that lim (11)2 = +.
11

(3) We actually do not care about the minus sign to the left of 0. We could
have left that part out of the sign chart. It is however important that we
include the zero as a cutoff point. This tells us that we can test any value
between 0 and 11 to find lim ().

64. Let be the function defined by

5 7
, 7
1 + ln| 8|
() =
15 cos( 7)
, >7
{ sin(7 ) + 3
Show that is continuous at = 7.
Solution:
5 77 5 0 5
lim () = = = = 5.
7 1+ln|78| 1+ln 1 1+0

15 cos(77) 15 cos 0 15(1) 15


lim () = = = = = 5.
7+ sin(77)+3 sin 0+3 0+3 3

77
So lim () = 5.
7

5 77
Also, (7) = = 5. So, lim () = (7).
1+ln|78| 7

It follows that is continuous at = 7.

LEVEL 3: PRECALCULUS
3
65. Let () = | cos() + |. The minimum value attained by is
2
5
(A)
2
3
(B)
2
(C) 0
1
(D)
2

Solution: The graph of = cos oscillates between 1 and 1. In other


words, |cos | 1, or equivalently 1 cos 1.
3 1 3 5 1 3 5
Adding , we have cos + . So |cos + | . In particular,
2 2 2 2 2 2 2
3 1
the minimum value attained by () = | cos() + | is , choice (D).
2 2

Notes: (1) Here is a sketch of the graph of = cos :

Note that the domain of = cos is < < . So the graph


continues to oscillate between 1 and 1 as goes to and .
3
(2) Since cos + is always positive (in fact, it is always at least
2
3 1 3 3
1 + = ), it follows that |cos + | = cos + .
2 2 2 2

78
3 3
This is why we can simply replace cos + by |cos + | in the inequality
2 2
given in the solution above.
66. For what value of will the graphs of = 3 + and 2 = 6
intersect in exactly one point?
Solution: We can find the points of intersection of the two graphs by
substituting 3 + in for in the second equation to get
(3 + )2 = 6
9 + 6 + 2 = 6
2

9 2 + 6( 1) + 2 = 0
This quadratic equation has exactly one solution when
[6( 1)]2 4(9) 2 = 0
36( 2 2 + 1) 36 2 = 0
36 2 72 + 36 36 2 = 0
72 + 36 = 0
36 = 72
36
= = or .5.
72

Notes: (1) The discriminant of the quadratic equation 2 + + = 0


is = 2 4.
(2) If > 0, the quadratic equation has two distinct real roots.
If = 0, the quadratic equation has exactly one real root.
If < 0, the quadratic equation has a pair of conjugate complex roots.
(3) In this problem we want the quadratic equation
9 2 + 6( 1) + 2 = 0
to have exactly one real root. We therefore set the discriminant equal to
0.
= 2 4 = [6( 1)]2 4(9) 2 .
67. Let () = 2 3 2 + 3 and () = 3 + 4 2 1. Find the
-coordinates of all points common to the graphs of and .
Solution: We set () = () to get 2 3 2 + 3 = 3 + 4 2 1, or
equivalently 3 5 2 + 3 + 1 = 0.

79
By observation we see that = 1 is a zero of 3 5 2 + 3 + 1, and so
we can factor 3 5 2 + 3 + 1 = ( 1)( 2 4 1).

We set 2 4 1 = 0 and solve for to get = 2 5.

So, the three -coordinates are 1, 2 + 5, and 2 5 .



Notes: (1) The rational zeros theorem says that if is a rational zero in

reduced form of the polynomial + 1 1 + + 1 + 0 , then
must divide 0 and must divide .
If we look at the polynomial 3 5 2 + 3 + 1, then 0 = 1 and 3 = 1.
So the only possible rational zeros of this polynomial are 1 and 1.
It is easy to see that 1 is actually a zero of this polynomial ( a quick way to
check if 1 is a zero of a polynomial is to simply check if the coefficients
add up to 0).
(2) The factor theorem says that = is a zero of a polynomial if and only
if is a factor of the polynomial.
In this problem, we have discovered that = 1 is a zero of the
polynomial. It follows that 1 is a factor of the polynomial.
So 3 5 2 + 3 + 1 = ( 1)( 2 + + ) for some real numbers
, , and .
(3) There are several ways to figure out the polynomial 2 + + .
Here are two methods.
Method 1: Since 2 = 3 , it follows that = 1. Since 1 = 1, it
follows that = 1. Finally, we must have 2 + 2 = 5 2 . So
1 + = 5, and = 4. So the polynomial is 2 4 1.
Method 2: We use synthetic division:
1 1 5 3 1
1 4 1
1 4 1 0
The 1 in the upper left is the zero of the polynomial. We are dividing by
1. The top line consists of the coefficients of the polynomial 3
5 2 + 3 + 1.
We begin by bringing down the first 1.

80
1 1 5 3 1

1
We now multiply this number by the zero in the upper left. So we have
(1)(1) = 1. We place this number under the 5.
1 1 5 3 1
1
1
Next we add 5 and 1.
1 1 5 3 1
1
1 4
We repeat this procedure to get (4)(1) = 4, then add 3 and 4 to
get 1.
1 1 5 3 1
1 4
1 4 1
Finally, we multiply (1)(1) = 1, then add 1 and 1 to get 0.
1 1 5 3 1
1 4 1
1 4 1 0
The bottom row gives the coefficients of the quotient which is a
polynomial of 1 degree less than the dividend..
So the quotient polynomial is 2 4 1.
(4) We can solve the equation 2 4 1 = 0 by completing the square
or using the quadratic formula.
Method 1: Here is the solution using the quadratic formula:
2 4 4164(1)(1) 420 445 445
= = = = =
2 2(1) 2 2 2

425 2(25)
= = = .
2 2

81
Method 2: Here is the solution by completing the square:
2 4 = 1
2 4 + 4 = 1 + 4
( 2)2 = 5
2 = 5
=
In the second line we added 4 to each side of the equation. We get 4 by
taking the coefficient of from the previous step (this is 4), halving this
4
number ( = 2), and then squaring the result ((2)2 = 4).
2

Observe that the intermediate result (2) tells us exactly how to factor
in the third line.
In the fourth line we used the square root property which says that if
2 = , then = .
1
68. Find the domain of k () = .
2 45

Solution: We need 2 4 5 > 0, ie. we need ( 5)( + 1) > 0.


This happens when < 1 or > 5. So the domain of is
(, ) (, ).
Notes: (1) There are two possible issues with the function .
First, there is a denominator that must be nonzero. So we must have
2 4 5 0, or equivalently 2 4 5 0.
Second, there is a square root. The expression under the square root must
be nonnegative. So we must have 2 4 5 0.
Putting these two results together, we must have 2 4 5 > 0.
(2) 2 4 5 = ( 5)( + 1).
So we must have ( 5)( + 1) > 0.
(3) There is a nice geometric method for solving the inequality
( 5)( + 1) > 0.

82
We begin by solving the corresponding equation ( 5)( + 1) = 0 to
get the two solutions = 5 and = 1. We now make a sign chart,
splitting up the real line into intervals using these two -values. We then
check the sign of the function in each subinterval formed.

In this case we split up the real line into three pieces. We then plug a real
number from each of these three intervals into the expression to see if
the answer is positive or negative. For example, (0 5)(0 + 1) < 0.
Note that we do not need to finish the computation. We only need to
know if the answer is positive or negative. From this we see
( 5)( + 1) > 0 when < 1 or > 5.
(4) Here is an alternative algebraic method for solving the inequality
( 5)( + 1) > 0.
A product of two factors is positive if both factors are positive or both
factors are negatives. So we must have
5 > 0 and + 1 > 0 or 5 < 0 and + 1 < 0
> 5 and > 1 or < 5 and < 1
>5 or < 1
Note that if > 5 and > 1, then we simply have > 5 because
5 > 1.
Similarly, < 5 and < 1 is equivalent to < 1.

83
LEVEL 3: DIFFERENTIATION


69. At which of the five points on the graph in the figure above is

2
negative and positive?
2

(A)
(B)
(C)
(D)

Solution: is negative precisely when the graph of is decreasing, and

2
is positive precisely when the graph of is concave up. So the answer
2
is point , choice (A).
Note: For a differentiable function = (),

> 0 if and only if the graph of is increasing.


< 0 if and only if the graph of is decreasing.

2
> 0 if and only if the graph of is concave up.
2

2
< 0 if and only if the graph of is concave down.
2

2
For point we have > 0 and > 0.
2

2
For point we have > 0 and 2 = 0. This point is a point of inflection

for the graph of because the graph changes concavity at this point.
2
For point we have > 0 and < 0.
2

84
2
For point we have < 0 and < 0.
2

70. Given the function defined by () = 5 3 3 5 , find all values


of for which the graph of is concave down.
2 2
(A) <<
2 2
2
(B) >
2
1 1
(C) < < 0 or >
2 2
2 2
(D) < < 0 or >
2 2

Solution: () = 15 2 15 4 and
() = 30 60 3 = 30(1 2 2 ).
The second derivative is 0 when = 0 and 1 2 2 = 0. We solve this
1 1 2
last equation for to get 2 2 = 1, or 2 = , or = =
2 2 2

Lets draw a sign chart for :

In this case we split up the real line into four pieces. Notice that the cutoff
2 2
points are , 0 and , the -values where () = 0. We then plug a
2 2
real number from each of these four intervals into to see if the answer
is positive or negative. For example, (2) = 30(2)(1 2(2)2 ) < 0.
Note that we do not need to finish the computation. We only need to
know if the answer is positive or negative. Whenever the answer comes
out positive we draw a smiley face with plus signs for eyes. Whenever the
answer comes out negative we draw a frowny face with minus signs for
eyes. This way we can see the correct concavity right inside the sign chart.

85
We can then clearly see that the function is concave down when
2 2
< < 0 or > . This is choice (D).
2 2

Notes: (1) In the sign chart above, we drew two plus signs (or two minus
signs) for visual purposes only (so that we could draw a face). There is no
other reason.
(2) Always remember positive people are happy, and negative people are
sad. This will help you to remember to draw a smiley face with the plus
signs, and a frowny face with the minus signs.
(3) When the second derivative of a function is positive, the graph of the
function is concave up.
(4) When the second derivative is negative, the graph of the function is
concave down.
(5) A point at which a function changes concavity is called a point of
inflection. In this problem there are three points of inflection. They occur
2 2
when = , = 0, and = .
2 2

(6) At a point of inflection, the second derivative of the function is either


0 or undefined.
(7) If the second derivative of a function is 0 or undefined at = , then
the function may or may not have a point of inflection there. Sometimes
the function will change concavity there, and sometimes it will not.
(8) Since each of the three zeros of the second derivative have multiplicity
1 (odd multiplicity), it follows that a point of inflection occurs at each of
them.
71. If the line 7 4 = 3 is tangent in the first quadrant to the curve
= 3 + + , then is
1
(A)
2
1
(B)
4
1
(C)
4
1
(D)
2

86
7
Solution: The slope of the line 7 4 = 3 is , and the derivative of the
4
7
given function is = 3 2 + 1. So we must have 3 2 + 1 = .
4
7 3
We subtract 1 from each side of this equation to get 3 2 = 1 = . We
4 4
2 1 1
divide by 3 to get = . By the square root property, = .
4 2
1
Since we want the first quadrant solution, we take = .
2
1
We substitute = into the equation of the line to get
2
1
7 ( ) 4 = 3.
2
7 1 1 1 1
So 4 = 3 = , and therefore = = . It follows that the point
2 2 2 4 8
1 1 1 1
of tangency is ( , ). We plug = and = into the equation for the
2 8 2 8
curve and solve for .
1 1 1
= ( )3 + +
8 2 2
1 1 1
= + +
8 8 2
1
=
2

This is choice (A).


Notes: (1) One way to find the slope of the line 7 4 = 3 is to put the
equation into slope-intercept form by solving for :
7 4 = 3
4 = 7 + 3
7 3
=
4 4
7
We can now see that the slope is .
4

(2) The slope of a line in the general form + = is .

7 7
In this problem = 7 and = 4, so that the slope is = .
(4) 4

(3) The derivative of a curve at a point is equal to the slope of the tangent
line to the curve at that point. This is why we set the derivative equal to
7
.
4

87
72. Which of the following statements about the function given by
() = 6 + 4 5 is true?
(A) The function has two relative extrema and the graph of the
function has two points of inflection.
(B) The function has one relative extremum and the graph of
the function has two points of inflection
(C) The function has two relative extrema and the graph of the
function has one point of inflection.
(D) The function has one relative extremum and the graph of
the function has one point of inflection.
Solution: () = 6 5 + 20 4 = 2 4 (3 + 10)
() = 30 4 + 80 3 = 10 3 (3 + 8)
From the factored forms of these derivatives, we see that the function has
one relative extremum and the graph of the function has two points of
inflection. This is choice (B).
10
Notes: (1) The critical numbers of are = 0 and = . Since = 0
3
is a zero of of multiplicity 4 (even multiplicity), a relative extremum
10
does not occur there. Since = is a zero of multiplicity 1 (odd
3
multiplicity), a relative extremum does occur there.
8
(2) The zeros of are = 0 and = . Since both of these zeros have
3
odd multiplicity, a point of inflection occurs at each of them.
(3) You may want to draw sign charts for and for practice and as an
extra way to check your work.

88
73. A fly is walking along a straight piece of string. The velocity ()
of the fly at time , 0 12, is given in the graph above.
According to the graph, at what time is the speed of the fly
greatest?
(A) 1
(B) 1.4
(C) 2
(D) 4
Solution: The speed is the absolute value of the velocity. So the speed is
greatest at = 2, choice (C).
Notes: (1) From the graph we see that (1) = 2 and (2) = 3.
(2) The velocity is greatest at time = 1, and the velocity is smallest at
= 2. The maximum velocity is 2 and the minimum velocity is 3.
(3) Lets suppose that the string is stretched out in front of us. We can
imagine that when the fly is walking along the string to the right that he
is moving in the positive direction, and when the fly is walking along the
string to the left that he is moving in the negative direction.
From time = 0 to time = 1, the fly is walking to the right and speeding
up. He then slows down from = 1 to approximately = 1.3 at which
point he changes direction. From time = 1.3 to time = 2, the fly is
walking to the left and speeding up. He then slows down from = 2 to
= 4 at which point he begins speeding up again.
Note that after time = 1.3, the fly is walking to the left for the duration
of his trip.

89
(4) The speed of the fly at time = 1 is 2, and the speed of the fly at time
= 2 is 3.
At time = 1 he is walking to the right, and at time = 2 he is walking to
the left.
1 1
74. If () = 3 ( + 3)2 for all , then the domain of is
(A) {| is a real number}
(B) {| 0}
(C) {| 0 and 2}
(D) {| > 3 and 0}
1 1 1 2
1 1
Solution: () = 3 ( + 3)2 + ( + 3)2 3 .
2 3

From the form of the derivative, we see that cannot be equal to 0, and
we must have + 3 > 0, or equivalently, > 3.
So the answer is choice (D).
1
3
Notes: (1) 3 =
1
(2) ( + 3)2 = + 3
1
1 1
(3) ( + 3)2 = 1 =
(+3)2 +3

2
1 1
(4) 3 = 2 = 3 .
3 2

3
+3
(5) Using the previous notes, we can rewrite as () = + 3
2+3 3 2
3
(6) Denominators cannot be 0. So we must have 2 0, and this is
equivalent to 0.
(3) Expressions under square roots (and in fact all even roots) must be
nonnegative. So we must have + 3 0. Since we have this square root
in the denominator, it also cannot equal zero. So we must, in fact, have
+ 3 > 0, or equivalently > 3.
(5) Cube roots (and in fact all odd roots) do not cause any problems. So
3
we do not need to worry about .

90

75. [sin1 ( )] =
2
1
(A)
4 2
1
(B)
4 2
1
(C)
21 2
1
(D)
21 2

Solution:
1 1 1 1 1 1
[sin1 ( )] = ( )= = = = .
2 2 2 2 2 2 4 2
1() 21 41 4 4(1 )
2 4 4

This is choice (B).


Notes: (1) Note that sin1 is the same thing as arcsin . This is the

inverse of the function sin , .
2 2

(2) This notation can actually be confusing. To see why I say this, note that
sin2 is an abbreviation for (sin )2 . But sin1 is not an abbreviation
1
for (sin )1 = .
sin

sin is the inverse function, whereas (sin )1 is the reciprocal of


1

sin .
(3) You should know the derivatives of the six basic inverse trig functions:

[ ] = [ ] =
||


[ ] = [ ] =
||


[ ] = [ ] =
+ +

(4) Let () = sin1 ( ). Then f () = (()) , where () = sin1
2

and () = .
2

To take the derivative of requires the chain rule. We have


1 1
() = (()) () = 2
( ).
2
1()
2

91
1 1 1
(5) To get from 2
( ) to the expression we simply
2 2
1() 21
2 4
multiplied the two numerators together (1 1 = 1), and we multiplied
2 2
the two denominators together (1 ( ) 2 = 2 1 ).
2 4

(6) We would now like to move the 2 in the denominator inside of the
square root. to do this we write 2 = 4, and use the fact that for any
nonnegative real numbers and , we have = .
1 1 1
So we have = = .
2 2 2
21 41 4 4(1 )
4 4

2 2
(7) Finally, we distribute the 4 to get 4 (1 ) = 4 4 ( ) = 4 2.
4 4

76. The derivative of is graphed above. Give a value of where


has a local minimum.
(A) 0
(B) 1
(C) 4
(D) 6
Solution: From the graph of we see that (6) = 0, so that = 6 is a
critical number of .
We have () < 0 for near 6 and to the left of 6, and () > 0 for
near 6 and to the right of 6.

92
It follows that () is decreasing for a little less than 6, and () is
increasing for a little greater than 6. Therefore has a local minimum
at = 6, choice (D).
Notes: (1) Recall that a critical number of a function is a real number
in the domain of such that () = 0 or () is undefined.
The function here has two critical numbers: = 1 and = 6.
(2) If a function is decreasing to the left of , and increasing to the right of
, then attains a local minimum at .
In terms of derivatives, if () < 0 for < , and () > 0 for > ,
then attains a local minimum at .
This happens at = 6.
(3) If a function is increasing to the left of , and decreasing to the right of
, then attains a local maximum at .
In terms of derivatives, if () > 0 for < , and () < 0 for > ,
then attains a local maximum at .
This happens at = 1.
(4) This method of finding the local minima and maxima of a function is
called the first derivative test.
(5) Lets make a sign chart

In this case we split up the real line into three pieces. Notice that the
cutoff points are 1 and 6, the critical numbers of . We then note that
() > 0 for < 1 and > 6, and () < 0 for 1 < < 6 (just look at
the given graph of ). Finally, we make a quick sketch that decreases
across the interval where there is a minus sign, and increases across the
intervals where there is a plus sign. We can then clearly see that there is
a local maximum at = 1 (not needed in this problem) and a local
minimum at = 6.

93
(5) The sketch of the graph in the sign chart is not meant to be completely
accurate. It is only being used to demonstrate where the function is
increasing and decreasing, so we can determine if there is a minimum or
maximum (or neither) at each critical number.
(6) Many students get confused about the distinction between and .
The graph that is given in the problem is the graph of , the derivative of
g, whereas the graph that we sketched in the sign chart gives the general
shape of the graph of .
Note that the graph of in the sign chart is increasing precisely when the
given graph of is above the -axis, and the graph of in the sign chart
is decreasing precisely when the given graph of is below the -axis.
The maximum and minimum points of in the sign chart at = 1 and
= 6, respectively, correspond to the zeros of , ie. where the given
graph of crosses the -axis.

77. If sec( 2 ) = , then =

2 sec( 2 )
(A)
1 sec( 2 )
2 tan( 2 )
(B)
cos( 2 ) 2 tan( 2 )
1
(C)
sec( 2 ) tan( 2 )
1 tan2 ( 2 )
(D)
tan2 ( 2 )

Solution: We have

sec( 2 ) tan( 2 ) ( 2 + 2) =


2 sec( 2 ) tan( 2 ) + 2 sec( 2 ) tan( 2 ) =


2 sec( 2 ) tan( 2 ) = 2 sec( 2 ) tan( 2 )


2 sec( 2 ) tan( 2 ) = (1 2 sec( 2 ) tan( 2 ))

2 sec( 2 ) tan( 2 )
2 2 2
=
1 sec( ) tan( )

94
2 tan( 2 )
2 2 2
=
cos( ) tan( )
This is choice (B).
Notes: (1) The given equation defines the dependent variable implicitly
as a function of . We therefore used implicit differentiation to find the

derivative . See problem 45 for more information on implicit

differentiation.
(2) Recall that the derivative of sec is sec tan .

To differentiate 2 , we use the product rule to get 2 + 2.

We then used the chain rule to get the derivative of sec( 2 ).



(3) After differentiating we must solve for .

We first distributed on the left as follows:



sec( 2 ) tan( 2 ) ( 2 + 2) = 2 sec( 2 ) tan( 2 ) + 2 sec( 2 ) tan( 2 )


We then brought the term on the left with over to the right hand side


of the equation, and then factored out . This gave the following:


2 sec( 2 ) tan( 2 ) = (1 2 sec( 2 ) tan( 2 )).


We then got by itself by dividing by 1 2 sec( 2 ) tan( 2 )):

2 sec( 2 ) tan( 2 )
=
1 2 sec( 2 ) tan( 2 )

Unfortunately, this expression doesnt look like any of the answer choices.
This is easily fixed however by multiplying both the numerator and
denominator by cos( 2 ).
1
Note that cos( 2 ) sec( 2 ) = 1 (because sec( 2 ) = ).
cos( 2 )

So we have
cos( 2 )[2 sec( 2 ) tan( 2 )] 2 tan( 2 )
= .
cos( 2 )[1 2 sec( 2 ) tan( 2 )] cos( 2 ) 2 tan( 2 )

95
Alternative solution using partial derivatives:
Let (, ) = sec( 2 ) . Then

sec( 2 ) tan( 2 )(2) sec( 2 ) tan( 2 )(2)
= = =
sec( 2 ) tan( 2 )( 2 )1 1sec( 2 ) tan( 2 )( 2 )

tan( 2 )(2) 2 tan( 2 )


= = .
cos( 2 )tan( 2 )( 2 ) cos( 2 ) 2 tan( 2 )

This is choice (B).


Note: Partial differentiation is not an AP Calculus topic, and therefore a
full explanation of this solution lies outside of the scope of this book.
78. A point moves in a straight line so that its distance at time from
a fixed point of the line is 2 3 9 2 + 12. The total distance
that the point travels from = 0 to = 4 is
(A) 32
(B) 33
(C) 34
(D) 35
Solution: The velocity of the point is
() = 6 2 18 + 12 = 6( 2 3 + 2) = 6( 1)( 2).
So the point might change direction at = 1 and = 2.
Let () = 2 3 9 2 + 12.
We have (0) = 0, (1) = 5, (2) = 4, and (4) = 32.
So the total distance traveled is
(5 0) + (5 4) + (32 4) = 5 + 1 + 28 = 34.
This is choice (C).
Notes: (1) A common mistake here would be to compute (4) (0).
This gives the total displacement of the point. In other words it tells us
how far the point is from its starting point ( = 0) at the time = 4.
If the point is travelling to the right, and never changes direction, then this
computation would give the correct answer. In this problem however the
point does change direction.

96
(2) To find out when the point changes direction, we set the velocity
() = () equal to 0.
In this case the position function is () = 2 3 9 2 + 12. It follows
that the velocity function is () = () = 6 2 18 + 12.
(3) Just because () = 0, it is not necessarily true that the point changes
direction. For this particular problem it is not necessary to determine if
the point actually changes direction, so we will not need to make a sign
chart.
(4) Once we find out that the velocity is 0 at = 1 and = 2, we can just
compute the distance traveled from = 0 to = 1, then from = 1 to
= 2, and then from = 2 to = 4. Adding up these three distances
gives us the total distance traveled by the point.
So we wish to compute
|(1) (0)| + |(2) (1)| + |(4) (2)|.
The quickest way to do this is to first compute (0), (1), (2), and (4),
then perform the three subtractions, dropping any minus signs, and finally
adding up the three resulting distances.
79. * Two particles start at the origin and move along the -axis. For
0 10, their position functions are given by = cos and
= ln(2) + 1. For how many values of do the particles have
the same velocity?
(A) None
(B) One
(C) Two
(D) Three
2 1
Solution: = sin and = = . We want to find out for how many
2
1
values of between 0 and 5 that = , ie. sin = . Now we use our

graphing calculator. Make sure that your calculator is in radian mode
(Press MODE and select Radian). Press Y= and next to Y1 enter sin X, and
next to Y2 enter 1/X. Then press WINDOW and set Xmin to 0 and Xmax to
10. Press GRAPH to display the graph (if you do not see the graph well,
you may have to adjust Ymin and Ymax as well, perhaps Ymin = 2, and
Ymax = 2).

97
You will see that the two graphs intersect three times, choice (D).
2
80. Consider the equation 2 + + 2 = 2. Find at (0,1).
2

Solution: We differentiate implicitly to get



2 + ( + ) + 2 =0


2 + + + 2 =0


+ 2 = 2


( + 2) = 2

2
= .
+2

2
So we have
2

( +2)(2[ ( +)+ ])(2 )( ( +)+ +2 )

=
( +2)2

1
At (0,1), we have = , and so
2
1 1 1
2 (2)(2[1(1)+1( )]])(1)(1+2( )) 2(2 )+(0)
2 2 2
= = = .
2 22 4

LEVEL 3: INTEGRATION
81. The area of the region in the first quadrant bounded by the graph
of = 2 1 3 , the line = 1, and the -axis is
2
(A)
9
22
(B)
9
23
(C)
9
4
(D)
9
3 1
1 1 2 2
Solution: 0 2 1 3 = (1 3 )2 |0 = , choice (A).
3 3 9

98
Notes: (1) To compute the area under the graph of a function that lies
entirely above the -axis (the line = 0) from = to = , we simply
integrate the function from to .

In this problem, the function is = 2 1 3 , = 0, and = 1.

Note that 2 0 for all , 1 3 0 for all in the interval [0,1], and
therefore 2 1 3 0 for all in the interval [0,1]. It follows that the
graph of = 2 1 3 lies entirely above the -axis for 0 1.

(2) = 0 and = 1 are the only two zeros of = 2 1 3 .

(3) To evaluate 2 1 3 , we can formally make the substitution


= 1 3 . It then follows that = 3 2 .
Since there is no 3 multiplying 2 we place 3 to the left of 2 , and
1
we place outside of the integral sign as follows:
3
1
2 1 3 = 3 1 3 (3) 2

We now have
1 1
2 1 3 = 3 1 3 (3) 2 = 3 =
3
1 3 3
1 1 2 1 2 2
=
2 3 + = 2 + = (1 3 )2 +
3 3 3 3 9
2

We get the second equality by replacing 1 3 by , and 3 2 by .


We get the fourth equality by applying the power rule for integrals
+1
= +1
+ .

And we get the rightmost equality by replacing with 1 3 .


(4) With a little practice, we can evaluate an integral like this very quickly
with the following reasoning: The derivative of 1 3 is 3 2 . So we
1
artificially insert a factor of 3 next to 2 , and outside the integral
3
sign. Now to integrate 3 2 1 3 we simply pretend we are
integrating but as we do it we leave the 1 3 where it is. This is
essentially what was done in the above solution.

99
Note that the 3 2 goes away because it is the derivative of 1 3 .
We need it to be there for everything to work.
(5) If we are doing the substitution formally, we can save some time by
changing the limits of integration. We do this as follows:
1
2
1 1 2
1 0
1 3
= 1 3 (3) =
0 3 0 3 1
3 0
1 2 2 2 2
= 2 |1 = (0) ( ) (1) = .
3 3 9 9 9

Notice that the limits 0 and 1 were changed to the limits 1 and 0,
respectively. We made this change using the formula that we chose for
the substitution: = 1 3 . When = 0, we have = 1 03 = 1. And
when = 1, we have = 1 13 = 0.
Note that this method has the advantage that we do not have to change
back to a function of at the end.

82. A particle moves in a straight line with velocity () =


beginning at time = 0. How far is the particle from its starting
point at time = 4 ?
(A) 0
(B) 2
8
(C)
3
40
(D)
3

4 2 2 3 4 16 24 16 8
Solution: 0 ( ) = ( 2 )|0 = 8 = =
2 3 3 3 3 3

This is choice (C).


Notes: (1) The particle actually changes direction at time = 1. To see
this note that (1) = 0, () < 0 for 0 < < 1, and () > 0 for > 1.
We do not actually need to worry about the particle changing direction in
this problem since we only want the particles position relative to its
starting point.
(2) If we know the velocity function of a particle, then we can find its
position function by integrating the velocity . That is () = ().

100

(3) () = () () gives the distance between the position of
the particle at time and the position of the particle at time .
(3) If we were asked to find the total distance traveled by the particle,
then we would need to be more careful. In this case we would compute
4
0 | |. See problem 92 for an example like this.

83. The acceleration () of a body moving in a straight line is given


in terms of time by () = 3 2. If the velocity of the body is
10 at = 1 and if () is the distance of the body from the origin
at time , then (5) (1) =
10
(A)
3
20
(B)
3
40
(C)
3
80
(D)
3

Solution: () = () = (3 2) = 3 2 + .
We are given that 10 = (1) = 3(1) 12 + = 3 1 + = 2 + . So
= 10 2 = 8. It follows that () = 3 2 + 8.
5 5 3 3 5
(5) (1) = 1 () = 1 (3 2 + 8) = ( 2 + 8) |1
2 3
3 53 3 (1)3 80
= ( (5)2 + 8 5) ( (1)2 + 8 1) = .
2 3 2 3 3
This is choice (D).
Notes: (1) , , and are generally used for position, velocity, and
acceleration, respectively.
(2) The velocity function is the derivative of the position function, and the
acceleration function is the derivative of the velocity function.
That is () = () and () = () = ().
In terms of integrals, () = () and () = ().

101
84. The area of the region completely bounded by the curve
= 2 + 2 + 5 and the line = 2 is
32
(A)
3
24
(B)
3
16
(C)
3
8
(D)
3

Solution: Lets draw a picture:

The area of the shaded region is


3 3 3 3
1[( 2 + 2 + 5) 2] = 1( 2 + 2 + 3) = ( 3
+ 2 + 3) |1

33 (1)3 32
= ( + 32 + 3 3) ( + (1)2 + 3(1)) = .
3 3 3

This is choice (A).


Notes: (1) The area between the curves = () and = () from

= to = is |() ()| . The -values and are usually
the -coordinates of points of intersection of the two graphs.
(2) If we are allowed to use our graphing calculator to solve this problem,
we can simply graph the functions () = 2 + 2 + 5 and () = 2
in our calculator, use the intersect feature under the CALC menu to find
that = 1 and = 3. We would then see that () lies above (),
3
and so we would compute 1[( 2 + 2 + 5) 2] .

102
We can do this last computation by hand (as was done in the solution
above), or by using the integration feature in our calculator.
(3) If we cannot use a calculator, we can find and by setting () equal
to (). We have 2 + 2 + 5 = 2, so that
0 = 2 2 3 = ( + 1)( 3).
This last equation has solutions = 1 and = 3. So = 1 and = 3.
(4) The graph of the quadratic function () = 2 + + is a

parabola with vertex at = . If > 0, the parabola opens upwards.
2
If < 0, the parabola opens downwards.
In this problem we have that the graph of () = 2 + 2 + 5 is a
2
downward facing parabola with vertex at = = 1.
2(1)

The -coordinate of this point is then (1) = 12 + 2(1) + 5 = 6.


So the vertex of the parabola is the point (1,6).
(5) Notes (3) and (4) allow us to sketch the region shown above without
using a calculator.
85. Let and be continuous functions such that () = () for

all . It follows that () =

(A) () ()
(B) () ()
(C) () ()
(D) () ()

Solution: () = () = () | = () ().

This is choice (B).


Notes: (1) The second Fundamental Theorem of Calculus says that if is

a Riemann integrable function on [, ], then () = () ()
where is any antiderivative of .
(2) If a function f is continuous on [, ], then is Riemann integrable on
[, ].
(3) Since is an antiderivative of , by the notes above, we have

103

() = () ().

86. If = + 3 and if = 5 when = 0, then =

2
(A) 5 +3
2
(B) +3
2
(C) 5 + +3
2
(D) 4 + +3

Solution: We have = + 3 = ( + 3). We separate variables to


get = ( + 3). Now we integrate both sides of this equation to get

ln|| = 2 + 3 + . We change to exponential form to get
2 +3+ 2 +3
|| = = .
2 +3 2 +3
So = = where is the constant .
Now we substitute in = 0 and = 5 to get 5 = 0 = 1 = . So
2
= 5, and we have = 5 +3 , choice (A).
Note: See problem 52 for more information on separable differential
equations.
1
87. The average value of over the interval 1 4 is

1
(A)
2
(B) 0
1
(C)
2
2
(D)
3
1 1 4
1 4 1 1 4 1 2 2
Solution: = 1 2 = 2 2 |1 = (2 1) = .
41 1 3 3 3 3

This is choice (D).


Notes: (1) The average value of the function over the interval [, ] is
1
() .

104
1
1 1
(2) = 1 = 2 .
2
1
1 1 1 1
1 2 1
(3) = 2 = 1 + = 2 + = 2 2 + = 2 2 + .
2
2

sin for < 0 1


88. Given () = { 2 , we have 2 () =
for 0
1 1
(A)
6
1
(B)
6
1 1
(C)
6
1
(D)
6
1 0 1
Solution: 2 () = 2 sin + 0 ( 2 )
1 0 3 2 1 1 1 1 1 1
= cos |2 + ( ) |0 = ( + ) + ( ) = .
3 2 3 2 6

This is choice (B).


Notes: (1) is a piecewise defined function. It is equal to sin for
negative values of , and it is equal to 2 for nonnegative values of .
(2) We have the following property of definite integrals:

() = () + ()
for any real number between and .
In this question = 2, = 1, and = 0. Between 2 and 0, we can
replace () by sin , and between 0 and 1, we can replace () by
2 .

89. If () = 2 2 , then (3() 2) =

(A) 2 2
(B) 2 2 2 2
(C) 2 2 2 2
(D) 3 2 3 2

Solution: (3() 2) = 3 () 2

105
2 2
= 3( 2 2 ) | = 3 2 3 2 ( 2 2 ) = 2 2 2 2 .
2

This is choice (B).


5
90. If 2 (3 + ) = where and are real numbers, then
15+
6+ () =

(A) 3
(B)

(C)
3
(D) 3 +
Solution: If we let = 3 + , then = 3, and we have
5 1 5 1 15+
= 2 (3 + ) = 2 (3 + ) 3 = 6+ () .
3 3
1 15+
But in the expression 6+ () , is a dummy variable. So it is also
3
1 15+
equal to 6+ () .
3
1 15+ 15+
So we have 6+ () = , and so 6+ () = 3, choice (A).
3

91. * Calculate the approximate area under the curve () = 2 and


bounded by the lines = 2 and = 3 by the trapezoidal rule,
using three equal subintervals.
(A) 1.285
(B) 3.176
(C) 6.352
(D) 12.704
3 1 1 7 8
Solution: 2 () [(2) + 2 ( ) + 2 ( ) + (3)]
2 3 3 3

1 49 64
= [4 + 2 ( ) + 2 ( ) + 9] 6.352
6 9 9
This is choice (C).
Notes: (1) The trapezoidal rule says
1
() 2 [(0 ) + 2(1 ) + + 2(1 ) + ( )] ,

106
where the interval [, ] is partitioned into equal subintervals with
endpoints = 0 , 1 ,,1 , = .
In this problem, we have = 2, = 3, = 3, and () = 2 .
1
So = is the length of each subinterval.
3
+
(2) Recall the formula for the area of a trapezoid: = ( 1 2 ). In other
2
words we take the average of the bases times the height.
(3) Here is a picture of the trapezoidal rule being used in this problem:

1
Note that all three trapezoids have a height of (the height runs along
3
the -axis).
7
The two bases of the leftmost trapezoid are (2) and ( ). It follows that
3
7
(2)+( ) 1
3
the area of the leftmost trapezoid is .
2 3
7 8
The two bases of the middle trapezoid are ( ) and ( ). It follows that
3 3
7 8
( )+( ) 1
3 3
the area of the middle trapezoid is .
2 3
8
The two bases of the rightmost trapezoid are ( ) and (3). It follows
3
8
( )+(3) 1
3
that the area of the rightmost trapezoid is .
2 3

Finally, we get the area of the shaded region by adding these three areas:

107
7 7 8 8
3 (2)+( ) 1 ( )+( ) 1 (3)+(3) 1
3 3 3
2 () 2
+
3 2
+
3 2

3
1 1 7 8
= [(2) + 2 ( ) + 2 ( ) + (3)].
2 3 3 3

92. A point moves in a straight line so that its velocity at time is


2 3 5 2 + 2. What is the total distance that the point travels
from t = 0 to t = 3 ?
Solution: The velocity of the point is
() = 2 3 5 2 + 2 = (2 2 5 + 2) = (2 1)( 2)
1
So the point might change direction at = and = 2.
2

4 5 3
() = (2 3 5 2 + 2) = 2

3
+ 2 + .

4 5 3
Let () = + 2.
2 3
1 7 4 9
We have (0) = 0, ( ) = , (2) = , (3) = .
2 96 3 2

The total distance traveled by the point is


3 1 1
0 |()| = | (2) (0)| + |(2) (2)| + |(3) (2)|
7 45 35
= + + = , 7.312 or 7.313.
96 32 6

Notes: (1) A common mistake here would be to simply compute


3
0 (2 3 5 2 + 2) . This gives the total displacement of the point. In
other words it tells us how far the point is from its starting point ( = 0)
at the time = 3.
If the point is travelling to the right, and never changes direction, then this
computation would give the correct answer. In this problem however the
point does change direction.
(2) To find out when the point might change direction, we set the velocity
() (the given function) equal to 0.

108
(3) Just because () = 0, it is not necessarily true that the point changes
direction. For this particular problem it is not necessary to determine if
the point actually changes direction, so we will not need to make a sign
chart.
1
(4) Once we find out that the velocity is 0 at = and = 2, we can just
2
1 1
compute the distance traveled from = 0 to = , then from = to
2 2
= 2, and then from = 2 to = 3. Adding up these three distances
gives us the total distance traveled by the point.
So we wish to compute
1 1
| ( ) (0)| + |(2) ( )| + |(3) (2)|.
2 2
1
The quickest way to do this is to first compute (0), ( ), (2), and (3),
2
then perform the three subtractions, dropping any minus signs, and finally
adding up the three resulting distances.
(5) If we can use a calculator for this problem, we can compute
3 3
0 |()| = 0 |2 3 5 2 + 2| using our TI-84 calculator by first
selecting fnInt( (or pressing 9) under the MATH menu, and then going to
the MATH menu again, moving right once to NUM and selecting abs( (or
pressing 1). The display will show the following:
fnInt(abs(
We then type the following: fnInt(abs(2X^3 5X^2 + 2X), X, 0, 3), and
press ENTER.
The display will show 7.312502794. So we can answer 7.312 or 7.313.

LEVEL 3: LIMITS AND CONTINUITY


93. If lim () = ( lim ) for all in the interval (, ), which of

the following must be true?
(A) is continuous on (, ).
(B) is differentiable on (, ).
(C) is a polynomial.
(D) () = 0 for some (, ).

109
Solution: lim = . Therefore (lim) = (). So the given condition

can be rewritten as lim () = ( ). This is precisely the definition for

to be continuous at = .
Since we are given that this condition is true for all in (, ), the answer
is choice (A).
1
94. Let be defined by () = + 3 for 2 10.
17 2
() + 2 for 2 4
Let be defined by () = { .
| 8| for 4 < 10
Is continuous at = 4? Use the definition of continuity to
explain your answer.
Solution:
1
lim () = (4) + 2 = + 4 3 + 2 = 1 + 1 + 2 = 4.
4 174 2

lim () = |4 8| = |4| = 4
4 +

So lim () = 4.
4

Also, (4) = (4) + 2 = 4. So, lim () = (4).


4

It follows that is continuous at = 4.


95. A 5000 gallon tank is filled to capacity with water. At time = 0,
water begins to leak out of the tank at a rate modeled by (),
measured in gallons per hour, where
300
, 04
() = { + 1
500 0.5 , > 4
Is continuous at = 4? Show the work that leads to your
answer.
Solution:
300(4)
lim () = = 240.
4 4+1

lim () = 500 0.5(4) = 500 2


4 +

110
Since lim () lim+ (), lim () does not exist.
4 4 4

It follows that is not continuous at = 4.


2 +412
96. The function () = 2 has a removable discontinuity at
+310
= . Find , and define a function such that is continuous
at = and () = () for all in the domain of .
2 +412 (2)(+6) +6
Solution: () = = = , 2.
2 +310 (2)(+5) +5

2+6 8
So lim () = = .
2 2+5 7

It follows that has a removable discontinuity at = 2. So = .

2 +412
2 +310 , 2
Define by () = { 8
.
, =2
7

Notes: (1) If is not continuous at = , then the discontinuity is


removable if lim () exists (and is a finite number). Otherwise the

discontinuity is nonremovable.
In this problem = 2 is a removable discontinuity of because
8
lim () = . On the other hand, = 5 is a nonremovable
2 7
discontinuity of because lim () does not exist (see problem 63 for
5
more information on how to compute this limit).
(2) If = is a removable discontinuity of , then we can extend to a
function such that is continuous at = , and () = () for all
in the domain of . We simply set () = lim (), and we let

() = () for all .

111
LEVEL 4: DIFFERENTIATION
97. Suppose that is an even function (so that () = () for all
), and that () exists. Then () must be equal to
(A) ()
(B) ()
1
(C)
()
1
(D)
()

Solution: () = ()(1) = (). So () = (), and


therefore () = (), choice (B).
Notes: (1) We used the chain rule here. If we let () = (), then is
the composition () = (()), where () = () and () = .

So () = (()) () = ()(1) = ().

But we also have () = (), so that () = ().


Therefore () = ().
(2) We just showed that if is an even function, is an odd function.
(3) It can also be shown that the derivative of an odd function is an even
function.
(4) See problem 5 for more information about even and odd functions.
1
98. The function () = 10 4 7 1 , > is invertible. The
2
derivative of 1 at = 3 is
7
(A)


(B)
7
(C) 1
1
(D)
33

Solution: First note that (1) = 10(1)4 7 11 = 10 7 = 3. It


follows that 1 (3) = 1.

112
Now, () = 40 3 7 1 , so that
1 1
(1 ) () = =
(1 ()) 40 3 7 1
where = 1 ().
1 1 1
So (1 ) (3) = = = , choice (D).
40(1)3 7 11 407 33

Notes: (1) Many students find it tricky to find the derivative of an inverse,
but the procedure is fairly simple.
Suppose we want to find ( 1 )(). First find (). Then take the
1 1
reciprocal to get (). Finally, replace by = 1 () to get 1 ()) .
(

Now the trickiest part comes when we need to plug in a value. We need
to remember that to compute ( 1 )() we plug in 1 (), and not .
This is why we started the given problem by figuring out what 1 (3) was.
(2) The question what is 1 (3)? is equivalent to of what is equal to
3? Equivalently, we need to find such that () = 3.
Often in calculus problems on the AP exam, this value can be found fairly
easily by simple observation, or guessing and checking simple numbers.
In this example, it is not too hard to see that (1) = 3, so that
1 (3) = 1.
(3) If a calculator were allowed to be used for this problem, then we could
solve the equation () = 3 by graphing = 10 4 7 1 and = 3
in our graphing calculator, and using the intersect button to find the
-coordinate of the point of intersection.
1
Be careful to find the -value that is greater than , since this is the
2
restriction placed on the given function (this was done to ensure that the
function passes the horizontal line test, and therefore is invertible).
(4) Another notation that is used that some students find helpful is
1
= .



In this problem, = 40 3 7 1 .

1
So = .
40 3 7 1

113
But remember that we are plugging in 1 for (and not the given value of
1 1 1
3). So we get |=1 = 3 11 = = .
40(1) 7 407 33

99. The radius of a spherical balloon is decreasing at a constant rate


of 0.5 centimeters per second. At the instant when the volume
becomes 288 cubic centimeters, what is the rate of decrease, in
square centimeters per second, of the surface area of the balloon?
(A) 24
(B) 48
(C) 64
(D) 72
4
Solution: Recall that the volume of a sphere with radius is = 3 ,
3
and the surface area of a sphere with radius is = 4 2 . We are given

that = 0.5, and we are being asked to find when = 288.

First note that when = 288, we have


4
288 = 3
3
3
288 = 3
4
216 = 3
6=
Now,

= 8 = 8(0.5) = 4.


|=6 = 4(6) = 24.

So the surface area of the balloon is decreasing at a rate of 24 cm2 /sec.


Therefore, the answer is choice (A).
Notes: (1) Observe that one can get the formula for the surface area of a
sphere by differentiating the formula for the volume of a sphere. This is
just a little trick that can be used to reduce the number of formulas to
memorize.
(2) Remember that the word rate generally indicates a derivative.
Increasing at a rate of indicates a positive derivative, and decreasing at
a rate of indicates a negative derivative.

114
(3) A radius is a length and is therefore measured in single units (in this
case centimeters). Area (and in particular surface area) is measured in
square units. Volume is measured in cubic units.
The type of units being mentioned usually gives a big hint as to what
measurement is being given or asked for.
(4) This is a related rates problem. In a related rates problem we
differentiate the independent and dependent variables with respect to a
new variable, usually named , for time.
(5) A related rates problem can be pictured as a dynamic (moving) process
that gets fixed at a specific moment in time.
For this problem we can picture a sphere shaped balloon deflating. We
then freeze time at the moment when the volume is 288 cm3.
At this moment in time we want to know what the rate of decrease of the
surface area is. The word rate indicates that we want the derivative of

surface area with respect to time, .

Dont forget to apply the chain rule when differentiating the right hand

side. In this case, the derivative of is .


100. If = , then =

(A) !
(B)
(C)
(D)
Solution: Lets take the first few derivatives and look for a pattern:

2
= ( ) = 2
2

3
= 2 ( ) = 3
3

That should be enough to see the pattern.

115

= . This is choice (D).

Notes: (1) See problem 12 for an explanation of how the chain rule is used
to differentiate = .
3
(2) In the equation 3 = 3 , observe how the exponent of (which

is 3) matches the derivative we are taking (the 3rd). So we can generalize
this to an nth derivative by changing the power of 3 to a power of n.
1
101. For small values of , the function 3 is best approximated by
27+
27
(A)
81
81
(B)
81
27
(C)
243
81
(D)
243

Solution: We are being asked to find the linear approximation of


1 4
1 1
() = 3 = 3 when = 27. Well we have () = 3 . So
3
4
(27) 1 1 1 1
= (27) 3 = = .
3 3 81 243

Now,
( + ) () ()
(27 + ) (27) (27)
1 1 1
3
27+ 3 243
1 1 1 81 81
3 = = .
27+ 3 243 243 243 243

This is choice (D).


Notes: (1) Recall the definition of the derivative:
(+)()
() = lim
0

(+)()
So for small values of we have () . Equivalently,

() ( + ) ().

116
(2) Geometrically () is the slope of the tangent line to the graph of
(+)()
at the point (, ()), whereas is the slope of the secant line

passing through the points (, ()) and ( + , ( + )). For very
small values of these two slopes are almost the same. For a picture of
this, see the figures in the notes following problem 102.

2.5 2.6 2.7 2.8


() 7 7.4 7.7 7.9

102. Let be a function that is concave down for all in the closed
interval [2,3], with selected values shown in the above table.
Which of the following inequalities must be true?
(A) (2.7) > 3
(B) 2 < (2.7) < 3
(C) 1 < (2.7) < 2
(D) 0 < (2.7) < 1
Solution: Since is concave down in the interval [2,3], the tangent line to
when = 2.7 lies above the graph of in this interval.
(2.8)(2.7) 7.97.7 .2
So (2.7) > = = = 2.
2.82.7 .1 .1
(2.7)(2.6) 7.77.4 .3
Also (2.7) < = = = 3.
2.72.6 .1 .1

So 2 < (2.7) < 3, choice (B).


Notes: (1) Here is a sketch of the graph of the function together with the
tangent line to the graph at = 2.7.

117
Observe that the graph is below the tangent line in the interval [2,3].
(2) Now lets also look at the secant line through the points (2.7,7.7)
and (2.8,7.9).

Notice that the tangent line has a greater slope than the secant line.
The slope of the tangent line is (2.7), and the slope of the secant line
(2.8)(2.7) (2.8)(2.7)
is . So we have (2.7) > .
2.82.7 2.82.7

(3) Now lets also look at the secant line through the points (2.6,7.4)
and (2.7,7.7).

Notice that the tangent line has a smaller slope than the secant line.
The slope of the tangent line is (2.7), and the slope of the secant line
(2.7)(2.6) (2.7)(2.6)
is . So we have (2.7) < .
2.72.6 2.72.6

118

103. Consider the differential equation = 1 (2 3 5). Let

= () be the particular solution to the differential equation
that passes through (1,1). Write an equation for the line tangent
to the graph of at the point (1,1), and use the tangent line to
approximate (1.1).
Solution: The slope of the tangent line at (1,1) is

= |(1,1) = 11 (2(1)3 5) = 0 (2 5) = (1)(3) = 3.

So an equation of the tangent line in pointslope form is


= ( ).
When = 1.1, we have 1 = 3(1.1 1) = 3(. 1) = .3.
So = .3 + 1 = .7.
It follows that (1.1) . .
Notes: (1) To find the slope of the tangent line to a function = () at a

point (0 , 0 ), we take the derivative = (), and substitute in 0 for

and 0 for .

In this problem, we were already given , and so we simply needed to

substitute in the -coordinate of the point for and the -coordinate of the
point for .
(2) The point-slope form of an equation of a line is
= ( )
where is the slope of the line and (, ) is any point on the line.
It is generally easiest to write an equation of a line in point-slope form once
the slope of the line and a point on the line are known.
In this problem, the slope is 3 and the point is (1,1).

119

0 1 2 3 4
(minutes)
()
0 2.2 4.5 6.8 8.3
(pints)
104. Gasoline is dripping out of a gas pump, filling up a bucket. The
amount of gasoline in the bucket at time , 0 4, is given by
a differentiable function , where is measured in minutes.
Selected values of (), measured in pints, are given in the table
above. Is there a time , 1 3, at which () = 2.3 ? Justify
your answer.
Solution: Since is differentiable, it follows that is continuous on [1,3].
(3)(1) 6.82.2
We have = = 2.3.
31 2

By the Mean Value Theorem, there is at least one time , 1 < < 3, for
which () = 2.3.
Note: See problem 46 for more information on the Mean Value Theorem.

2
105. Consider the differential equation = . On the axes

provided above, sketch a slope field for the differential equation
at the twelve points indicated.

120
Solution:

02
Notes: (1) When = 0, we have = = 0 (as long as 0). So

at each of the four points on the -axis, the slope is 0. That is, the the
tangent line is horizontal. So we draw a horizontal line segment at the
points (0,1), (0,2), (0, 1), and (0, 2).
1
(2) At the points (1,1) and (1,1), we have = = 1.
1
1
(3) At the points (1, 1) and (1, 1), we have = = 1.
1
1
(4) At the points (1,2) and (1,2), we have = .
2
1 1
(5) At the points (1, 2) and (1, 2), we have = = .
2 2

121
106. A bus is traveling on a straight road. For 0 15 seconds, the
buss velocity (), in feet per second, is modeled by the function
shown in the graph above. For each of (3), (8), and (11),
find the value or explain why it does not exist. Indicate units of
measure.
()(3) 35 ()(3)
Solution: lim ( )= and lim+ ( ) = 0.
3 3 3 3 3

()(3) ()(3)
Since lim ( ) lim+ ( ), (3) does not exist.
3 3 3 3

1035
(8) = = ft/sec2.
117
()(11) 25 ()(11) 10 5
lim ( )= and lim+ ( )= = .
11 11 4 11 11 4 2

()(3) ()(3)
Since lim ( ) lim+ ( ), (11) does not exist.
11 3 11 3

Notes: (1) At = 3 and = 11 (as well as = 7) there are sharp edges.


A function of is not differentiable at any such -value. The reason is that
these sharp edges indicate a disagreement in the slope of the graph
from the left and from the right.
For example, to the left of 3, the slope of the line is positive (in fact it is
350 35
= ), whereas, to the right of 3, the slope of the line is 0.
30 3
35
Symbolically, we can write (3) = and (3)+ = 0. Since the slopes
3
from the left and right disagree, there is no well-defined slope at = 3.

122
()(3)
Note that (3) is just a shorthand notation for lim ( ), and
3 3
()(3)
similarly (3)+ is shorthand for lim+ ( ).
3 3

(2) To compute (3) , we simply compute the slope of the leftmost line
segment. This line segment passes through the points (0,0) and (3,35).
350 35
So the slope is (3) = = .
30 3

Similarly, to compute (8), we simply compute the slope of the third line
segment from the left. This line segment passes through the points (7,35)
1035 25
and (11,10). So the slope is (8) = = .
117 4

We can compute all the other slopes in a similar fashion.


107. Two particles are moving along the -axis. For 0 10, the
position of particle at time is given by () = 3sin , and the
position of particle at time by () = 3 12 2 + 21 1.
For 0 10, find all times during which the two particles
travel in opposite directions.
Solution: () = 3 cos and
() = 3 2 24 + 21 = 3( 2 8 + 7) = 3( 1)( 7).
3 5
Now, () = 0 when = , , and on the interval [0,10], and
2 2 2
() = 0 when =1, and 7.
3 5
() > 0 for 0 < , and <<
2 2 2
3 5
() < 0 for << , and < 10
2 2 2

() > 0 for 0 < 1, and 7 < 10


() < 0 for 1 < < 7
Therefore, the particles are travelling in opposite directions for
3 5
1 < < , < < 7, and < 10.
2 2 2

In interval notation, the two particles are travelling in opposite directions


3 5
for in (1, ) ( , 7) ( , 10].
2 2 2

123
Notes: (1) We can determine which direction a particle is moving by
looking at its velocity function. If the velocity is positive, the particle is
moving to the right. If the velocity is negative, the particle is moving to
the left.
(2) The velocity function of a particle is the derivative of its position
function. So in this problem we want to determine when and have
opposite signs.
(3) We start by finding the critical numbers of and . In other words we
want to find all real numbers such that () = 0 and () = 0.
Finding the critical numbers of is straightforward.
To find the critical numbers of , recall that cos = 0 whenever is an
3 5
odd multiple of . In other words, = , , ,
2 2 2 2
3 5
It follows that the critical numbers of on [0,10] are = , , and
2 2 2
(use your calculator if allowed, or estimate as a little more than 3 to see
this).
See problem 65 for the graph of = cos .
(4) Here is a sign chart for :

And here is a sign chart for :

Lets combine these two sign charts into a single chart:

124
From this last chart it is easy to see where and have opposite signs.
0 0<<1 1 1<<2 2 2<<3 3
() 4 3 1 2
() 3 + 0 + 0 5

108. The differentiable function is defined for all real numbers .


Values of and for various values of are given in the table
above. Find the -coordinate of each relative maximum of on
the interval [0,3]. Justify your answers.
Solution: The critical numbers of are = 1 and = 2. But = 2 is the
only critical number at which changes sign from positive to negative.
Therefore = 2 is the only -coordinate where has a relative
maximum.
Notes: (1) See problem 48 for more detailed information on the first
derivative test and how to interpret the chart above.
In particular observe that () > 0 for 1 < < 2. It follows that is
increasing for 1 < < 2.
Similarly, () < 0 for 2 < < 3 implies that is decreasing for 2 <
< 3.
So has a relative maximum at = 2.
At the critical number = 1 there is no sign change ( () > 0 both to
the left and right of = 1). It follows that is increasing before and after
= 1. So there is no relative minimum or maximum at = 1.

125
109. Consider a differentiable function with domain all positive real
2
numbers, satisfying () = 3 for > 0. Find the

-coordinates of all relative minima and maxima, find all
intervals on which the graph of is concave up, and find all
intervals on which the graph of is concave down. Justify your
answers.
Solution: () = 0 at = 2, () > 0 for 0 < < 2, and () < 0
for > 2. It follows that has a relative maximum at = .
3 (1)(2)3 2 2 [3(2)] 2(3)
() = = = .
6 6 4

() = 0 at = 3.
() < 0 for 0 < < 3, and () > 0 for > 3.
So the graph of graph of is concave down for < < and the
graph of is concave up for > .
Notes: (1) = 2 is a critical number of since (2) = 0.
(2) See problem 48 for more detailed information on the first derivative
test and how to interpret the information about .
(3) At = 3 the graph of has a point of inflection because changes
concavity there (from concave down to concave up).
(4) See problem 70 for more detailed information on how to interpret
the information about .


0 1 2 3 4
(minutes)
()
0 2.2 4.5 6.8 8.3
(pints)
110. Gasoline is dripping out of a gas pump, filling up a bucket. The
amount of gasoline in the bucket at time , 0 4, is given by
a differentiable function , where is measured in minutes.
Selected values of (), measured in pints, are given in the table
above. Use the data in the table to approximate (1.5). Show the
computations that lead to your answer, and indicate units of
measure.

126
(2)(1) 4.52.2
Solution: (1.5) = = . pints/minute.
21 1

111. A bus is traveling on a straight road. For 0 15 seconds, the


buss velocity (), in feet per second, is modeled by the function
defined by the graph above. Let () be the buss acceleration at
time , in feet per second per second. For 0 < < 15, write a
piecewise-defined function for ().
35
if 0 < < 3
3
0 if 3 < < 7
Solution: () = 25
if 7 < < 11
4
5
{ 2 if 11 < < 15

() does not exist at = 3, = 7, and = 11.


Notes: (1) The acceleration function () is the derivative of the velocity
function (). That is, () = ().
(2) See the notes after the the solution to problem 106 for more detailed
information.
(3) If 0 < < 3, then () is simply the slope of the leftmost line segment.
This line segment passes through the points (0,0) and (3,35). So the slope
350 35
is () = = .
30 3

(4) If 3 < < 7, then () is the slope of the second line segment from
the left. This line segment is horizontal and therefore has a slope of
() = 0.

127
(5) If 7 < < 11, then () is the slope of the third line segment from the
left. This line segment passes through the points (7,35) and (11,10). So
1035 25
the slope is () = = .
117 4

(6) If 11 < < 15, then () is the slope of the rightmost line segment.
This line segment passes through the points (11,10) and (15,0). So the
010 10 5
slope is () = = = .
1511 4 2

0 0<<1 1 1<<2 2 2<<3 3


() 5 + 2 + 3 + 0
() 2 0 + 0 8

112. The twice-differentiable function is defined for all real


numbers . Values of and for various values of are given
in the table above. Explain why there must be a value , for 0 <
< 3, such that () = 2.
Solution: Since is differentiable, it follows that is continuous on the
interval [0,3].
(3) (0) 8(2) 6
We have = = = 2.
30 30 3

By the Mean Value Theorem, there is at least one real number with
0 < < 3 such that () = 2.
Note: See problem 46 for more information on the Mean Value Theorem.

128
LEVEL 4: INTEGRATION
ln()
113. The region in the -plane bounded by the graph of = ,

= 1 , = 4, and the -axis is rotated about the -axis. What is
the volume of the solid generated?
(ln 4)3
(A) 1
3
(ln 4)3
(B) 1
3
(ln 4)3 1
(C)
3
(ln 4)3
(D)
3

Solution:
4 ln() 2 4 (ln )2 (ln )3 4
1 ( ) = 1 = |1 = ((ln 4)3 (ln 1)3 )
3 3

(ln 4)3
= ((ln 4)3 (0)3 ) = (ln 4)3 = , choice (D).
3 3 3

Notes: (1) It is helpful to begin by drawing a picture of the region. We do


not need an accurate sketch of the function. We need only show where it
ln(1)
lies above and below the -axis. In this case, we have = 0, and for
1
ln()
all > 1, we have > 0.

(2) We will use the disk method to find the requested volume. A disk is a
circle together with its interior. A typical disk can be described as follows:

129
First we take a value between 1 and 4. Find this number on the -axis.
We then draw the radius of the disk by drawing a vertical segment from
the -axis straight up until we hit the curve.
Next we draw a circle from the top of this line segment that sweeps below
the -axis, and whos radius is as specified in the last step.
Here is a picture:

ln()
The radius of this disk is . It follows that the area of this disk is

ln() 2
2 = ( ) .

The disk method requires us to integrate this area over the given interval,
in this case from 1 to 4.
4 4 4
ln() (ln )2
= 2 = ( )2 =
1 1 1
(ln )2
(3) To evaluate , we can formally make the substitution

= ln . It then follows that
1
=

We now have
(ln )2 1 3 (ln )3
= (ln )2 = 2 = + = +
3 3

(ln )2 (ln )3
It follows that = + .
3

130
(4) If we are doing the substitution formally, we can save some time by
changing the limits of integration. We do this as follows:
4 (ln )2 ln 4 3 ln 4 (ln 4)3
1 = 0 2 = |0 = ((ln 4)3 03 ) = .
3 3 3

Notice that the limits 1 and 4 were changed to the limits 0 and ln 4,
respectively. We made this change using the formula that we chose for
the substitution: = ln . When = 1, we have = 0 and when = 4,
we have = ln 4.
114. * Water is leaking from an air conditioning unit at the rate of

() = 750 3 quarts per hour, where is measured in hours.
Approximately how much water has leaked out of the unit after 5
hours?
(A) 47
(B) 142
(C) 608
(D) 1825
Solution:
5 5
5
0 750 3 = 750(3) 3 |0 = 2250 ( 3 0 ) 1825.

This is choice (D).


Notes: (1) The word rate indicates a derivative.
(2) gives the rate at which the amount of water in the air conditioning
unit is decreasing. So we can find the amount of water in the tank by

integrating () = () to get () = 2250 3 + .
Note that we do not have enough information to figure out what is. We
therefore cannot determine the amount of water in the unit at a specific
time .
(3) Although we do not have enough information to figure out the amount
of water in the unit at a specific time, we can figure out how much water
has leaked out over any specified interval of time.
The amount of water that has leaked out from time = to time =
is the difference between the amount in the unit at time and time . So

we would simply compute () .

131
In this problem we want to find the amount of water that has leaked out
5
after 5 hours, ie. from = 0 to = 5. So we compute 0 () .

(4) To evaluate 750 3 , we can formally make the substitution
1 1
= = . It then follows that =
3 3 3

We now have

1
750 3 = 750(3) 3 ( 3) = 2250

= 2250 + = 2250 3 +
(5) If we are doing the substitution formally, we can save some time by
changing the limits of integration. We do this as follows:
5 5 5
5
0 750 3 = 2250 0 3 = 2250 |0 3 = 2250( 3 0 )

1825.
5
Notice that the limits 0 and 5 were changed to the limits 0 and ,
3
respectively. We made this change using the formula that we chose for
1
the substitution: = . When = 0, we have = 0 and when = 5,
3
5
we have = .
3

115. * The average value of (ln )5 on the interval [2,5] is


(A) 0.584
(B) 1.299
(C) 3.896
(D) 11.687
1 5
Solution: (ln )5
52 2
3.896, choice (C).

Notes: (1) The average value of the function over the interval [, ] is
1
() .

5
(2) We can compute 2 (ln )5 directly in our TI-84 calculator as
follows:

132
Press MATH, followed by 9 (or scroll up 2 times and select 9:fnInt( ). Type
((ln(X))^5, X, 2, 5) followed by ENTER. The display will show 11.6865333.
1 1
(2) Remember to multiply 11.6865333 by = to get
52 3
1 5
(ln )5
3 2
3.896.
ln(+1)
116. * If () = cos( ) and (0) = 2, then (3) =
4

(A) 4.919
(B) 2.919
(C) 0.75
(D) 1.5
3 3 ln(+1)
Solution: (3) = 2 + 0 () = 2 + 0 cos( ) 4.919.
4

This is choice (A).


Notes: (1) Given a continuous function with an antiderivative of ,
and initial condition () = , it follows that

() = + () .

To see why this is true, simply observe that () = () (),

so that + () = () + (() ()) = ().

The formula given above is especially useful when you cannot easily find
a closed form for . In other words, this form is very useful when solving
this type of problem with a calculator.
(2) In this problem, we have = , = , = 0, = 3, and = 2, so
that
3
(3) = 2 + 0 () .

117. * Let be the region enclosed by = sin and = cos for



0 . The volume of the solid generated when is revolved
4
around the line = 4 is
(A) 9.715
(B) 4.857
(C) 1.793
(D) 0.897

133

Solution: 2 04 (4 )(cos sin ) 9.715, choice (A).

Notes: (1) It is helpful to begin by drawing a picture of the region and


the line = 4.

(2) We will use the shell method to find this volume. A shell is cylinder. A
typical shell can be described as follows:

First we take a value between 0 and . Find this number on the -axis.
4

We then draw the height of the shell directly above this number between
the two graphs.
Next we draw circles from the top and bottom of this line segment (the
height) that sweep across the line = 4, and whos radius is the distance
from the height to the line = 4.
Here is a picture:

134
The height of this cylinder is () () = cos sin , and the radius
of this cylinder is 4 . It follows that the volume of this cylinder is
2 = 2(4 )(cos sin ).
The shell method requires us to integrate this volume over the given

interval, in this case from 0 to .
4

= 2 0 = 2 04 (4 )(cos sin ) .
4


118. * Let the function be defined by () = 0 cos(4 ) on the
closed interval [0,1.5]. has a local maximum at =
(A) 0
(B) 1.120
(C) 1.571
(D) 1.473
Solution: () = cos( 4 ). Now we use our graphing calculator. First
make sure that your calculator is in radian mode (Press MODE and select
Radian). Next press Y= and next to Y1 enter cos (X^4). Then press
WINDOW and set Xmin to 0 and Xmax to 1.5. Press GRAPH to display the
graph (if you do not see the graph well, you may have to adjust Ymin and
Ymax as well).
Press CALC (2nd TRACE), select zero (or press 2). Move the cursor to the
left of the zero and press ENTER. Then move the cursor to the right of the
zero and press ENTER. Press ENTER one more time, and you will get an
answer of = 1.120 (rounded). This is choice (B).
Notes: (1) We need to find the critical numbers of in the interval [0,1.5].
To do this we set () = 0.
(2) To compute () we use the first Fundamental Theorem of Calculus

which says that if () = () , for some constant , then
() = ().
So in this problem we must solve the equation cos( 4 ) = 0.
(3) Note that the graph of is above the -axis to the left of the critical
number, and below the -axis to the right of the critical number. So
() > 0 for < 1.120 and () < 0 for > 1.120.

135
It follows that is increasing for 0 < < 1.120 and decreasing for
1.120 < < 1.473. So has a local maximum at = 1.120.
(4) = 1.473 is also a critical number of . But by an argument similar to
that used in note (3), has a local minimum at = 1.473.

1 3 1 3 2 3 3 3 100
119. The expression ( + + ++ ) is a
100 100 100 100 100
Riemann sum approximation for
1 100 3
(A)
100 0

1 13
(B)
100 0
1 13
(C) 100
100 0
13
(D) 0
3
Solution: Let () = and partition the interval [0,1] into 100 equal
subintervals as follows:
0 1 2 99 100
Let 0 = = 0 , 1 = , 2 = ,, 99 = , 100 = = 1.
100 100 100 100 100
1 1 2 99
We get the corresponding subintervals [0, ],[ , ] ,, [ , 1].
100 100 100 100

The length of each subinterval (and therefore the base of each rectangle)
10 1
is = .
100 100

For the height of each rectangle we choose the right endpoint of each
subinterval , and compute ( ).

1 3 1
So the height of the leftmost rectangle will be ( )= . It follows
100 100
1 3 1
that the area of the leftmost rectangle is .
100 100

1 3 2
Similarly, the area of the next rectangle will be .
100 100

Continuing in this fashion, we get that the area of the rightmost rectangle
1 3 100
is .
100 100

136
3
So we are approximating the area under the graph of () = from
= 0 to = 1 by

1 3 1 1 3 2 1 3 100
+ ++
100 100 100 100 100 100

1 3 1 3 2 3 100
= ( + ++ ).
100 100 100 100

This is precisely the expression given in the problem. So the answer is


choice (D).
Notes: (1) A Riemann Sum Approximation of over the interval
= [, ] with partition = {[0 , 1 ] , [1 , 2 ] , , [1 , ]} where
= 0 < 1 < 2 < < = is given by
= =1 ( )( 1 ) , where 1
or equivalently
= (1 )(1 0 ) + (2 )(2 1 ) + + ( )( 1 ) .
where 0 1 1 , 1 2 2 ,, 1 .
(2) Geometrically (and in English), what note (1) says is that we first chop
up the interval [, ] into pieces, called subintervals.
For each subinterval we then choose an -value in that subinterval, and
draw a rectangle whose base is the length of the subinterval, and whose
height is the corresponding -value on the graph (more precisely, if we
choose in the subinterval, then the height is ( )).
Finally we add up the areas of all of these rectangles.
(3) Very often (such as in this problem) we want all of the subintervals in
our partition to have equal length. In this case, the length of each

subinterval is (remember that and are the left and right endpoints

of the original interval, and is the number of pieces we are chopping it
up into). The formula for the Riemann Sum Approximation then simplifies
to

= =1 ( )

or equivalently

137

= [(1 ) + (2 ) + + ( )].

(4) Here is a sketch of the Riemann Sum approximation for this problem:

Note that only part of the picture is drawn here. In reality there should be
1
100 rectangles, each with a base of .
100

Also note that for each subinterval we chose the right endpoint of the
interval to form the height of the rectangle. For example, the height of
1 3 1
the leftmost rectangle is ( )= . It follows that the area of the
100 100
1 3 1
leftmost rectangle is .
100 100

1 3 2
Similarly, the area of the second rectangle is .
100 100

Continuing in this fashion, we get that the area of the rightmost rectangle
1 3 100
is .
100 100

It follows that the sum of all of these rectangles is

1 3 1 1 3 2 1 3 100
+ ++
100 100 100 100 100 100

1 3 1 3 2 3 100
= ( + ++ ).
100 100 100 100

138
Observe that this computation led to an approximation of the area under
3
the graph of () = from = 0 to = 1. In other words, we
13
approximated 0 .


0 1 2 3 4
(minutes)
()
0 2.2 4.5 6.8 8.3
(pints)
120. Gasoline is dripping out of a gas pump, filling up a bucket. The
amount of gasoline in the bucket at time , 0 4, is given by
a continuous function , where is measured in minutes. Selected
values of (), measured in pints, are given in the table above.
Use a midpoint sum with two subintervals of equal length
1 4
indicated by the given data to approximate 0 () . Using
4
1 4
correct units, explain the meaning of 0 () in the context
4
of the problem.
Solution:
1 4 1 1
() [2(1) + 2(3)] = (2 2.2 + 2 6.8) = . pints.
4 0 4 4
1 4
()
4 0
is the average amount of gasoline in the bucket, in pints, over
the time interval 0 4 minutes.
Notes: (1) Here is a picture of the midpoint sum in this problem:

Note that both rectangles have a width of 2.

139
The leftmost rectangle has a length (or height) of (1) = 2.2. It follows
that the area of the leftmost rectangle is (2)(2.2) = 4.4
The rightmost rectangle has a length (or height) of (3) = 6.8. It follows
that the area of the rightmost rectangle is (2)(6.8) = 13.6
Finally, we get the area of the shaded region by adding these two areas:
4
0 () 4.4 + 13.6 = 18.
1 4 18
(2) Remember we actually want 0 () = 4.5.
4 4

(3) Recall that the average value of a function over the interval [, ] is
1
() .

So the average value of over the interval [0,4] is


1 4 1 4
()
40 0
= 0 () .
4

121. Consider the differential equation = 1 (2 3 5). Let

= () be the particular solution to the differential equation
that passes through (1,1). Find = ().
Solution: We separate variables to get 1 = (2 3 5). Now we
4
integrate both sides of this equation to get 1 = 5 + .
2
1
Now we substitute in = 1 and = 1 to get 0 = 5 + . So
2
1 7 4 7
= 1 + 5 = , and we have 1 = 5 + . We multiply
2 2 2 2
1 4 7
each side of this equation by 1 to get = + 5 , and then we
2 2
take the natural logarithm of each side of this last equation to get
4 7
1 = ln( + 5 ), We subtract 1 to get
2 2

4 7
= ln ( + 5 ) 1
2 2

Finally, we multiply by 1 to get = ( + ).

Notes: See problem 52 for more information on separable differential


equations.

140
0 1 2 3
() 2 7 1 1
() 3 0 0 5
() 2 5 2 4
() 8 9 6 0

122. The differentiable functions and are defined for all real
numbers . Values of , , , and for various values of are
3
given in the table above. Evaluate 0 (())() .
3 =3
Solution: 0 (())() = [(())]=0 = ((3)) ((0))

= (1) (2) = 5 (2) = 5 + 2 = .

Notes: (1) By the chain rule, the derivative of (()) is (()) ().
It follows that (()) () = (()) + .

(2) To evaluate (()) () , we can formally make the


substitution = (). It then follows that = ().
We now have

(()) () = () = () + = (()) + .
(3) If we are doing the substitution formally, we can save some time by
changing the limits of integration. We do this as follows:
3 (3) 1 1
0 (())() = (0) () = 2 () = () |2

= (1) (2) = 5 (2) = 5 + 2 = 3.


Notice that the limits 0 and 3 were changed to the limits (0) and (3),
respectively. We made this change using the formula that we chose for
the substitution: = (). When = 0, we have = (0) and when
= 3, we have = (3).
(4) Using the table we see that (0) = 2 and (3) = 1. This is how we
got the third equality in the main solution above, and also the second
equality in note (3) above.

141
(5) Using the table we see that (1) = 5 and (2) = 2. This is how
we got the fourth equality in the main solution above, and also the fifth
equality in note (3) above.

123. A bus is traveling on a straight road. For 0 15 seconds, the


buss velocity (), in feet per second, is modeled by the function
15
defined by the graph above. Find 0 () . Using correct units,
15
explain the meaning of 0 () .

Solution:
15
1 35 + 10 1
() = (3)(35) + (4)(35) + ( ) (4) + (4)(10)
0 2 2 2
= . .
The bus travels 302.5 feet in these 15 seconds.
15
Notes: (1) Geometrically we can interpret 0 () as the area under
the graph of () from = 0 to = 15. We split this area into four pieces
as follows:

142
1 1
(2) The area of the leftmost triangle is = (3)(35).
2 2

(3) The area of the rectangle is = (7 3)(35) = (4)(35).


(4) The area of the trapezoid is
(1 +2 ) 35+10 35+10
=( ) (11 7) = ( ) (4).
2 2 2

Note that for the trapezoid, the height lies along the -axis.
1 1 1
(5) The area of the last triangle is = (15 11)(10) = (4)(10).
2 2 2

124. The function is defined on the closed interval [5,10]. The


graph of consists of four line segments and is shown in the

figure above. Let be defined by () = 0 () . Compute
(5)+(4)
.
(10)(7)

143
5 0 3+1
Solution: (5) = 0 () = 5 () = ( ) 5 = 10.
2
4
(4) = 0 () = (4)(1) = 4.
10 9 10
(10) (7) = 7 () = 7 () + 9 ()
1 1 3 9
= (2)(6) (1)(3) = 6 = .
2 2 2 2
(5)+(4) 9 2 12
So = (10 + 4) = 6 = = .
(10)(7) 2 9 9


Notes: (1) () = () .

We used this theorem in the first computation to get


5 0
0 () = 5 () .

(2) If < < , then () = () + () .

We used this theorem in the third computation to get


10 9 10
7 () = 7 () + 9 () .

(3) Geometrically, if the graph of lies above the -axis between and ,

then () is the area under the graph of between = and
= .

If the graph of lies below the -axis between and , then ()
is the negative of the area above the graph of between and .

144
0
So 5 () is the leftmost area of the trapezoid with bases of length
3 and 1, and height 5 (note that the height lies along the -axis). So we
0 1 +2 3+1 4
have 5 () = ( ) = ( ) 5 = 5 = 2 5 = 10.
2 2 2
4
0 () is the area of the rectangle with base 4 and height 1. So we
4
have 0 () = = (4)(1) = 4.
9
7 () is the area of the first shaded triangle with base 2 and height
9 1 1
6. So we have 7 () = = (2)(6) = 6.
2 2
10
9 () is the negative of the area of the rightmost triangle with base
10 1 1 3
1 and height 3. So we have 9 () = = (1)(3) = .
2 2 2

125. * A 5000 gallon tank is filled to capacity with water. At time


= 0, water begins to leak out of the tank at a rate modeled by
(), measured in gallons per hour, where
300
, 04
() = { + 1
500 0.5 , > 4
Find the average rate at which water is leaking from the tank
between time = 0 and time = 10 hours. Then write, but do
not solve, an equation involving an integral to find the time
when the amount of water in the tank is 1000 gallons.
Solution: The average rate at which water is leaking from the tank
between time = 0 and time = 10 hours is
1 10 1 4 300 10
() = (0 + 4 500 0.5 ) . .
100 0 10 +1

To find the time when the amount of water in the tank is 1000 gallons,

we would solve the equation () = .

Notes: (1) Recall that the average value of a function over the interval
1
[, ] is () .

1 10
So the average value of over the interval [0,10] is () .
100 0

145
(2) The use of the word rate in this problem can be confusing. In general
we associate the word rate with a derivative (as we should). In this
problem, note that the function is a rate function. So when we are asked
to find the average rate at which water is leaking from the tank, we are
being asked to find the average value of .

(3) If < < , then () = () + () .

We used this theorem to write


10 4 10
0 () = 0 () + 4 () .
4 300
(4) We can compute 0 directly in our TI-84 calculator as follows:
+1

Press MATH, followed by 9 (or scroll up 2 times and select 9:fnInt( ). Type
(300X/(X + 1), X, 0, 4) followed by ENTER. The display will show
717.1686263.
10
Similarly, we can compute 4 500 0.5 in our TI-84 calculator:

Press MATH, followed by 9 (or scroll up 2 times and select 9:fnInt( ). Type
(500e^(-.5X), X, 4, 10) followed by ENTER. The display will show
128.5973362.
(5) Since () gives the rate at which water is leaking out of the tank, it

follows that 0 () gives the total amount of water that has leaked
out of the tank at time .
Since we started with 5000 gallons of water in the tank, it follows that

5000 0 () is the amount of water in the tank at time .

Since we want to find the time when the amount of water in the tank is

1000 gallons, we set 5000 0 () equal to 1000.

146
126. Joe is driving his car along a straight road. For 0 15
seconds, his velocity (), in miles per minute, is modeled by the
function defined by the graph above. Using correct units, explain
15
the meaning of 0 |()| in terms of Joes trip. Then find the
15
value of 0 |()| .
15
Solution: 0 |()| is the total distance, in miles, that Joe drove during
the 15 minutes from = 0 to = 15.

15 4 8 15
0 |()| = 0 () 4 () + 8 ()

= 0.6 + 0.6 + 4(0.7) = miles.


15
Notes: (1) 0 |()| is equal to the following shaded area:

147
1 1
The leftmost triangle has area = (4)(0.3) = 0.6.
2 2

The second triangle also has an area of 0.6.


1 1
The third triangle has area = (2)(0.7) = 0.7.
2 2

The rectangle has area = (1)(0.7) = 0.7.


1 1
The rightmost triangle has area = (4)(0.7) = 1.4.
2 2

So the total area is 0.6 + 0.6 + 0.7 + 0.7 + 1.4 = 4.

1 5 6 8 11
() 3 5 1 3 4

127. Let be a function that is continuous for all real numbers. The
table above gives values of for selected points in the closed
interval [1,11]. Use a left Riemann Sum with subintervals
11
indicated by the data in the table to approximate 1 () .
Show the work that leads to your answer.
11
Solution: 1 () (3)(4) + (5)(1) + (1)(2) + (3)(3) = .

Notes: (1) See problem 119 for general information about Riemann Sum
Approximations.
(2) In this problem we are partitioning the interval [1,11] into = 4
subintervals: = {[1,5] , [5,6] , [6,8] , [8,11]}.

148
We have = 0 = 1, 1 = 5, 2 = 6, 3 = 8, 4 = = 11.
(3) We are choosing to be the left endpoint.
So 1 = 1, 2 = 5, 3 = 6, and 4 = 8.

(4) We have = 4=1 ( )( 1 )


= (1 )(1 0 ) + (2 )(2 1 ) + (3 )(3 2 ) + (4 )(4 3 )

= (1)(5 1) + (5)(6 5) + (6)(8 6) + (8)(11 8)


= (3)(4) + (5)(1) + (1)(2) + (3)(3) = 12 + 5 + 2 9 = 10.
(5) Geometrically (and in English), we are chopping up the interval [1,11]
into = 4 subintervals. Note that in this case the subintervals are not
equal.
For each subinterval we then choose the left endpoint of that interval,
and draw a rectangle whose base is the length of the subinterval, and
whose height is the corresponding -value on the graph (more precisely,
the height is ( ), where is the left endpoint of the interval).
Finally we add up the areas of all of these rectangles.
(6) Here is a sketch of the Riemann Sum approximation for this problem:

Note that for each subinterval we chose the left endpoint of the interval
to form the height of the rectangle. For example, the height of the
leftmost rectangle is (1) = 3 . It follows that the area of the leftmost
rectangle is (3)(5 1) = (3)(4) = 12.

149
Similarly, the area of the second rectangle is (5)(1) = 5, the area of the
third rectangle is (1)(8 6) = (1)(2) = 2, and the area of the rightmost
rectangle is (3)(11 8) = (3)(3) = 9.
Note that when we compute the Riemann Sum, we subtract the area of
the rightmost rectangle since it lies below the -axis.
The Riemann Sum is then 12 + 5 + 2 9 = 10.
128. * Let be the region in the first and second quadrants bounded
48
above by the graph of = and below by the horizontal line
2+ 4
= 4. The region is the base of a solid whose cross sections
perpendicular to the -axis are semicircles. Find the volume of
this solid.
48
Solution: We first find the -values where the graph of =
2+ 4
48
intersects the line = 4. To do this we solve the equation = 4.
2+ 4
Multiplying by the denominator on the left hand side gives the equation
48 = 8 + 4 4 . We subtract 8 to get 40 = 4 4 , divide by 4 to get
4
10 = 4 , and finally take the fourth root of each side to get = 10.
So the volume we are looking for is

4
10 1 48 2
4 [ (
2 10 2 2+ 4
4)] . .

Notes: (1) It is helpful to begin by drawing a picture of the region

(2) Lets draw a cross section that is perpendicular to the -axis.

150
4 4
First we take a value between 10 and 10 and find this number on
the -axis.
We then draw the diameter of the semicircle. This diameter has a length
48
of () () = 4 4. It follows that the length of a radius of this
2+
1 48
semicircle is = ( 4). So the area of the semicircle is
2 2+ 4

1 48 2
= 2 = [ ( 4 4)] .
2 2 2 2+

To get the desired volume, we now simply integrate this expression over
4 4
the interval [ 10, 10]:

4
10 1 48 2
= 410 [ ( 4 4)] .
2 2 2+
4
10 1 48 2
(3) We can compute 410 [ ( 4 4)] directly in our TI-84
2 2+
calculator as follows:
Press MATH, followed by 9 (or scroll up 2 times and select 9:fnInt( ). Type

((1 / 2(48 / (2 + X^4) 4))^2, X, 4 10, 4 10) followed by ENTER. The
display will show 180.9284648. Multiply this result by / 2, and the
display will show 284.201768.
4
(4) We can compute 10 in our TI-84 calculator by typing 4, then selecting

from the MATH menu, then typing 10 followed by ENTER. We needed


to do this for the calculator computation in note (3).

151
LEVEL 5: FREE RESPONSE QUESTIONS

+1
129 132 Consider the differential equation = and the figure

above.

129. On the axes provided, sketch a slope field for the differential
equation at the twelve points indicated, and for > 1, sketch the
solution curve passing through the point (1,0). Then describe all

points in the -plane, 0, for which = 1.

Solution:

+1
= 1 = 1 + 1 = = .

Notes: (1) The figure above on the left gives the slope field for the
differential equation at the twelve indicated points. For more
information on how to draw this slope field see problem 105.
(2) The figure on the right shows the solution curve that passes
through the point (1,0). Observe that at any point of the form
(1)+1
(, 1), the slope is = = = 1. So the solution

curve is a line with slope 1.

152
130. Write an equation for the line tangent to the solution curve at the
point (2,0). Use the equation to approximate (1.5) where
= () is the particular solution of the differential equation
with initial condition (2) = 0.
Solution: The slope of the tangent line at (2,0) is
0+1 1
= |(2,0) = = .
2 2

So an equation of the tangent line in pointslope form is


1
0 = ( (2)), or equivalently
2
1
= 1
2
1 1
When = 1.5, we have = (1.5) 1 = .
2 4

It follows that (1.5) or . .

Notes: (1) To find the slope of the tangent line to a function = () at



a point (0 , 0 ), we take the derivative = (), and substitute in 0

for and 0 for .

In this problem, we were already given , and so we simply need to

substitute in the -coordinate of the point for and the -coordinate of
the point for .

(2) The point-slope form of an equation of a line is

= ( )

where is the slope of the line and (, ) is any point on the line.

It is generally easiest to write an equation of a line in point-slope form


once the slope of the line and a point on the line are known.
1
In this problem, the slope is and the point is (2,0).
2

131. Find = (), the particular solution to the differential equation


with the initial condition (2) = 0.

153

Solution: We separate variables to get = . Now we integrate both
+1
sides of this equation to get ln| + 1| = ln|| + . So we have that
+1
ln| + 1| ln|| = , and so ln | | = . Changing to exponential form

+1
yields = . We can simply rename the constant as .

+1
So we have = .

1 +1
Now we substitute in = 2 and = 0 to get = . So we have =
2
1 1 1
, and so + 1 = , and finally, = 1.
2 2 2

So () = .

Notes: (1) See problem 52 for more information on separable differential


equations.
(2) Observe that in this problem, the solution curve is a line, and therefore
it is equal to its tangent line (so the approximation found in problem 130
is actually the exact value).
132. Describe the region in the -plane in which all solution curves
to the differential equation are concave down.
+1
2 (+1)(1) ( )(+1) (+1)(+1) 0

Solution: = = = = = 0.
2 2 2 2 2

2
Since 2 is always 0, it follows that there are no solution curves which are

concave down. So the region is empty.
Notes: (1) The graph of a function = () is concave down at -values
2
such that = () < 0. Therefore, we are looking for solutions to
2
2
the differential equation for which < 0.
2
+1
(2) We get the third expression above by replacing with .

2
(3) Since 2 = 0, all solutions to this differential equation are in fact

linear. Only linear functions have second derivatives that are 0
everywhere.

154
2
To see this, we integrate each side of 2 = 0 to get = where is

some constant. We integrate again to get = + , where is
another constant.
133 136 * Suppose that the average annual salary of an NBA player
is modeled by the function () = 161.4(1.169 ), where () is
measured in thousands of dollars and is measured in years since 1980
(for example, since (0) = 161.4, the average salary of an NBA
player in 1980 was $161,400).
133. * Find the average rate of change of () over the interval
0 20. Interpret this answer and indicate units of measure.
Solution: The average rate of change of () over 0 20 is
(20)(0)
= . (or . )
200

So the average rate of change in the annual salary of an NBA player from
1980 through 2000 was $175,230 (or $175,231) per year
Notes: (1) The average rate of change of the function on the interval
()()
is .

(2) (20) = 161.4(1.16920 ) and (0) = 161.4(1.1690 ) = 161.4.


(20)(0)
(3) We perform the computation right in our calculator.
200

(4) When = 0 the year is 1980, and when = 20 the year is 2000.
(5) Since () is measured in thousands of dollars, we interpret the
answer 175.230 as $175,230.
134. * Find the value of (10). Using correct units, interpret the
meaning of the value in the context of the problem.
Solution: We use our calculator to compute
(10) = .
So the average annual salary of an NBA player is increasing at a rate of
$120,112 per year at the beginning of 1990.
Notes: (1) We can compute (10) directly in our TI-84 calculator as
follows:

155
Press MATH, followed by 8 (or scroll up 3 times and select 8:nDeriv( ).
Type 161.4(1.169^X), X, 10) followed by ENTER. The display will show
120.1121575. We truncate this number to 120.112.
(2) As an alternative, we can first compute the derivative
() = 161.4(1.169 )(ln(1.169)
and then use our calculator to get
(10) = 161.4(1.16910 )(ln(1.169)
(3) Since (10) is positive, it follows that the average annual salary of an
NBA player is increasing when = 10.
(4) = 10 at the beginning of the year 1990.
(5) Since () is measured in thousands of dollars, we interpret the
answer 120.112 as $120,112.
135. * Use a right Riemann sum with five equal subintervals to
1 20
approximate ().
20 0
Does this approximation
overestimate or underestimate the average salary from the
beginning of 1980 through the end of 2000? Explain your
reasoning.
Solution:
1 20 1
() 4[(4) + (8) + (12) + (16) + (20)]
20 0 20
. or . .
This approximation is an overestimate, because a right Riemann sum is
used and the function is strictly increasing.
Note: See problem 119 for general information about Riemann Sum
Approximations, and problem 127 for more details about this type of
problem.
136. * Find the year in which it occurs that the average annual salary
is equal to the average salary from the beginning of 1980 through
the end of 2000.
Solution: The average salary from the beginning of 1980 through the end
of 2000 is

156
1 20
()
200 0
1122.203
1 20
So we solve the equation () = 0 () and get 12.418 or
20
12.419.
It follows that the average annual salary is equal to the average salary
from the beginning of 1980 through the end of 2000 in 1992.
20
Notes: (1) We can compute 0 () directly in our TI-84 calculator as
follows:
Press MATH, followed by 9 (or scroll up 2 times and select 9:fnInt( ). Type
161.4(1.169^X), X, 0, 20) followed by ENTER. The display will show
22444.0633.
1
(2) Remember to multiply 22444.0633 by to get
20
1 20
()
20 0
= 1122.203165.
1 20
(3) We can solve the equation () = 0 () using the graphing
20
features of our TI-84 calculator as follows:
Press Y=, and next to Y1 type 161.4(1.169^X), and next to Y2 type
1122.203. Then press WINDOW, set Xmin to 0, Xmax to 20, Ymin to 0, and
Ymax to a number greater than 1122.203 (such as 1500). Next press
GRAPH. After the two graphs appear press CALC (2nd TRACE), followed by
5 (or scroll down and select 5:intersect). Press ENTER three times, and we
see that X is approximately 12.418.
(4) = 12 at the beginning of the year 1992. It follows that the average
annual salary is equal to the average salary from the beginning of 1980
through the end of 2000 in 1992.

157
137 144 Let be the continuous function defined on [5,8] whose
graph, consisting of three line segments and a semicircle centered at
(4,0), is shown above. Let be the function that is defined by

() = 2 () .

137. Find the values of (8) and (1).


8 6 8
Solution: (8) = 2 () = 2 () + 6 ()
1
= (2)2 + (2)(2) = + .
2 2
1 2 0 2
(1) = 2 () = 1 () = [1 () + 0 () ]
1+2 1 3
= [ ( ) (1) (2)(2)] = ( 2) = .
2 2 2
6
Notes: (1) 2 () is the area under the semicircle of radius 2 shown.

The area of this semicircle is 2 = (2)2 = 2.
2 2
8
(2) 6 () is the area under the rightmost triangle shown. The area of
1 1
this triangle is = (2)(2) = 2.
2 2
0
(3) 1 () is the negative of the area of the trapezoid from 1 to 0
with bases of length 1 and 2, and a height of length 1 (note that the height
+ 1+2 3
is along the -axis). This area is 1 2 = ( ) (1) = .
2 2 2
2
(4) 0 () is the negative of the area of the triangle from 0 to 2 with
1 1
bases and height both 2. This area is = (2)(2) = 2.
2 2

158
138. For each of (4), (4), (6), and (6), find the value or
explain why it does not exist.
Solution: (4) = (4) = .
(4) = (4) = .
(6) = (6) = .
(6) = (6) which does not exist because the graph of has a cusp
at = 6.
Notes: (1) To compute () we use the first Fundamental Theorem of

Calculus which says that if () = () , for some constant , then
() = ().
(2) To compute () we can simply use the given graph of . For example,
(6) = 0 because the point (6,0) is on the graph of .
(3) () is the slope of the tangent line to the graph of at .
At = 4, the graph of is a line with slope 1.So (4) = 1.
At = 6, the slope from the left is and the slope from the right is 1.
That is (6) = and (6)+ = 1. Since these two values disagree,
(6) does not exist.
(4) See problem 106 for more detailed information about the notation
() and ()+ .
139. On what open intervals contained in 5 < < 8 is the graph of
both increasing and concave up? Justify your answer.
Solution: The graph of is increasing when = > 0 (or equivalently,
the graph of is above the -axis). The graph of is concave up when
= > 0, or equivalently when is increasing.
is above the -axis and increasing on the intervals < < and
< < .
140. Find the -coordinate of each point at which the graph of has a
horizontal tangent line. For each of these points, determine
whether has a relative minimum, relative maximum, or neither
a minimum nor a maximum at the point. Justify your answer.
Solution: () = () = 0 when = , = , and = .

159
= changes sign from positive to negative at = 2. Therefore has
a relative maximum at = 2.
= changes sign from negative to positive at = 2. Therefore has
a relative minimum at = 2.
= does not change sign at = 6. Therefore has neither a relative
minimum nor a relative maximum at = 6.
141. For 5 < < 8, find all values of for which the graph of has
a point of inflection. Explain your reasoning.
Solution: The graph of has a point of inflection at = , = , and
= because = changes sign at each of these values.
Note: Recall that < 0 is equivalent to decreasing, and > 0 is
equivalent to increasing.
So changing sign is equivalent to changing from increasing to
decreasing, or from decreasing to increasing. We can check this easily by
looking at the graph of .
142. Find the absolute minimum and absolute maximum of over the
closed interval [5,8]. Explain your reasoning.
Solution: We will evaluate at each critical number of and the two
endpoints of the interval.
5 3+2 1 5 1
(5) = 2 () = [( ) 1 + 0 (2)(2)] = ( 2) =
2 2 2 2
2 2 1
(2) = 2 () = 2 () = ( ) (4)(2) = 4
2
2
(2) = 2 () = 0
6
(6) = 2 () = (2)2 = 2
2
8 1
(8) = 2 () = 2 + (2)(2) = 2 + 2
2

The absolute minimum of on [5,8] is (5) = .

The absolute maximum of on [5,8] is (8) = + .


()
143. The function is defined by () = ln . Find (4).
2

160
2 2 ()2()
Solution: () = ( )( ).
() 4 2

8 8 (4)2(4) 8 8222
So (4) = ( )( ) = ( )( ) = or 1.25.
(4) 64 2 64

4 2
Notes: (1) (4) = 2 () = 4 ()
0 2 1
= [4 () + 0 () ] = [0 (2)(2)] = 2.
2
(4)
(2) = (4) = 2 because the point (4,2) is on the graph of .
3 2 2
144. The function is defined by () = ( 1 ). Find an

equation of the tangent line to the graph of at the point where
= 1.
3(1)2 21 32
Solution: (1) = ( ) = ( ) = (1) = 1.
11 0

3 2 2 1 (62)(3 2 2) 1
() = ( )( ).
1 22
41
So (1) = (1) ( ) = 3 (1) = 3 1 = 3.
1

So the line passes through the point (1, 1) and has slope = 3. An
equation of the line in point-slope form is then
+ = ( ).

145 148 * Let be the region in the first quadrant enclosed by the
graph of () = 6 3.5 4 + 7 and the horizontal line = 5, as
shown in the figure above.

161
145. * Write an equation for the tangent line to the graph of at
= 0.5.
Solution: (0.5) = (0.5)6 3.5(0.5)4 + 7 6.797.
() = 6 5 14 3 .
So (0.5) = 6(0.5)5 14(0.5)3 1.563.
So the line passes through (0.5,6.797) and has slope = 1.563. An
equation of the line in point-slope form is then
. = . ( + . ).

146. * Find the volume of the solid generated when is rotated about
the horizontal line = 3.
1.822 2 2
Solution: = .934 [(5 (3)) ( 6 3.5 4 + 7 (3)) ]
1.822
= .934 [64 ( 6 3.5 4 + 10)2 ] . .

Notes: (1) We begin by finding the -values where intersects the line
= 5. We can do this in our calculator as follows:
Press Y=, and next to Y1 type X^6 3.5X^4 + 7 and next to Y2 type 5. Then
press WINDOW, set Xmin to 0, Xmax to 5, Ymin to 0, and Ymax to 10 (other
windows will work, of course you may need to experiment a bit to find
the right window). Next press GRAPH. After the two graphs appear press
CALC (2nd TRACE), followed by 5 (or scroll down and select
5:intersect).Press ENTER twice, then place the cursor close to the leftmost
point of intersection and press ENTER again. The display will show that
.934. Repeat the procedure again, but this time place the cursor close
to the rightmost point of intersection before pressing ENTER the third
time. This time the display will show that 1.822.
(2) We will use the washer method to find the requested volume. A
washer is a disk with a hole in it. A typical washer can be described as
follows:
First we take a value between .934 and 1.822 Find this number on the
-axis.
We then draw the outer radius of the washer by drawing a vertical
segment from the line = 3 straight up until we hit the line = 5.

162
Next we draw a circle from the top of this line segment that sweeps below
the line = 3, and whos radius is as specified in the last step.
We draw the inner radius of the washer by drawing a vertical line segment
from the line = 3 straight up until we hit the graph of .
We then draw a circle from the top of this line segment that sweeps below
the line = 3, and whos radius is as specified in the last step.
Here is a picture:

The picture on the left shows the disk formed from the outer radius. This
radius is = 5 (3) = 5 + 3 = 8. It follows that the area of this disk
is 2 = (8)2 = 64.
The picture on the right shows the formation of the hole from the inner
radius. This radius is = () (3) = () + 3. It follows that the
area of this disk is 2 = (() + 3)2
So the area of the washer is outer disk inner hole. This is
2 2 = 64 (() + 8)2 = [64 (() + 3)2 ]
The washer method requires us to integrate this area over the given
interval, in this case from .934 to 1.822.

163
1.822
= .934 [64 (() + 3)2 ]
1.822
= .934 [64 ( 6 3.5 4 + 10)2 ]
1.822
(3) We can compute .934 [64 ( 6 3.5 4 + 10)2 ] directly in our
TI-84 calculator as follows:
Press MATH, followed by 9 (or scroll up 2 times and select 9:fnInt( ). Type
64 (X^6 3.5X^4 + 10)^2, X, .934, 1.822) followed by ENTER. The display
will show 29.67169214.
Remember to multiply this result by to get approximately 93.216.
147. * The region is the base of a solid. For this solid, each cross
section perpendicular to the -axis is an equilateral triangle. Find
the volume of the solid.
3 1.822
Solution: =
4
.934 ( 6 + 3.5 4 2)2 . .

Notes: (1) To see where the limits of integration come from, see note (1)
from problem 146.
(2) Lets draw a cross section that is perpendicular to the -axis.

First we take a value between . 934 and 1.822 and find this number on
the -axis.
We then draw a side of the triangle between = () and = 5. This
side has length 5 () = 5 ( 6 3.5 4 + 7) = 6 + 3.5 4 2.

The area of an equilateral triangle with side length is = (see note

(4) below).

164

It follows that the area of this triangle is ( 6 + 3.5 4 2)2 .

To get the desired volume, we now simply integrate this expression over
the interval [.934,1.822]:
3 1.822
=
4
.934 ( 6 + 3.5 4 2)2 .
1.822
(3) We can compute .934 ( 6 + 3.5 4 2)2 directly in our TI-84
calculator as follows:
Press MATH, followed by 9 (or scroll up 2 times and select 9:fnInt( ). Type
(-X^6 + 3.5X^4 2)^2, X, .934, 1.822) followed by ENTER. The display will
show 7.89719.
3
Remember to multiply this result by to get approximately . .
4

(4) Most students do not know the formula for the area of an equilateral
triangle, so here is a quick derivation.
Lets start by drawing a picture of an equilateral triangle with side length
, and draw an altitude from a vertex to the opposite base. Note that an
altitude of an equilateral triangle is the same as the median and angle
bisector (this is in fact true for any isosceles triangle).


So we get two 30, 60, 90 right triangles with a leg of length and
2
hypotenuse of length .
We can find by recalling that the side opposite the 60 degree angle has
length 3 times the length of the side opposite the 30 degree angle. So
3
= .
2

Alternatively, we can use the Pythagorean Theorem to find :

165
2 2 3 2 3
2 = 2 ( ) = 2 = . So = .
2 4 4 2

It follows that the area of the triangle is


1 3 1 3 3 2
= ( + )( ) = ( ) = .
2 2 2 2 2 2 4

148. The vertical line = divides into two regions with equal
areas. Write, but do not solve, an equation involving integral
expressions whose solution gives the value .
Solution:
1.822
.934[5 ()] = [5 ()]
1.822
.934[5 ( 6 3.5 4 + 7 )] = [5 ( 6 3.5 4 + 7 )]
.
.( + . ) = ( + . )

149 152 Let and be twice-differentiable functions such that


(1) = 4, (8) = 6, (3) = 1, and (2) = 8. Let be the
function given by () = (()).

149. Let satisfy 4 < < 6. Explain why there must be a value
for 3 < < 2 such that () = .
Solution: Since and are twice-differentiable, they are continuous. It
follows that is continuous.

(3) = ((3)) = (1) = 4 and (2) = ((2)) = (8) = 6.

Since is a continuous function satisfying (3) < < (2), the


Intermediate Value Theorem guarantees that there is a value , with
3 < < 2, such that () = .
Notes: (1) The Intermediate Value Theorem says that if is a continuous
function on the interval [, ] and is between () and (), then there
is a real number with < < such that () = .
Note that is between () and () means either () < < () or
() < < (). It depends if () < () or () < ().
(2) In this problem the interval is [3,2], (3) = 4, and (2) = 6,
= , and = . Here is a picture:

166
Lets just clarify the order of events that take place in this picture. First,
is given between (3) = 4 and (2) = 6. Since is continuous, the
Intermediate Value theorem guarantees that the graph of must pass
through the point (, ) for some between 3 and 2. In other words,
there is a real number , with 3 < < 2, such that () = .
150. Is there a value for 3 < < 2 such that () = 2. Justify
your answer.
Solution: Since and are differentiable, so is . In particular, is
continuous on [3,2] and differentiable on (3,2). So the Mean Value
Theorem guarantees that there is a value , with 3 < < 2 such that
(2)(3) ((2))((3)) (8)(1) 6(4) 10
() = = = = = = 2.
2(3) 2+3 5 5 5

Note: See problem 46 for more information on the Mean Value Theorem.
151. Suppose that (1) = (2) and (8) = (3). Explain why
there must be a value , with 3 < < 2 such that () = 0.
Solution: () = (()) ()

(3) = ((3)) (3) = (1) (3)

(2) = ((2)) (2) = (8) (2) .

Since (1) = (2) and (8) = (3), we have (3) = (2).

167
Also, since and are twice-differentiable, so is . So is differentiable.
In particular, is continuous on [3,2] and differentiable on (3,2). So
the Mean Value Theorem guarantees that there is a value , with
(2) (3) 0
3 < < 2 such that () = = = 0.
2(3) 2+3

Note: See problem 46 for more information on the Mean Value Theorem.
152. Suppose that () = () = 0 for all . Find all points of
inflection on the graph of .
Solution 1: Since () = () = 0 for all , it follows that and are
linear functions. The composition of two linear functions is also linear.
Therefore is a linear function, and so the graph of has no points of
inflection.
Notes: (1) Since () = 0, it follows by integration that () = for
some constant . Integrating again gives () = + for another
constant . That is is a linear function.
Similarly, is a linear function, lets say () = + .
(2) Lets compose and as defined in the first note.

() = (()) = ( + ) = ( + ) + = + ( + ).

So is a linear function with slope = and -intercept (0, + ).

Solution 2: () = (()) ()

() = (()) () + () (()) ().

Since () = () = 0 for all , we have

() = (()) 0 + () 0 () = 0.
Since () = 0 for all , there are no -values at which changes sign.
So the graph of has no points of inflection.
153 160 * A particle moves along the -axis so that its velocity at
time 0 is given by () = cos( 2 ). The position of the particle at
time is () and its position at time = 0 is (0) = 2.

153. * Find the acceleration of the particle at time = 4. Is the speed


of the particle increasing or decreasing at time = 4? Justify your
answer.

168
Solution: () = () = 2 sin( 2 ). So (4) = 8 sin 16 . .
Also (4) = cos 16 .958.
Since (4) < 0 and (4) > 0, the speed of the particle is decreasing at
= 4.
Notes: (1) The acceleration function () is the derivative of the velocity
function (). That is, () = ().
(2) Since (4) < 0, the particle is moving to the left at time = 4. Since
(4) > 0, the acceleration is acting in the opposite direction of the
velocity. It follows that the particle is slowing down at time = 4.
In general, if the velocity and acceleration have the same sign, then the
particle is speeding up, and if the velocity and acceleration have opposite
signs, then the particle is slowing down.
Important: Make sure that your calculator is in radian mode before
beginning problems 153-160 (Press MODE and select Radian). Otherwise
many of the answers will be incorrect!
154. * Find all values of in the interval 0 2 for which the
1
speed of the particle is . For each such value of determine if the
2
particle is moving to the right or to the left.
1
Solution: We solve the equation |cos( 2 )| = and get the following two
2
solutions in the interval 0 2: . and . .
Now, cos(1.0232 ) .501 > 0 so that the particle is moving to the right
at = 1.023.
Also cos(1.4472 ) .499 < 0 so that the particle is moving to the left
at = 1.447.
1
Notes: (1) We can solve the equation |cos( 2 )| = in our calculator as
2
follows:
Press Y=, and next to Y1 type abs(cos(X^2)) and next to Y2 type 1/2. Note
that to get abs(, press MATH, scroll right to NUM, and press ENTER.
Next press WINDOW, set Xmin to 0, Xmax to 2, Ymin to 0, and Ymax to 1
(other windows will work, of course, but this window is not too big and
contains all of the information we need). Next press GRAPH.

169
After the two graphs appear press CALC (2nd TRACE), followed by 5 (or
scroll down and select 5:intersect). Press ENTER twice, then place the
cursor close to the leftmost point of intersection and press ENTER again.
The display will show that 1.023. Repeat the procedure again, but
this time place the cursor close to the rightmost point of intersection
before pressing ENTER the third time. This time the display will show that
1.447.
155. * Find the total distance traveled by the particle from time = 0
to = 2.
2
Solution: Total distance = 0 |()| . .
3 3
Notes: (1) We can compute 0 |()| = 0 |cos( 2 )| using our TI-84
calculator by first selecting fnInt( (or pressing 9) under the MATH menu,
and then going to the MATH menu again, moving right once to NUM and
selecting abs( (or pressing 1). The display will show the following:
fnInt(abs(
We then type the following: fnInt(abs(cos(X^2), X, 0, 2), and press ENTER.
The display will show 1.493441841. So we can answer 1.493.
(2) See problems 78, 92, and 126 for further explanation. Note that those
problems required a lot more work because a calculator was not
necessarily allowed.
156. * Find the position of the particle at time = 4.
4 4
Solution: (4) = 2 + 0 () = 2 + 0 cos( 2 ) . .

Notes: (1) We do not need to worry about the particle changing direction
in this problem since we only want the particles position relative to its
starting point.
(2) If we know the velocity function of a particle, then we can find its
position function by integrating the velocity . That is () = ().

(3) () = () () gives the distance between the position of
the particle at time and the position of the particle at time .
4
So 0 () is the distance between the position of the particle at time
0 and the position of the particle at time 4.

170
Since the particle started at position (0) = 2, we must add 2 to
4
0 () to get the final position.

(4) Compare this to problem 155 where we were asked to find the total
distance traveled by the particle. In that case we are concerned about
where the particle changes direction (this is why we integrate the
absolute value of the velocity instead of just the velocity), whereas here
we are only looking for the final position and therefore do not care about
changes in direction.
157. * Find all times in the interval 0 3 at which the particle
changes direction. Justify your answer.
Solution: We first solve the equation () = 0 (using our calculator), to
get 1.253, 2.171, and 2.802.
Next we observe the following:
() > 0 for 0 < 1.253 () < 0 for 1.253 < < 2.171
() > 0 for 2.171 < < 2.802 () < 0 for 2.802 < 3
It follows that the particle changes direction at = . , . , and
. .
Note: See problems 73 and 107 for more information about detecting
changes in direction.
158. Determine all values 0 for which the acceleration is 0. Then
determine all values of , 0 3, for which the particle is
speeding up.
Solution: We first solve the equation () = () = 2 sin( 2 ) = 0
(using our calculator), to get = , . , and . .
Next we observe the following:
() < 0 for 0 < < 1.772 () > 0 for 1.772 < < 2.507
() < 0 for 2.507 < 3
Lets put this information together with the information from problem
157 into one sign chart:

171
So the particle is speeding up for 1.253 < < 1.772, 2.171 < < 2.507,
and 2.802 < 3.
In interval notation, the particle is speeding up for in
(1.253,1.772) (2.171,2.507) (2.802,3].
Notes: (1) The particle is speeding up when the velocity and acceleration
have the same sign. The particle is slowing down when the velocity and
acceleration have opposite signs.
(2) See problem 107 for more details.
159. * For 0 3, find the time at which the particle is farthest
to the right. Explain your answer.
Solution: The particle changes from moving right to moving left at those
times for which () = 0 with () changing from positive to negative.
This happens at = 1.253, and = 2.802.
The particles position at = 1.253 is
1.253 1.253
2 + 0 () = 2 + 0 cos( 2 ) 2.977

The particles position at = 2.802 is


2.802 2.802
2 + 0 () = 2 + 0 cos( 2 ) 2.803

So the particle is farthest to the right at = 1.253


Note: See problem 156 for more explanation on these integrals.
160. * Find the value of the constant for which () satisfies
() cos( 2 ) 2() sin( 2 ) = 0, where () is the
acceleration of the particle at time .
Solution: We solve [2 sin( 2 )] [cos( 2 )] 2 cos( 2 ) sin( 2 ) = 0.
2 sin( 2 ) cos( 2 ) ( + 1) = 0.
So + 1 = 0, and = .

172
SUPPLEMENTAL PROBLEMS
QUESTIONS

LEVEL 1: PRECALCULUS

1. The figure above shows one cycle of the graph of the function
= sin for 0 2. If the maximum value of the
function occurs at point , then the coordinates of are

(A) ( ,1)
3

(B) ( , 0)
3

(C) ( , )
2
3
(D) ( ,1)
2

2. If and are in the domain of a function and () = (),


which of the following must be true?
(A) =
(B) fails the vertical line test.
(C) there is a horizontal line that intersects the graph of at
least twice.
(D) the graph of is a horizontal line.

173
3 2
3. If () = log 5 2 + 1, what is 1 ( ) ?
3

(A) 8.7
(B) 10.2
(C) 11.5
(D) 12.0
4. Which of the following lines is an asymptote of the graph of
(x) = 5 ln( + 2) ?
(A) = 2
(B) =0
(C) = 2
(D) =0
5. Which of the following is the graph of a function that is both even
and odd?

6. If ( 2)() = 3 5 2 + 10 8 where () is a
polynomial in , then () =
(A) +4
(B) 2 + 4
(C) 2 + 4
(D) 2 3 + 4
7. How many roots does the function () = 3 + 2 2 have?
(A) One
(B) Two
(C) Three
(D) Four

174
2 4 3 ln 2
8. If () = and () = , then (( 2)) =
2 8

(A) .12
(B) .5
(C) .51
(D) .75

LEVEL 1: DIFFERENTIATION
9. If () = 5 3 + 2 sec , then () =
(A) 15 2 sec tan
(B) 15 2 + sec tan
(C) 15 2 + 2 sec tan
5
(D) 4 + 2 ln|sec + tan |
4

+2 2
10. If () = ( ) , then (2) =
2
1
(A)
4
1
(B)
2
(C) 0
1
(D)
4
1 1
11. If () = 6 2 ln(3 2) + , then () =
6
2 1
(A) 5 +
32 2 3
6 1
(B) 5
32 2 3
1 2
(C) 5 +
6 32
1 2
(D) 7 + 2
42 ln(32)

175
12. The slope of the tangent line to the graph of = 2 at = ln 3
is
(A) 9
(B) 18
(C) 18 ln 3
(D) 18 ln 3 + 9
13. The instantaneous rate of change at = 1 of the function
() = ln(2 2 1) is
(A) 4
(B) 2
5
(C)
4
1
(D)
2

2 5
14. [ +3 + ] =
7 4

15. If = cot , then =


(2) log3
16. Differentiate () = 3 and express your answer as a

simple fraction.

LEVEL 1: INTEGRATION
17. (3 2 6 + ) =
3
(A) 6 + +

(B) 3 4 3 + +
(C) 3 3 + +
(D) 3 3 + 1 +

18. 0 (3 2 2 cos ) =

(A) 3
(B) 3 2
(C) 6
(D) 6 2

176
2 +ln
19. =
(A) 2 ln +
1 2
(B) +
2
2
(C) +
2
(D) 2 +
5 +2
20. =

1 2 + 3
21. 0 =
3 + 3
3 5 3 3
22. ( 5 + 5 + 5 ) =
4

23. 2 =
(3 +1) ln(3 2 +1)

24. 3 cos ( sin cos ) =

LEVEL 1: LIMITS AND CONTINUITY


2 310
25. What value does approach as approaches 2?
5+10
7
(A)
5
(B) 1
5
(C)
7
(D) 5
3 2 1
26. lim =
25
1
(A)
2
3
(B)
2
(C)
(D) The limit does not exist

177
2 21
27. The graph of the rational function where () = 2 has
4
asymptotes = , = , and = . What is the value of
a+b+c?
1
(A)
4
1
(B)
4
(C) 1
(D) 3
28. If the function is continuous for all real numbers and if
3 +2 2 15
() = for all 3, then (3) =
3
(A) 12
(B) 24
(C) +
(D) (3) does not exist.
tan4 2
29. lim
0 4
5 sin 5 sec2 sin
30. lim
0 3

31. Let be a function such that lim () = () for all in the



interval [, ]. Which of the following must be true?
(A) is continuous on (, )
(B) is differentiable on (, )
(C) () = () ()
1
(D) ()

= ()

178
32. The graph of the function is shown in the figure above. Which
of the following statements about is false?
(A) lim () =

(B) lim+ () = ()

(C) lim () =

(D) lim () does not exist

LEVEL 2: PRECALCULUS

33. The figures above show the graphs of the functions and . The
function is defined by () = 2| + 2| and the function is
defined by () = ( + ) + , where and are constants.
What is the value of | |?
(A) 5
(B) 4
(C) 3
(D) 2

179
3
34. If () = ( + 3) , for all > 0, then which of the following
functions is equal to 1 () when restricted to > 27 ?
3
(A)
3
(B) 3
3 2
(C) ( 3)
(D) ( 3)2
1
35. What is the range of the function defined by 2?
2

(A) All real numbers


(B) All real numbers except 0
(C) All real numbers except 2
(D) All real numbers greater than 2
36. If > 0 and > 1, and log 2 = 4, then which of the
following can be FALSE?
(A) log = 2
(B) log 2 =
(C) log = 2

(D) log =1
2

37. What is the range of the following function?


() = 4 sin(2 + ) + 3
(A) 7 1
(B) 1 7
(C) 1 4
(D) 4 7

38. Consider the functions () = 1 and () = + . In


the standard (, ) coordinate plane, = (()) passes through
(0, 1) and (2,3). What is the value of + ?
(A) 1
(B) 2
(C) 4
(D) 6

180
3
39. What is the period of the graph of y = 4 sec( 1) ?
7

(A)
3
14
(B)
3
14
(C)
3
14
(D)
15

40. If arccos(cos ) = and 0 2, then could equal
4

(A) 0

(B)
6
3
(C)
4
7
(D)
4

LEVEL 2: DIFFERENTIATION
2
41. If = tan , then =

42. If () = cos 2 (2 ) sin(3 6), then (2) is equal to

181
43. The function , whose graph consists of three line segments, is
shown above. Which of the following are false for on the open
interval (, ) ?
I. has a nonremoval discontinuity at = .
II. is differentiable on the open interval (, ).
III. The derivative of is negative on the interval (, 0).
(A) I only
(B) II only
(C) III only
(D) I and III only
3
44. If () = , then () =
2 2 +5

45. The slope of the tangent line to the curve + = 3 at the


point (1,0) is
46. Let () = ln . A value of that satisfies the conclusion of the
Mean Value Theorem for on the interval [, 2 ] is
3
2 +
47. If () = 3 , then (8) =
2

48. The derivative of () = attains its minimum value at =

LEVEL 2: INTEGRATION
ln 3
49. 0 =
1+ 2

50. 04 sec (tan sec )
2 3 +2 2 2
51. 0 =
2 1

4
52. The solution to the differential equation = 2 , where y(1) = 1,

is

182
53. Each of the following is an antiderivative of cos sin EXCEPT
cos2
(A)
2
sin2
(B)
2
1+sin2
(C)
2
cos2
(D)
2
5
54. =
3 2

55. The area of the region enclosed by the graph of = 2 2 + 3 and


the line = 7 is
3
56. A radioactive substance is decaying at a rate of 30 5 per year.
At = 0 years, there is 100 pounds of this substance. Find the
amount present after 5 years.

LEVEL 2: LIMITS AND CONTINUITY


ln(+)ln
57. What is lim ?
0
1 1
2
58. lim (3 2 ) =

1 5+
59. lim log 3 ( ) is equal to
0 5

ln||
1 , for 0.
60. Let the function be defined by () = {
0 , for = 0.
Which of the following are true about f ?
I. lim () exists.
0
II. (0) exists.
III. is continuous at = 0.
(A) I only
(B) 1I only
(C) I and II only
(D) I, II, and III

183
61. Suppose that the differentiable function satisfies
(3+) (3)
lim = 1. Which of the following must be true ?
0

I. is continuous at = 3
II. is continuous at = 3
III. (3) exists
(A) I only
(B) 1I only
(C) I and II only
(D) I, II, and III
7 6 +3 8
62. lim =
65 2 +2 4
+2
63. lim =
11 (11)3

64. Let be the function defined by


5 ln| 3| + 15
, 2
1 + cos( 2)
() =
15 sin( )
4 , >2
{ 2
Is continuous at = 2 ?

LEVEL 3: PRECALCULUS

65. If cot = , then for all in the interval 0 < < , sin =
2
1
(A)
1+

(B)
1+ 2
1
(C)
1+ 2
1
(D)
1 2

184
66. Suppose the graph of () = 2 3 is translated 2 units down and
3 units left. If the resulting graph represents (), what is the
value of (.5) ?
(A) 33.25
(B) 28.75
(C) 17.25
(D) 11.5
1
67. If () = 3 and () = , then the domain of is
2 1

(A) all real numbers


(B) all such that 3
(C) all such that 2
(D) all such that 2 and 3

68. If 0 < < , then sec(arctan ) =
2
1
(A)

1
(B)
2 +1

(C) 2 + 1
1
(D)
1 2

LEVEL 3: DIFFERENTIATION

69. At which of the five points on the graph in the figure above are
2
and both negative?
2

(A)
(B)
(C)
(D)

185
70. Given the function defined by () = 2 , find all values of
for which the graph of is concave up.
71. If the line 2 3 = 3 is tangent in the first quadrant to the curve
= ln(3 + 1), then is
72. Find all relative extrema and points of inflection for the function
defined by () = ln( 2 + 1).

73. The graph of () is given above for 0 12. On the same


set of axes, sketch the graph of .
1
3
74. If () = 1 for all , then the domain of is
(+3)2

2
75. [cot 1 ( )] =
4

76. The derivative of is graphed above. Give all -values at which


has local extrema and points of inflection.

186

77. If sin( 3 ) = , then =

78. A point moves in a straight line so that its distance at time from
2 1
a fixed point of the line is . The total distance that the point

travels from = 0 to = 3 is
79. * Two particles start at the origin and move along the -axis. For
0 2, their position functions are given by = tan2 and
2
= +1 . For how many values of do the particles have the
same velocity?
(A) None
(B) One
(C) Two
(D) Three
2
80. Consider the equation ln() = 1. Find at (1,1).
2

LEVEL 3: INTEGRATION
81. The area of the region in the first quadrant bounded by the graph
ln(2+1)
of = , the -axis, and the line = 1 is
4+2

82. A particle moves in a straight line with velocity


2 +2+1(+1)3
() = beginning at time = 0. How far is the
+1
particle from its starting point at time = 3 ?
83. The acceleration () of a body moving in a straight line, in

ft/sec 2 is given in terms of time by () = 5 cos . If at
2
time = 0 seconds, the body is moving to the right at a speed of
3 ft/sec, and if () is the distance of the body from the origin at
time , then (2) (0) =
84. The area of the region completely bounded by the curves
= 2 + 3 2 and = 2 1 is
85. Let and be continuous functions such that () = () for

all . It follows that 2 ( 2 + 1) =

86. If = and if = 4 when = 0, then =
+1

187

87. The average value of tan over the interval is
6 3
1
88. Evaluate 1 () , where
2

cos sin2 for < 0


() = {
esin cos for 0
2
89. If 2 () = , then (5(2) 3) =

1 ln[()] ()
90. If (0) = 1, (1) = , then 0 =
()

91. Calculate the approximate area under the curve () = log 2


and bounded by the lines = 1 and = 4 by the trapezoidal rule,
using three equal subintervals.
92. A point moves in a straight line so that its velocity at time is
1
. What is the total distance that the point travels from t
2 215
= 0 to t = 2 ?

LEVEL 3: LIMITS AND CONTINUITY


93. If is continuous for all in the interval (, ), which of the
following can be false?
(A) () is defined for all in (, )
(B) lim () = ( lim ) for all in (, )

(C) is Riemann integrable on (, ).
()()
(D) There is a in (, ) such that () = .

94. Let be defined by


cos for 2
() = {
for < 10 .
sin(+ )
2

Is continuous at = ? Use the definition of continuity to


explain your answer.

188
95. A 5000-gallon tank is filled to capacity with water. At time
= 0, water begins to leak out of the tank at a rate modeled by
(), measured in gallons per hour, where
300
, 04
() = { + 1
12 20.5 + 228, > 4

Is continuous at = 4? Show the work that leads to your

answer.
3 2 2 3
96. Let () = 3 2 . Find each -value at which is
2 2 4
discontinuous, and classify each such discontinuity as removable
or nonremovable. Define a function such that is continuous
at each removable discontinuity of and such that () = ()
for all in the domain of .

LEVEL 4: DIFFERENTIATION
97. Suppose that is an odd function (so that () = () for all
), and that () exists. Then () must be equal to
(A) ()
(B) ()
1
(C)
()
1
(D)
()

98. The function () = 2 3 + 3 has an inverse function . Find


(15).
99. A container is in the form of a right circular cone with radius 4
inches, height 16 inches, vertex pointing downward. Water is
being poured into the container at the constant rate of 16 inches3
per second. How fast is the water level rising when the water is 8
inches deep?
73
100. If = sin , where is a nonzero constant, then =
73

189
101. The radius of a circle increases from 10 inches to 10.1 inches. Use
a linear approximation to find the approximate increase in the
circles area.

4.2 4.3 4.4 4.5


() 3 3.1 3.3 3.7

102. Let be a function that is concave up for all in the closed


interval [4,5], with selected values shown in the above table.
Which of the following inequalities must be true?

(A) (4.3) > 3


(B) 2 < (4.3) < 3
(C) 1 < (4.3) < 2
(D) 0 < (4.3) < 1

103. Consider the differential equation = Let = () be the

particular solution to the differential equation that passes through
(0,0). Write an equation for the line tangent to the graph of at
the point (0,0), and use the tangent line to approximate (0.1).

104. A bus is traveling on a straight road. For 0 15 seconds, the


buss velocity (), in feet per second, is modeled by the function
defined by the graph above. Find the average rate of change of
over the interval 1 6. Does the Mean Value Theorem
guarantee a value of for 1 < < 6, such that () is equal to
this average rate of change? Why or why not?

190

105. Consider the differential equation = ( + 1)2 sin . On the
2
axes provided above, sketch a slope field for the differential
equation at the twelve points indicated. There is a horizontal line
with equation = that satisfies his differential equation. Find
the value of .

106. The graph of the derivative of a function is shown above. For


each of (), (), (), (), (), and (), find the
value or explain why it does not exist.

107. Two particles are moving along the -axis. For 0 5, the
position of particle at time is given by () = cos , and the
position of particle at time is given by
() = 3 6 2 + 9 + 2. For 0 5, find all times during
which the two particles travel in the same direction.

191
0 0<<1 1 1<<2 2 2<<3 3
() 4 3 1 2
() 3 + 0 + 0 5

108. The differentiable function is defined for all real numbers .


Values of and for various values of are given in the table
above. Find the -coordinate of each relative minimum of on
the interval [0,3]. Justify your answers.

109. Consider a differentiable function with domain (, ),


3
satisfying () = ( 2 7). Find the -coordinates of all
relative minima and maxima, find all intervals on which the graph
of is concave up, and find all intervals on which the graph of
is concave down. Justify your answers.

110. Let be a function with derivative () = 2 + 7 such that


(3) = 1. Estimate (3.1) and determine if this estimate is too
large or too small. Justify your answer.

111. Joe is driving his car along a straight road. For 0 15


seconds, his velocity (), in miles per minute, is modeled by the
function defined by the graph above. Let () be the cars
acceleration at time , in miles per minute. For 0 < < 15, write
a piecewise-defined function for ().

192
112. Does there exist a differentiable function such that
(3) = 3, (5) = 11, and () < 1 for all x ? Justify your
answer.

LEVEL 4: INTEGRATION
113. The region in the -plane bounded by the graphs of = 2 and
= 2 is rotated about the line = 1. What is the volume of
the solid generated?
114. * The rate, in tons per hour, at which bricks are arriving at a
3
construction site is modeled by () = 5 + 32 sin ( ), where
128
is measured in hours and 0 5. Approximately how many
tons of bricks will arrive at the site during the 5 hours? Round
your answer to the nearest ton.
115. Find the average value of 2 cos 3 on the interval [0,2].
3 cos
116. * If () = and (1) = 3, then (2) =

117. Let be the region in the first quadrant bounded by = 5 and


= 10. Write, but do not evaluate, an integral expression that
gives the volume of the solid generated when is revolved
around the line = 2.

118. * The function defined by () = 0 (u5 3u2 + 2) on the
closed interval [0,5], has a local minimum at = and a local
maximum at = . Find and .

1 3 3 1 3 2 3 99
119. The expression ( 0 + + + + ) is a Riemann
75 75 75 75
1
sum approximation for 0 () for () =

193
(minutes) 0 2 4 6 8
()
110 100 92 86 84
(degrees F)
120. As a cup of hot water cools, the temperature of the water is
modeled by a continuous function , where is measured in
minutes. Selected values of (), measured in degrees
Fahrenheit, are given in the table above. Use a trapezoidal sum
with four subintervals indicated by the given data to approximate
1 8
8
0 () . Using correct units, explain the meaning of
1 8
()
8 0
in the context of the problem.

121. Consider the differential equation = 2 3 Let = () be

the particular solution to the differential equation that passes
through (2,1). Find = (), and the domain of .

0 1 2 3
() 2 3 0 4
() 3 3 4 1
() 1 2 3 4
() 2 1 3 1

122. The differentiable functions and are defined for all real
numbers . Values of , , , and for various values of are
3
2
given in the table above. Evaluate 1 2 (( 3 ))( 3 ) .

194
123. Let be the continuous function defined on [5,8] whose
graph, consisting of three line segments and a semicircle
8
centered at (4,0), is shown above. Compute 5 () .

124. * The function is defined for > 0 by


1
() = cos( + ) and (3) = 1. Write an equation for

the line tangent to the graph of at = 3.2.
2
125. * Let () = 0 sin3 . Find the average value of ()
on the interval [0,2].
126. * The rate, in tons per hour, at which bricks are arriving at a
3
construction site is modeled by () = 5 + 32 sin ( ), where
128
is measured in hours and 0 5. At time = 0, there are
already 10 tons of unused bricks at the site, and bricks are being
used (for the construction) at a constant rate of 7 tons per hour.
Write an expression for the amount of unused bricks at the
construction site at any time . Then find the maximum amount
(in tons) of unused bricks at the site for 0 3. Justify your
answer.

1 5 6 8 11
() 3 5 1 -3 4

127. Let be a function that is continuous for all real numbers. The
table above gives values of for selected points in the closed
interval [1,11]. Use a right Riemann Sum with subintervals
11
indicated by the data in the table to approximate 1 () .
Show the work that leads to your answer.

195
128. * Let be the region in the first and second quadrants bounded
48
above by the graph of = and below by the horizontal line
2+ 4
= 4. The region is the base of a solid whose cross sections
perpendicular to the -axis are rectangles with height twice the
length of the base. Find the volume of this solid.

LEVEL 5: FREE RESPONSE QUESTIONS


129 133 Consider the differential equation = 2 + 1 and the

figure above.
129. On the axes provided, sketch a slope field for the differential
equation at the nine points indicated. Then describe all points in

the -plane, 0, for which = 1.

130. Write an equation for the line tangent to the solution curve at the
point (0,1). Use the equation to approximate (0.5) where
= () is the particular solution of the differential equation
with initial condition (0) = 1.
2
131. Find in terms of and . Describe the region in the
2
-plane in which all solution curves to the differential equation
are concave down.
132. Let = () be a particular solution to the differential equation
with the initial condition (1) = 1. Does have a relative
minimum, a relative maximum, or neither at = 1 ? Justify your
answer.
133. Find the values of the constants and , for which = +
is a solution to the differential equation.

196
134 137 * Machine removes salt from a container at a rate modeled
3
by the function , given by () = 3 + 2 cos ( ). Simultaneously,
20
machine adds salt to the container at a rate modeled by the function

, given by () = . Both () and () have units of cubic feet
2+
per hour and is measured in hours for 0 5. At time = 0, the
container contains 20 cubic feet of salt.
134. * How much salt will machine remove from the container
during this 5 hour period, and how much salt will machine add
to the container during this 5 hour period? Indicate units of
measure.
135. Write an expression for (), the total number of cubic feet of
salt in the container at time .
136. * Find the rate at which the total amount of salt in the container
is changing at time = 2.
137. For 0 5, at what time is the amount of salt in the container
a minimum? At what time is the amount of the salt in the
container a maximum? What are the maximum and minimum
values? Justify your answers.

138 145 Let be the continuous function defined on [5,5] whose


graph, consisting of three line segments, a quarter circle centered at the
origin, and a semicircle centered at (4,0), is shown above. Let

() = 3 + 0 () .

138. Find the values of (2) and (5).


139. Find (). Then evaluate (1) and (1).
140. Determine the -coordinate of the point at which has an
absolute maximum on the interval [5,5]. Justify your answer.

197
141. For 5 < < 5, find all values of for which the graph of has
a point of inflection. Explain your reasoning.
142. On what open intervals contained in 5 < < 5 is the graph of
concave up? Justify your answer.
143. Find the average rate of change of on the interval [5, 2].
There is no point with 5 < < 2, for which () is equal
to that average rate of change. Explain why this statement does
not contradict the Mean Value Theorem.
144. Suppose that is defined for all real numbers and is periodic
with a period of length 10. The graph above shows one period of
. Find (10).
145. The function is defined by () = (ln()). Find an equation
1
of the tangent line to the graph of at the point where = .

146 151 * Let be the region bounded by the graphs of


() = ln and () = 3.

146. * Write an equation for the tangent line to the graph of at


= 2.
147. * Find the area of .
148. * Find the volume of the solid generated when is rotated about
the horizontal line = 5.
149. * Find the volume of the solid generated when is rotated about
the -axis.
150. * The region is the base of a solid. For this solid, each cross
section perpendicular to the -axis is a square. Find the volume
of the solid.
151. The horizontal line = divides into two regions with equal
areas. Write, but do not solve, an equation involving integral
expressions whose solution gives the value .

198
152 155 Let be a twice-differentiable function that is defined for
all real numbers and satisfies the following conditions:
(1) = 3, (1) = 5, (1) = 2, (3) = 2, (3) = 1, (3) = 0
152. Let satisfy 2 < < 3 and let satisfy 1 < < 5. Explain why
there must be real numbers and with 1 < , < 3 such that
()() = .
153. Is there a value for 1 < < 3 such that () = 2 ? Justify
your answer.
154. The function is given by () = () + for all real ,
with a constant. Find (1) in terms of .
155. The function is given by () = sin() () for all real ,
with an integer. Write an equation for the line tangent to the
graph of at = 3.
156 160 A particle moves along the -axis so that its acceleration at
ln
time > 0 is given by () = . At time = 1, the velocity of the

particle is (1) = 8 and the position of the particle is (1) = 9.
156. Write an expression for the velocity of the particle ().
157. Find all values of > 0 for which the speed of the particle is 6.
For each such value of determine if the particle is moving to the
right or to the left.
158. Set up, but do not evaluate an expression involving an integral
that gives the total distance traveled by the particle from time
= 1 to = 2.
159. Find all times 0 at which the particle changes direction.
Justify your answer.
160. Determine all values 0 for which the particle is speeding up,
and all values 0 for which the particle is slowing down.

199
ANSWERS TO
SUPPLEMENTAL PROBLEMS

LEVEL 1: PRECALCULUS
1. D
2. C
3. D
4. C
5. D
6. D
7. C
8. D

LEVEL 1: DIFFERENTIATION
9. A
10. C
11. B
12. D
13. A
20
14. 3 7 + (1 + ln )
3
sin cos ln
15. cot ( )
sin2

3[(2)+ (ln 3) log3 ](2)(ln 3) log3


16. 3
3(ln 3) 4

LEVEL 1: INTEGRATION
17. B
18. A
19. B
2 11 4
20. + 3 +
11 3
ln(1+ 3 )
21. +
3

200
3
3 15 4 5
22. + 5 ln|| + 15 +
4 4 4

ln(ln(32 +1))
23. +
6
3 cos
24. = +
ln 3

LEVEL 1: LIMITS AND CONTINUITY


25. A
26. B
27. C
28. B
29. 16
30. 5
31. A
32. D

LEVEL 2: PRECALCULUS
33. A
34. C
35. D
36. D
37. B
38. D
39. B
40. D

LEVEL 2: DIFFERENTIATION
2
41. 2 tan tan sec 2
42. 3
43. D
15
44.
(2 2 +5)3

45. 1

46. 1

201
127
47.
48

48. 2

LEVEL 2: INTEGRATION

49.
12

50. 2 2
51. 6
3
52. =
32
53. A
54. 53 2 +
162
55.
3
56. 50 3 + 50 pounds

LEVEL 2: LIMITS AND CONTINUITY


1
57.

58. 3
1
59.
5 ln 3
60. D
61. D
3
62.
2
63.
64. No

LEVEL 3: PRECALCULUS
65. C
66. A
67. D
68. C

202
LEVEL 3: DIFFERENTIATION
69. A
70. (, 2 2) (2 + 2, )
7
71.
6
72. rel. max: (0,0), points of inflection: (1, ln 2)
73.

74. {| > 3 and 0}


8
75.
4 +16
76. local max at = 1, local min at = 6, pt of inf at = 4
3 cos(3 )
77.
32 cos(3 )
3 +8
78.
3
79. B
80. 2

LEVEL 3: INTEGRATION
1
81. (ln 3)2
8

203
17
82.
6
83. 5 2 13 4 ft.
1
84.
24
85. ( 2 + 1) (2 + 1)
86. 2( + 1)
3
87. ln 3

1
88.
3
5
89. + 3 3
2
1
90.
2
91. 2 + log 2 3
16
92. ln
15

LEVEL 3: LIMITS AND CONTINUITY


93. D
94. Yes, () = lim () = lim+ () =

95. Yes, , (4) = lim () = lim+ () = 240
4 4
96. = 1 (removable), = 0 (removable), = 2 (nonremovable)
3 2 2 3
, 1, 0
2 3 2 2 4
2
() = , = 1
3
3
{ , =0
4

LEVEL 4: DIFFERENTIATION
97. A
1
98.
25
4
99. in/sec

100. 73 cos

204
101. 2 in2
102. C
103. = , (0.1) .1
104. 5, no because is not differentiable at = 3
105. = 1


106. () = , () = , () = , () =

()+ > 0, () < 0. So () does not exist,
()+ < 0, () > 0. So () does not exist
107. (0,1) (3, )
108. There are no relative minima. The critical numbers of are
= 1 and = 2. At = 1, does not change sign, and
at = 2, changes sign from positive to negative
indicating a relative maximum.
109. rel max at = 0, rel min at = 7 and = 7, concave
up on (, 1), (1, ), concave down on (1,0), (0,1)
110. (3.1) 0.6. The estimate is too small because
() > 0 for > 0.
111.
3
if 0 < < 2
20
3
if 2 < < 6
20
3
() = if 6 < < 8
20
7
if 8 < < 10
20
0 if 10 < < 11
7
{ 40 if 11 < < 15

205
112. No. By the Mean Value Theorem there must exist a , 3 <
113
< 5 such that () = = 1.
5(3)

LEVEL 4: INTEGRATION
29
113.
30
114. 62
1
115. sin 8
6
116. 4.467
1 2 2
2
117. 04[( + 2) ( + 2) ]
10 25
118. 1.121, = 1
3
119.
1 8
120. 0 () 93.75
8
1 8
()
8 0
is the average temperature of the water, in
degrees Fahrenheit, over the 8 minutes.
1 3 3
121. = , <<
92 2 2 2
5
122.
3
5
123. + 2
2

124. .808 = .932( 3.2) or = .932 + 3.79


125. 0.614

126. () = 10 + 0 (() 7)
() = () 7 = 0 when 2, (0) = 10, (2) 7,
(3) 9. So the maximum amount is 10 tons of bricks.
127. 27
128. 1447.428

206
LEVEL 5: FREE RESPONSE QUESTIONS

129. = 1 = 2

130. = 2 + 1, (.05) 0
2
131. = 2 1, Solution curves will be concave down on
2
the half-plane below the line = 2 + 1.

132. has a relative maximum at = 1 because |(1,1) = 0

2
and |(1,1) = 2 < 0.
2

133. = 2, = 1
5 5
134. 0 () = 18.001 ft 3 , 0 () = 25.247 ft 3

135. () = 20 + 0 (() ())
136. decreasing at a rate of 2.328 ft 3 /hr
137. () = 0 when () = (), ie. when 2.82,
(0) = 20, (2.82) = 11.514, (5) = 27.246. So the
amount of salt is a minimum when 2.82 hours, and the
amount of salt is a maximum when = 5 hours. The
minimum value is 11.514 ft 3 and the maximum value is
27.246 ft 3 .
760
138. (2) = 6, (5) =
4
() (1)
139. = 3 + (), = 3, (1) = 3

207
140. () = 0 when () = 3. This occurs at = 0. We also
need to check the endpoints, = 5 and = 5.
760
(5) = 18, (0) = 0, (5) = . So has an
4
absolute maximum at = 5.
141. The graph of has a point of inflection at = 0 and = 4
because = changes sign at each of these values.
142. is concave up when = > 0, ie. when is
increasing. This occurs on the intervals (2,0) and (4,5).
143. Average rate of change is 2. The statement does not
contradict the Mean value Theorem because is not
differentiable at = 3.
7132
144.
4
145. = 3 3
146. .693 = 0.5( 2) or = 0.5 .307
147. 5.694
148. 163.542
149. 63.881
150. 8.945
1.505
151. 2.948( + 3 ) = ( + 3 )

152. Since is twice-differentiable, and are continuous.


Since is a continuous function with (3) < < (1),
the Intermediate Value Theorem guarantees that there is a
value , with 1 < < 3, such that () = . Since is a
continuous function with (3) < < (1), the
Intermediate Value Theorem guarantees that there is a
value , with 1 < < 3, such that () = . Multiplying
these two equations yields ()() = .
153. Yes. Since is twice-differentiable, is differentiable. In
particular, is continuous on [1,3] and differentiable on
(1,3). The Mean Value Theorem guarantees that there is a
(3) (1)
value , 1 < < 3 such that () = = 2.
31
154. 2 + ( + 2)
155. = 2( 3)

208
(ln ) 2
156. () = 8
2
157. The particle is moving to the left at = 2 and = 2, and
moving to the right at = 27 and = 27 .
2 (ln )2
158. 9 + 1 | 8|
2
159. () = 0 when = 4 and = 4 .
() > 0 for 0 < < 4 and > 4
() < 0 for 4 < < 4
It follows that the particle changes direction at = 4
and = 4 .
160. The particle is speeding up for 4 < < 1 and > 4 . The
particle is slowing down for 0 < < 4 and 1 < < 4

209
. About the Author
Dr. Steve Warner, a New York native, earned his Ph.D. at Rutgers
University in Pure Mathematics in May, 2001. While a graduate student,
Dr. Warner won the TA Teaching
Excellence Award.
After Rutgers, Dr. Warner joined the
Penn State Mathematics Department as
an Assistant Professor. In September,
2002, Dr. Warner returned to New York
to accept an Assistant Professor position
at Hofstra University. By September
2007, Dr. Warner had received tenure
and was promoted to Associate
Professor. He has taught undergraduate
and graduate courses in Precalculus, Calculus, Linear Algebra, Differential
Equations, Mathematical Logic, Set Theory and Abstract Algebra.
Over that time, Dr. Warner participated in a five year NSF grant, The
MSTP Project, to study and improve mathematics and science curriculum
in poorly performing junior high schools. He also published several articles
in scholarly journals, specifically on Mathematical Logic.
Dr. Warner has more than 15 years of experience in general math tutoring
and tutoring for standardized tests such as the SAT, ACT and AP Calculus
exams. He has tutored students both individually and in group settings.
In February, 2010 Dr. Warner released his first SAT prep book The 32
Most Effective SAT Math Strategies, and in 2012 founded Get 800 Test
Prep. Since then Dr. Warner has written books for the SAT, ACT, GRE, SAT
Math Subject Tests and AP Calculus exams.

Dr. Steve Warner can be reached at

steve@SATPrepGet800.com

210
BOOKS BY DR. STEVE WARNER

211
CONNECT WITH DR. STEVE WARNER

212

Vous aimerez peut-être aussi